Slot System
Featured Buckets
Featured Buckets Admin

Should you still recommend omega-3 supplements?

Article Type
Changed
Tue, 07/14/2020 - 15:06
Display Headline
Should you still recommend omega-3 supplements?
PRACTICE CHANGER

Stop recommending omega-3 fatty acid supplements for cardiovascular protection. They have no significant impact on all-cause mortality, acute myocardial infarction, sudden death, or stroke.1

Strength of Recommendation

A: Based on a meta-analysis of randomized controlled trials (RCTs).

Rizos E, Ntzani E, Bika E, et al. Association between omega-3 fatty acid supplementation and risk of major cardiovascular disease events: a systematic review and meta-analysis. JAMA. 2012;308:1024-1033.

 

ILLUSTRATIVE CASE

A 59–year-old patient who had a myocardial infarction (mi) 3 years ago is taking an ace inhibitor, a statin, and a b-blocker. He asks you whether he should also take omega-3 fatty acid supplements to further decrease his risk of heart disease. What should you tell him?

Coronary artery disease (CAD) kills more than 500,000 Americans every year2, and medical and dietary therapies for primary and secondary cardiovascular protection are paramount. Omega-3 polyunsaturated fatty acid (PUFA) supplementation is one such therapy. Omega-3 PUFAs are precursors to certain prostaglandins that decrease the proinflammatory state in patients with CAD. They also lower triglyceride levels and produce an antiarrhythmic effect by promoting electrical stability.

But do PUFA supplements provide cardioprotection?
The American Heart Association’s Nutrition Committee recommends either omega-3 PUFA supplementation with 250 to 500 mg eicosapentaenoic acid (EPA) and docosahexaenoic acid (DHA) per day or 2 servings of oily fish per week for both primary and secondary prevention of CAD.3 The European Society of Cardiology also encourages increased consumption of oily fish.4

These recommendations are based on primary and secondary prevention studies, performed between 1989 and 2007, that found a 15% to 29% decrease in all-cause mortality and nonfatal cardiovascular events associated with regular intake of omega-3 fatty acids.5-7 The systematic review and meta-analysis detailed below revisited the effect of omega-3 supplementation on major cardiovascular outcomes.1

STUDY SUMMARY: Omega-3 supplements don’t lower cardiovascular risk

This meta-analysis included 20 RCTs with a total of 68,680 patients. The median age was 68 years, with a range from 49 to 70 years. Thirteen of the studies evaluated omega-3 PUFAs for secondary prevention of cardiovascular outcomes, 4 assessed both primary and secondary prevention, and 3 looked at outcomes in patients with implantable cardioverter defibrillators. All lasted longer than one year, and most were high quality, with a low risk of bias.

The median treatment duration was 2 years, with a maximum of 6.2 years. The mean omega-3 PUFA dose evaluated in the studies was 1.5 g per day, with the exception of 2 studies in which patients received omega-3 PUFAs through dietary sources. Twelve studies used a dose of 1 g or more per day. Half of the included trials were performed during the period when statins were routinely prescribed for cardiovascular risk modification (1998 or later).

Outcomes included all-cause mortality (17 studies), cardiac death (13 studies), sudden death (7 studies), MI (13 studies), and stroke (9 studies).

This meta-analysis found trends toward a decrease in all-cause mortality, cardiac death, sudden death, and MI in patients taking omega-3 PUFAs, but no statistically significant association between any of the outcomes and omega-3 PUFA supplementation. The relative risk for all-cause mortality was 0.96 (95% confidence interval, 0.91-1.02; P=.17). Prespecified subgroup analysis found no association between treatment effect and omega-3 fatty acid dose.

Are dietary sources of omega-3s more effective?
In the 2 trials involving dietary supplementation with omega-3 PUFAs, the results for all-cause mortality and cardiac death were conflicting, with one showing an increase in all-cause mortality and cardiac death and the other showing a decrease in both outcomes compared with the control group. No harmful effects of omega-3 PUFAs were found in either the supplement- or diet-based studies. 

 

 

 

WHAT’S NEW: More evidence of little benefit 

The meta-analysis by Rizos et al is the most up-to-date, comprehensive look at the value of omega-3 fatty acids for primary and secondary prevention of cardiovascular events.It differs from previous reviews in that most included studies were well-done RCTs. In addition, the studies were performed in both primary and secondary cardiovascular disease prevention settings and involved different forms of omega-3 PUFA supplementation, including dietary sources and supplements. The trials were predominantly larger than those included in previous systematic reviews, as well. The baseline risk for cardiovascular disease in the newer studies (7 of the 20 RCTs were completed after 2007) may be different from that of previous studies because of increased use of certain medications,such as statins. In recent years, other studies of omega-3 PUFAs have had similar results. A metaanalysis of 14 RCTs found that omega-3 PUFA supplementation offered no benefit for the secondary prevention of cardiovascular disease.8 The FORWARD trial—published earlier this year—showed that omega-3 PUFAs did not decrease the recurrence of atrial fibrillationin patients with a history of confirmed paroxysmal atrial fibrillation.9 And an earlier(2006) analysis of RCTs and cohort studies found no benefit from omega-3 fatty acids for primary prevention of cardiovascular disease or cancer.10

CAVEATS: No significant help, and no harm

There’s no need to tell patients who wish to take omega-3 supplements not to do so, but we should not promote their use for the sole purpose of cardiovascular disease protection.

While this meta-analysis found no statistically significant benefits from omega-3 PUFAs, there is no evidence of harm from PUFA intake, whether from dietary sources or supplements. There is no need to tell patients who wish to take omega-3 supplements not to do so. But we should not promote their use for the sole purpose of cardiovascular disease prevention.

CHALLENGES TO IMPLEMENTATION: Changing minds won’t be easy

Despite recent findings indicating that omega-3 PUFAs provide little primary or secondary protection against cardiovascular events, advertising from supplement manufacturers may make it hard to change patients’ minds. Because diets and supplements containing these fatty acids do not cause apparent harm, patients and physicians may decide that a small potential benefit is worth the expense.

Acknowledgement

The PURLs Surveillance System was supported in part by Grant Number UL1RR024999 from the National Center for Research Resources, a Clinical Translational Science Award to the University of Chicago. The content is solely the responsibility of the authors and does not necessarily represent the official views of the National Center for Research Resources or the National Institutes of Health.

Files
References

1. Rizos E, Ntzani E, Bika E, et al. Association between omega-3 fatty acid supplementation and risk of major cardiovascular disease events: a systemic review and meta-analysis. JAMA.  2012;308:1024-1033.

2. Centers for Disease Control and Prevention. FastStats. Deaths and mortality. 2010. Available at: http://www.cdc.gov/nchs/fastats/deaths.htm. Updated April 5, 2013. Accessed July 21, 2013.

3. Kris-Etherton PM, Harris WS, Appel LJ, et al. Fish consumption, fish oil, omega-3 fatty acids and cardiovascular disease. Circulation.  2002;106:2747-2757.

4. Van de Werf F, Bax J, Betriu A, et al. Management of acute myocardial infarction in patients presenting with persistent ST-segment elevation: the Task Force on the management of ST-segment elevation acute myocardial infarction of the European Society of Cardiology. Eur Heart J. 2008;29:2909-2945.

5. Artham SM, Lavie CJ, Milani RV, et al. Fish oil in primary and secondary cardiovascular prevention. Ochsner J. 2008;8:49-60.

6. Gruppo Italiano per lo Studio della Sopravvivenza nell’Infarto miocardico. Dietary supplementation with n-3 polyunsaturated fatty acids and vitamin E after myocardial infarction: results of GISSI-Prevenzione trial. Lancet. 1999;354:447-455.

7. Yokoyama M, Origasa H, Matsuzaki M, et al. Effects of eicosapentaenoic acid on major coronary events in hypercholesteroleamic patients (JELIS): a randomized open-label, blinded endpoint analysis. Lancet. 2007;369:1090-1098.

8. Kwak SM, Myung SK, Lee YJ, et al. Efficacy of omega-3 fatty acid supplements (eicosapentaenoic acid and docosahexaenoic acid) in the secondary prevention of cardiovascular disease: a metaanalysis of randomized, double-blind, placebo-controlled trials.  Arch Intern Med. 2012;172:686-694.

9. Macchia A, Grancelli H, Varini S, et al. Omega-3 fatty acids for the prevention of recurrent symptomatic atrial fibrillation: results of the FORWARD (Randomized Trial to Assess Efficacy of PUFA for the Maintenance of Sinus Rhythm in Persistent Atrial Fibrillation) trial. J Am Coll Cardiol. 2013;61:463-468.

10. Hooper L, Thompson RL, Harrison RA, et al. Risks and benefits of omega 3 fats for mortality, cardiovascular disease, and cancer: systemic review. BMJ. 2006;332:752-760.

Article PDF
Author and Disclosure Information

Julie Monaco, MD;
Anne Mounsey, MD;
Jennifer Bello Kottenstette, MD

Department of Family Medicine, University of North Carolina at Chapel Hill (Drs. Monaco and Mounsey); Department of Family Medicine, The University of Chicago (Dr. Kottenstette)

PURLs EDITOR
James Stevermer, MD, MSPH
Department of Family Medicine, University of Missouri-Columbia

Issue
The Journal of Family Practice - 62(8)
Publications
Topics
Page Number
422-424
Legacy Keywords
Julie Monaco; MD; Anne Mounsey; MD; Jennifer; Bello Kottenstette; MD; PURLs; omega-3 supplements; polyunsaturated fatty acid; PUFA; eicosapentaenoic acid; EPA; docosahexaenoic acid; DHA; PUFA supplements
Sections
Files
Files
Author and Disclosure Information

Julie Monaco, MD;
Anne Mounsey, MD;
Jennifer Bello Kottenstette, MD

Department of Family Medicine, University of North Carolina at Chapel Hill (Drs. Monaco and Mounsey); Department of Family Medicine, The University of Chicago (Dr. Kottenstette)

PURLs EDITOR
James Stevermer, MD, MSPH
Department of Family Medicine, University of Missouri-Columbia

Author and Disclosure Information

Julie Monaco, MD;
Anne Mounsey, MD;
Jennifer Bello Kottenstette, MD

Department of Family Medicine, University of North Carolina at Chapel Hill (Drs. Monaco and Mounsey); Department of Family Medicine, The University of Chicago (Dr. Kottenstette)

PURLs EDITOR
James Stevermer, MD, MSPH
Department of Family Medicine, University of Missouri-Columbia

Article PDF
Article PDF
PRACTICE CHANGER

Stop recommending omega-3 fatty acid supplements for cardiovascular protection. They have no significant impact on all-cause mortality, acute myocardial infarction, sudden death, or stroke.1

Strength of Recommendation

A: Based on a meta-analysis of randomized controlled trials (RCTs).

Rizos E, Ntzani E, Bika E, et al. Association between omega-3 fatty acid supplementation and risk of major cardiovascular disease events: a systematic review and meta-analysis. JAMA. 2012;308:1024-1033.

 

ILLUSTRATIVE CASE

A 59–year-old patient who had a myocardial infarction (mi) 3 years ago is taking an ace inhibitor, a statin, and a b-blocker. He asks you whether he should also take omega-3 fatty acid supplements to further decrease his risk of heart disease. What should you tell him?

Coronary artery disease (CAD) kills more than 500,000 Americans every year2, and medical and dietary therapies for primary and secondary cardiovascular protection are paramount. Omega-3 polyunsaturated fatty acid (PUFA) supplementation is one such therapy. Omega-3 PUFAs are precursors to certain prostaglandins that decrease the proinflammatory state in patients with CAD. They also lower triglyceride levels and produce an antiarrhythmic effect by promoting electrical stability.

But do PUFA supplements provide cardioprotection?
The American Heart Association’s Nutrition Committee recommends either omega-3 PUFA supplementation with 250 to 500 mg eicosapentaenoic acid (EPA) and docosahexaenoic acid (DHA) per day or 2 servings of oily fish per week for both primary and secondary prevention of CAD.3 The European Society of Cardiology also encourages increased consumption of oily fish.4

These recommendations are based on primary and secondary prevention studies, performed between 1989 and 2007, that found a 15% to 29% decrease in all-cause mortality and nonfatal cardiovascular events associated with regular intake of omega-3 fatty acids.5-7 The systematic review and meta-analysis detailed below revisited the effect of omega-3 supplementation on major cardiovascular outcomes.1

STUDY SUMMARY: Omega-3 supplements don’t lower cardiovascular risk

This meta-analysis included 20 RCTs with a total of 68,680 patients. The median age was 68 years, with a range from 49 to 70 years. Thirteen of the studies evaluated omega-3 PUFAs for secondary prevention of cardiovascular outcomes, 4 assessed both primary and secondary prevention, and 3 looked at outcomes in patients with implantable cardioverter defibrillators. All lasted longer than one year, and most were high quality, with a low risk of bias.

The median treatment duration was 2 years, with a maximum of 6.2 years. The mean omega-3 PUFA dose evaluated in the studies was 1.5 g per day, with the exception of 2 studies in which patients received omega-3 PUFAs through dietary sources. Twelve studies used a dose of 1 g or more per day. Half of the included trials were performed during the period when statins were routinely prescribed for cardiovascular risk modification (1998 or later).

Outcomes included all-cause mortality (17 studies), cardiac death (13 studies), sudden death (7 studies), MI (13 studies), and stroke (9 studies).

This meta-analysis found trends toward a decrease in all-cause mortality, cardiac death, sudden death, and MI in patients taking omega-3 PUFAs, but no statistically significant association between any of the outcomes and omega-3 PUFA supplementation. The relative risk for all-cause mortality was 0.96 (95% confidence interval, 0.91-1.02; P=.17). Prespecified subgroup analysis found no association between treatment effect and omega-3 fatty acid dose.

Are dietary sources of omega-3s more effective?
In the 2 trials involving dietary supplementation with omega-3 PUFAs, the results for all-cause mortality and cardiac death were conflicting, with one showing an increase in all-cause mortality and cardiac death and the other showing a decrease in both outcomes compared with the control group. No harmful effects of omega-3 PUFAs were found in either the supplement- or diet-based studies. 

 

 

 

WHAT’S NEW: More evidence of little benefit 

The meta-analysis by Rizos et al is the most up-to-date, comprehensive look at the value of omega-3 fatty acids for primary and secondary prevention of cardiovascular events.It differs from previous reviews in that most included studies were well-done RCTs. In addition, the studies were performed in both primary and secondary cardiovascular disease prevention settings and involved different forms of omega-3 PUFA supplementation, including dietary sources and supplements. The trials were predominantly larger than those included in previous systematic reviews, as well. The baseline risk for cardiovascular disease in the newer studies (7 of the 20 RCTs were completed after 2007) may be different from that of previous studies because of increased use of certain medications,such as statins. In recent years, other studies of omega-3 PUFAs have had similar results. A metaanalysis of 14 RCTs found that omega-3 PUFA supplementation offered no benefit for the secondary prevention of cardiovascular disease.8 The FORWARD trial—published earlier this year—showed that omega-3 PUFAs did not decrease the recurrence of atrial fibrillationin patients with a history of confirmed paroxysmal atrial fibrillation.9 And an earlier(2006) analysis of RCTs and cohort studies found no benefit from omega-3 fatty acids for primary prevention of cardiovascular disease or cancer.10

CAVEATS: No significant help, and no harm

There’s no need to tell patients who wish to take omega-3 supplements not to do so, but we should not promote their use for the sole purpose of cardiovascular disease protection.

While this meta-analysis found no statistically significant benefits from omega-3 PUFAs, there is no evidence of harm from PUFA intake, whether from dietary sources or supplements. There is no need to tell patients who wish to take omega-3 supplements not to do so. But we should not promote their use for the sole purpose of cardiovascular disease prevention.

CHALLENGES TO IMPLEMENTATION: Changing minds won’t be easy

Despite recent findings indicating that omega-3 PUFAs provide little primary or secondary protection against cardiovascular events, advertising from supplement manufacturers may make it hard to change patients’ minds. Because diets and supplements containing these fatty acids do not cause apparent harm, patients and physicians may decide that a small potential benefit is worth the expense.

Acknowledgement

The PURLs Surveillance System was supported in part by Grant Number UL1RR024999 from the National Center for Research Resources, a Clinical Translational Science Award to the University of Chicago. The content is solely the responsibility of the authors and does not necessarily represent the official views of the National Center for Research Resources or the National Institutes of Health.

PRACTICE CHANGER

Stop recommending omega-3 fatty acid supplements for cardiovascular protection. They have no significant impact on all-cause mortality, acute myocardial infarction, sudden death, or stroke.1

Strength of Recommendation

A: Based on a meta-analysis of randomized controlled trials (RCTs).

Rizos E, Ntzani E, Bika E, et al. Association between omega-3 fatty acid supplementation and risk of major cardiovascular disease events: a systematic review and meta-analysis. JAMA. 2012;308:1024-1033.

 

ILLUSTRATIVE CASE

A 59–year-old patient who had a myocardial infarction (mi) 3 years ago is taking an ace inhibitor, a statin, and a b-blocker. He asks you whether he should also take omega-3 fatty acid supplements to further decrease his risk of heart disease. What should you tell him?

Coronary artery disease (CAD) kills more than 500,000 Americans every year2, and medical and dietary therapies for primary and secondary cardiovascular protection are paramount. Omega-3 polyunsaturated fatty acid (PUFA) supplementation is one such therapy. Omega-3 PUFAs are precursors to certain prostaglandins that decrease the proinflammatory state in patients with CAD. They also lower triglyceride levels and produce an antiarrhythmic effect by promoting electrical stability.

But do PUFA supplements provide cardioprotection?
The American Heart Association’s Nutrition Committee recommends either omega-3 PUFA supplementation with 250 to 500 mg eicosapentaenoic acid (EPA) and docosahexaenoic acid (DHA) per day or 2 servings of oily fish per week for both primary and secondary prevention of CAD.3 The European Society of Cardiology also encourages increased consumption of oily fish.4

These recommendations are based on primary and secondary prevention studies, performed between 1989 and 2007, that found a 15% to 29% decrease in all-cause mortality and nonfatal cardiovascular events associated with regular intake of omega-3 fatty acids.5-7 The systematic review and meta-analysis detailed below revisited the effect of omega-3 supplementation on major cardiovascular outcomes.1

STUDY SUMMARY: Omega-3 supplements don’t lower cardiovascular risk

This meta-analysis included 20 RCTs with a total of 68,680 patients. The median age was 68 years, with a range from 49 to 70 years. Thirteen of the studies evaluated omega-3 PUFAs for secondary prevention of cardiovascular outcomes, 4 assessed both primary and secondary prevention, and 3 looked at outcomes in patients with implantable cardioverter defibrillators. All lasted longer than one year, and most were high quality, with a low risk of bias.

The median treatment duration was 2 years, with a maximum of 6.2 years. The mean omega-3 PUFA dose evaluated in the studies was 1.5 g per day, with the exception of 2 studies in which patients received omega-3 PUFAs through dietary sources. Twelve studies used a dose of 1 g or more per day. Half of the included trials were performed during the period when statins were routinely prescribed for cardiovascular risk modification (1998 or later).

Outcomes included all-cause mortality (17 studies), cardiac death (13 studies), sudden death (7 studies), MI (13 studies), and stroke (9 studies).

This meta-analysis found trends toward a decrease in all-cause mortality, cardiac death, sudden death, and MI in patients taking omega-3 PUFAs, but no statistically significant association between any of the outcomes and omega-3 PUFA supplementation. The relative risk for all-cause mortality was 0.96 (95% confidence interval, 0.91-1.02; P=.17). Prespecified subgroup analysis found no association between treatment effect and omega-3 fatty acid dose.

Are dietary sources of omega-3s more effective?
In the 2 trials involving dietary supplementation with omega-3 PUFAs, the results for all-cause mortality and cardiac death were conflicting, with one showing an increase in all-cause mortality and cardiac death and the other showing a decrease in both outcomes compared with the control group. No harmful effects of omega-3 PUFAs were found in either the supplement- or diet-based studies. 

 

 

 

WHAT’S NEW: More evidence of little benefit 

The meta-analysis by Rizos et al is the most up-to-date, comprehensive look at the value of omega-3 fatty acids for primary and secondary prevention of cardiovascular events.It differs from previous reviews in that most included studies were well-done RCTs. In addition, the studies were performed in both primary and secondary cardiovascular disease prevention settings and involved different forms of omega-3 PUFA supplementation, including dietary sources and supplements. The trials were predominantly larger than those included in previous systematic reviews, as well. The baseline risk for cardiovascular disease in the newer studies (7 of the 20 RCTs were completed after 2007) may be different from that of previous studies because of increased use of certain medications,such as statins. In recent years, other studies of omega-3 PUFAs have had similar results. A metaanalysis of 14 RCTs found that omega-3 PUFA supplementation offered no benefit for the secondary prevention of cardiovascular disease.8 The FORWARD trial—published earlier this year—showed that omega-3 PUFAs did not decrease the recurrence of atrial fibrillationin patients with a history of confirmed paroxysmal atrial fibrillation.9 And an earlier(2006) analysis of RCTs and cohort studies found no benefit from omega-3 fatty acids for primary prevention of cardiovascular disease or cancer.10

CAVEATS: No significant help, and no harm

There’s no need to tell patients who wish to take omega-3 supplements not to do so, but we should not promote their use for the sole purpose of cardiovascular disease protection.

While this meta-analysis found no statistically significant benefits from omega-3 PUFAs, there is no evidence of harm from PUFA intake, whether from dietary sources or supplements. There is no need to tell patients who wish to take omega-3 supplements not to do so. But we should not promote their use for the sole purpose of cardiovascular disease prevention.

CHALLENGES TO IMPLEMENTATION: Changing minds won’t be easy

Despite recent findings indicating that omega-3 PUFAs provide little primary or secondary protection against cardiovascular events, advertising from supplement manufacturers may make it hard to change patients’ minds. Because diets and supplements containing these fatty acids do not cause apparent harm, patients and physicians may decide that a small potential benefit is worth the expense.

Acknowledgement

The PURLs Surveillance System was supported in part by Grant Number UL1RR024999 from the National Center for Research Resources, a Clinical Translational Science Award to the University of Chicago. The content is solely the responsibility of the authors and does not necessarily represent the official views of the National Center for Research Resources or the National Institutes of Health.

References

1. Rizos E, Ntzani E, Bika E, et al. Association between omega-3 fatty acid supplementation and risk of major cardiovascular disease events: a systemic review and meta-analysis. JAMA.  2012;308:1024-1033.

2. Centers for Disease Control and Prevention. FastStats. Deaths and mortality. 2010. Available at: http://www.cdc.gov/nchs/fastats/deaths.htm. Updated April 5, 2013. Accessed July 21, 2013.

3. Kris-Etherton PM, Harris WS, Appel LJ, et al. Fish consumption, fish oil, omega-3 fatty acids and cardiovascular disease. Circulation.  2002;106:2747-2757.

4. Van de Werf F, Bax J, Betriu A, et al. Management of acute myocardial infarction in patients presenting with persistent ST-segment elevation: the Task Force on the management of ST-segment elevation acute myocardial infarction of the European Society of Cardiology. Eur Heart J. 2008;29:2909-2945.

5. Artham SM, Lavie CJ, Milani RV, et al. Fish oil in primary and secondary cardiovascular prevention. Ochsner J. 2008;8:49-60.

6. Gruppo Italiano per lo Studio della Sopravvivenza nell’Infarto miocardico. Dietary supplementation with n-3 polyunsaturated fatty acids and vitamin E after myocardial infarction: results of GISSI-Prevenzione trial. Lancet. 1999;354:447-455.

7. Yokoyama M, Origasa H, Matsuzaki M, et al. Effects of eicosapentaenoic acid on major coronary events in hypercholesteroleamic patients (JELIS): a randomized open-label, blinded endpoint analysis. Lancet. 2007;369:1090-1098.

8. Kwak SM, Myung SK, Lee YJ, et al. Efficacy of omega-3 fatty acid supplements (eicosapentaenoic acid and docosahexaenoic acid) in the secondary prevention of cardiovascular disease: a metaanalysis of randomized, double-blind, placebo-controlled trials.  Arch Intern Med. 2012;172:686-694.

9. Macchia A, Grancelli H, Varini S, et al. Omega-3 fatty acids for the prevention of recurrent symptomatic atrial fibrillation: results of the FORWARD (Randomized Trial to Assess Efficacy of PUFA for the Maintenance of Sinus Rhythm in Persistent Atrial Fibrillation) trial. J Am Coll Cardiol. 2013;61:463-468.

10. Hooper L, Thompson RL, Harrison RA, et al. Risks and benefits of omega 3 fats for mortality, cardiovascular disease, and cancer: systemic review. BMJ. 2006;332:752-760.

References

1. Rizos E, Ntzani E, Bika E, et al. Association between omega-3 fatty acid supplementation and risk of major cardiovascular disease events: a systemic review and meta-analysis. JAMA.  2012;308:1024-1033.

2. Centers for Disease Control and Prevention. FastStats. Deaths and mortality. 2010. Available at: http://www.cdc.gov/nchs/fastats/deaths.htm. Updated April 5, 2013. Accessed July 21, 2013.

3. Kris-Etherton PM, Harris WS, Appel LJ, et al. Fish consumption, fish oil, omega-3 fatty acids and cardiovascular disease. Circulation.  2002;106:2747-2757.

4. Van de Werf F, Bax J, Betriu A, et al. Management of acute myocardial infarction in patients presenting with persistent ST-segment elevation: the Task Force on the management of ST-segment elevation acute myocardial infarction of the European Society of Cardiology. Eur Heart J. 2008;29:2909-2945.

5. Artham SM, Lavie CJ, Milani RV, et al. Fish oil in primary and secondary cardiovascular prevention. Ochsner J. 2008;8:49-60.

6. Gruppo Italiano per lo Studio della Sopravvivenza nell’Infarto miocardico. Dietary supplementation with n-3 polyunsaturated fatty acids and vitamin E after myocardial infarction: results of GISSI-Prevenzione trial. Lancet. 1999;354:447-455.

7. Yokoyama M, Origasa H, Matsuzaki M, et al. Effects of eicosapentaenoic acid on major coronary events in hypercholesteroleamic patients (JELIS): a randomized open-label, blinded endpoint analysis. Lancet. 2007;369:1090-1098.

8. Kwak SM, Myung SK, Lee YJ, et al. Efficacy of omega-3 fatty acid supplements (eicosapentaenoic acid and docosahexaenoic acid) in the secondary prevention of cardiovascular disease: a metaanalysis of randomized, double-blind, placebo-controlled trials.  Arch Intern Med. 2012;172:686-694.

9. Macchia A, Grancelli H, Varini S, et al. Omega-3 fatty acids for the prevention of recurrent symptomatic atrial fibrillation: results of the FORWARD (Randomized Trial to Assess Efficacy of PUFA for the Maintenance of Sinus Rhythm in Persistent Atrial Fibrillation) trial. J Am Coll Cardiol. 2013;61:463-468.

10. Hooper L, Thompson RL, Harrison RA, et al. Risks and benefits of omega 3 fats for mortality, cardiovascular disease, and cancer: systemic review. BMJ. 2006;332:752-760.

Issue
The Journal of Family Practice - 62(8)
Issue
The Journal of Family Practice - 62(8)
Page Number
422-424
Page Number
422-424
Publications
Publications
Topics
Article Type
Display Headline
Should you still recommend omega-3 supplements?
Display Headline
Should you still recommend omega-3 supplements?
Legacy Keywords
Julie Monaco; MD; Anne Mounsey; MD; Jennifer; Bello Kottenstette; MD; PURLs; omega-3 supplements; polyunsaturated fatty acid; PUFA; eicosapentaenoic acid; EPA; docosahexaenoic acid; DHA; PUFA supplements
Legacy Keywords
Julie Monaco; MD; Anne Mounsey; MD; Jennifer; Bello Kottenstette; MD; PURLs; omega-3 supplements; polyunsaturated fatty acid; PUFA; eicosapentaenoic acid; EPA; docosahexaenoic acid; DHA; PUFA supplements
Sections
PURLs Copyright

Copyright © 2013. The Family Physicians Inquiries Network. All rights reserved.

Disallow All Ads
Alternative CME
Use ProPublica
Hide sidebar & use full width
render the right sidebar.
Conference Recap Checkbox
Not Conference Recap
Clinical Edge
Display the Slideshow in this Article
Article PDF Media
Media Files

Is this pregnancy viable?

Article Type
Changed
Tue, 07/14/2020 - 14:52
Display Headline
Is this pregnancy viable?
PRACTICE CHANGER

Measure serum progesterone levels of women with bleeding or pain and inconclusive ultrasound in early pregnancy to rule out viability, potentially eliminating the need for serial b-hormone human chorionic gonadotropin (b-hCG) testing.1

STRENGTH OF RECOMMENDATION
A: Based on a systematic review and meta-analysis of 26 diagnostic accuracy studies.

Verhaegen J, Gallos ID, van Mello NM, et al. Accuracy of a single progesterone test to predict early pregnancy outcome in women with pain or bleeding: meta-analysis of cohort studies. BMJ. 2012;345:e6077.

 

Illustrative case

A 20-year-old woman with an estimated gestational age of 7 weeks comes to your clinic because of vaginal bleeding, which started 4 hours ago. A transvaginal ultrasound is inconclusive for an intrauterine pregnancy. Should you obtain a serum progesterone measurement?

Between 21% and 27% of pregnant women experience vaginal bleeding in their first trimester.2,3 This leads to concern, both for patients and physicians, as it can be the first sign of a miscarriage or an ectopic pregnancy. A longitudinal population-based Swedish study of women who had ever been pregnant found that one in 4 had experienced an early pregnancy failure. Overall, about 12% of clinically recognized pregnancies ended in miscarriage.4

Our ability to predict early pregnancy loss is limited

Tools used by clinicians to evaluate vaginal bleeding or pain in the first trimester include transvaginal ultrasound (TVUS) and serial serum β-hCG measurements.5 Even when combined with risk factors for pregnancy loss (serum levels of estradiol, inhibin A, and inhibin B; maternal age; smoking; past history of spontaneous miscarriage; and vaginal bleeding), TVUS is not accurate at predicting early pregnancy loss.6,7

A suboptimal rise in β-hCG (<66%) after 48 hours has historically been used to indicate possible miscarriage or ectopic pregnancy,1but studies have found similarly low rates of increase in some viable pregnancies, as well.8,9 And β-hCG measurements need to be done on more than one occasion, making this an inconvenient means of predicting miscarriage.

Moreover, β-hCG levels vary based on gestational age, leaving family physicians with no solid diagnostic rule regarding the appropriate level of rise in a viable pregnancy.10 Thus, there is a need for a test that complements TVUS and β-hCG to increase diagnostic accuracy in predicting nonviable pregnancies.

Can serum progesterone testing fill the gap?
Serum progesterone measurement is a noninvasive predictive tool, with low values associated with miscarriage and ectopic pregnancy and higher levels with a viable pregnancy.10   Studies have found that serum progesterone combined with β-hCG measurements has the highest reliability in predicting nonviable pregnancy, with a diagnostic accuracy of 85.7% (sensitivity, 88.1%; specificity, 84.3%). This compares with a diagnostic accuracy of 72.5% (sensitivity, 76.1%; specificity, 70.4%) for a single progesterone test alone, and 74.8% (sensitivity, 64.1%; specificity, 81.4%) for β-hCG alone.10-12 The data are from older studies, including a meta-analysis, that did not include the use of TVUS.10-12 But TVUS is now in widespread use and included in the systematic review and meta-analysis this PURL addresses.

Study summary:

Progesterone test is predictive—when combined with ultrasound
Verhaegen et al performed a comprehensive literature search to identify studies in which a single serum progesterone measurement was used to predict the viability of pregnancy vs miscarriage or ectopic pregnancy. They included studies of women with spontaneous pregnancy of <14 weeks. Trials of women who had conceived after ovulation induction or in vitro fertilization or received progesterone supplementation were excluded.

Twenty-six cohort studies met the inclusion criteria. These included 7 mostly high-quality studies, with a total of 2379 women with pain or bleeding and inconclusive TVUS, and 19 intermediate-quality studies (n=7057) of women who had pain or bleeding but no ultrasound.

Five of the 7 studies in women with symptoms and inconclusive TVUS had a similar progesterone test cutoff value (3.2-6 ng/mL). In these 5 studies (n=1998), the progesterone test predicted a nonviable pregnancy with a pooled sensitivity of 74.6% (95% CI, 50.6%-89.4%) and specificity of 98.4% (95% CI, 90.9%-99.7%), a positive likelihood ratio of 45 (7.1- 289) and a negative likelihood ratio of 0.26 (0.12-0.57). When progesterone was below the cutoff value, the probability of a nonviable pregnancy increased to 99.2%. In women with pain or bleeding but no ultrasound, a single progesterone test is less accurate in ruling out a viable pregnancy.

 

 

 

What's new

This test can end days of anxious waiting

This meta-analysis provides strong evidence that a single progesterone measurement is useful in predicting nonviable pregnancies in women with pain or bleeding when TVUS is inconclusive. In such patients, a low serum progesterone is highly predictive of a nonviable pregnancy.1 This finding enables the physician to counsel the woman immediately on the likely pregnancy loss, without waiting days for serial β-hCG results.

Caveats

Progesterone is a poor predictor of ectopic pregnancy

An important caveat to our recommendation is that a single serum progesterone test has a poor predictive value for ectopic pregnancy and should not be used for this purpose. A combination of TVUS and serial β-hCG remains the optimal strategy for diagnosing ectopic pregnancy.13

It is important to note that there is no universally accepted definition of a low serum progesterone level: This meta-analysis included studies with a cutoff value of 3.2 to 6 ng/mL in women who had had a previous ultrasound. What’s more, these studies did not evaluate the predictive value of a serum progesterone test combined with β-hCG measurements.

Challenges to implementation

There are none
We do not see any challenges to the implementation of this recommendation.

ACKNOWLEDGEMENT 

The PURLs Surveillance System was supported in part by Grant Number UL1RR024999 from the National Center for Research Resources, a Clinical Translational Science Award to the University of Chicago. The content is solely the responsibility of the authors and does not necessarily represent the official views of the National Center of Research Resources or the National Institutes of Health.

Files
References

1. Verhaegen J, Gallos ID, van Mello NM, et al. Accuracy of single progesterone test to predict early pregnancy outcome in women with pain or bleeding: meta-analysis of cohort studies. BMJ. 2012;345:e6077.

2. Hasan R, Baird DD, Herring AH, et al. Patterns and predictors of vaginal bleeding in the first trimester of pregnancy. Ann Epidemiol. 2010;20:524-531.

3. Everett C. Incidence and outcome of bleeding before the 20th week of pregnancy: prospective study from general practice. BMJ. 1997;315:32-34.

4. Blohm F, Friden B, Milsom I. A prospective longitudinal population-based study of clinical miscarriage in an urban Swedish population. BJOG. 2008;115:176-183.
Chen BA, Creinin MD. Contemporary management of early pregnancy failure. Clin Obstet Gynecol. 2007;50:67-88.

5. Jauniaux E, Johns J, Burton GJ. The role of ultrasound imaging in diagnosing and investigating early pregnancy failure. Ultrasound Obstet Gynecol. 2005;25:613–624.

6. Gagnon A, Wilson RD, Audibert F, et al. Obstetrical complications associated with abnormal maternal serum markers analytes.J Obstet Gynaecol Can. 2008;30:918-949.

7. Barnhart KT, Sammel MD, Rinaudo PF, et al. Symptomatic patients with an early viable intrauterine pregnancy; hCG curves redefined. Obstet Gynecol. 2004;104:50-54.

8. Morse CB, Sammel MD, Shaunik A, et al. Performance of human chorionic gonadotropin curves in women at risk for ectopic pregnancy: exceptions to the rules. Fertil Steril. 2012;97:101e2-106.e2.

9. Mol BW, Lijmer JG, Ankum WM, et al. The accuracy of single serum progesterone measurement in the diagnosis of ectopic pregnancy: a meta-analysis. Hum Reprod. 1998;13:3220-3227.

10. Duan L, Yan D, Zeng W, et al. Predictive power of progesterone combined with beta human chorionic gonadotropin measurements in the outcome of threatened miscarriage. Arch Gynecol Obstet. 2011;283:431-4355.

11. Phipps MG, Hogan JW, Peipert JF, et al. Progesterone, inhibin, and hCG multiple marker strategy to differentiate viable from nonviable pregnancies. Obstet Gynecol. 2000;95:227-231.

12. American College of Obstetricians and Gynecologists. Practice Bulletin no. 94: Medical management of ectopic pregnancy. Obstet Gynecol. 2008;111:1479-1485.

Author and Disclosure Information

Andrew H. Slattengren, DO; Shailendra Prasad, MBBS, MPH; Sonia Oyola, MD
North Memorial Family Medicine Residency, University of Minnesota, Minneapolis (Drs. Slattengren and Prasad); Department of Family Medicine, The University of Chicago (Dr. Oyola)

PURLs Editor
Anne Mounsey, MD
University of North Carolina at Chapel Hill

Issue
The Journal of Family Practice - 62(06)
Publications
Topics
Page Number
305-305, 316
Legacy Keywords
Andrew H. Slattengren; DO; Shailendra Prasad; MBBS; MPH; Sonia Oyola; MD; PURLs; pregnancy; serum progesterone; β-hCG; TVUS; progesterone test
Sections
Files
Files
Author and Disclosure Information

Andrew H. Slattengren, DO; Shailendra Prasad, MBBS, MPH; Sonia Oyola, MD
North Memorial Family Medicine Residency, University of Minnesota, Minneapolis (Drs. Slattengren and Prasad); Department of Family Medicine, The University of Chicago (Dr. Oyola)

PURLs Editor
Anne Mounsey, MD
University of North Carolina at Chapel Hill

Author and Disclosure Information

Andrew H. Slattengren, DO; Shailendra Prasad, MBBS, MPH; Sonia Oyola, MD
North Memorial Family Medicine Residency, University of Minnesota, Minneapolis (Drs. Slattengren and Prasad); Department of Family Medicine, The University of Chicago (Dr. Oyola)

PURLs Editor
Anne Mounsey, MD
University of North Carolina at Chapel Hill

PRACTICE CHANGER

Measure serum progesterone levels of women with bleeding or pain and inconclusive ultrasound in early pregnancy to rule out viability, potentially eliminating the need for serial b-hormone human chorionic gonadotropin (b-hCG) testing.1

STRENGTH OF RECOMMENDATION
A: Based on a systematic review and meta-analysis of 26 diagnostic accuracy studies.

Verhaegen J, Gallos ID, van Mello NM, et al. Accuracy of a single progesterone test to predict early pregnancy outcome in women with pain or bleeding: meta-analysis of cohort studies. BMJ. 2012;345:e6077.

 

Illustrative case

A 20-year-old woman with an estimated gestational age of 7 weeks comes to your clinic because of vaginal bleeding, which started 4 hours ago. A transvaginal ultrasound is inconclusive for an intrauterine pregnancy. Should you obtain a serum progesterone measurement?

Between 21% and 27% of pregnant women experience vaginal bleeding in their first trimester.2,3 This leads to concern, both for patients and physicians, as it can be the first sign of a miscarriage or an ectopic pregnancy. A longitudinal population-based Swedish study of women who had ever been pregnant found that one in 4 had experienced an early pregnancy failure. Overall, about 12% of clinically recognized pregnancies ended in miscarriage.4

Our ability to predict early pregnancy loss is limited

Tools used by clinicians to evaluate vaginal bleeding or pain in the first trimester include transvaginal ultrasound (TVUS) and serial serum β-hCG measurements.5 Even when combined with risk factors for pregnancy loss (serum levels of estradiol, inhibin A, and inhibin B; maternal age; smoking; past history of spontaneous miscarriage; and vaginal bleeding), TVUS is not accurate at predicting early pregnancy loss.6,7

A suboptimal rise in β-hCG (<66%) after 48 hours has historically been used to indicate possible miscarriage or ectopic pregnancy,1but studies have found similarly low rates of increase in some viable pregnancies, as well.8,9 And β-hCG measurements need to be done on more than one occasion, making this an inconvenient means of predicting miscarriage.

Moreover, β-hCG levels vary based on gestational age, leaving family physicians with no solid diagnostic rule regarding the appropriate level of rise in a viable pregnancy.10 Thus, there is a need for a test that complements TVUS and β-hCG to increase diagnostic accuracy in predicting nonviable pregnancies.

Can serum progesterone testing fill the gap?
Serum progesterone measurement is a noninvasive predictive tool, with low values associated with miscarriage and ectopic pregnancy and higher levels with a viable pregnancy.10   Studies have found that serum progesterone combined with β-hCG measurements has the highest reliability in predicting nonviable pregnancy, with a diagnostic accuracy of 85.7% (sensitivity, 88.1%; specificity, 84.3%). This compares with a diagnostic accuracy of 72.5% (sensitivity, 76.1%; specificity, 70.4%) for a single progesterone test alone, and 74.8% (sensitivity, 64.1%; specificity, 81.4%) for β-hCG alone.10-12 The data are from older studies, including a meta-analysis, that did not include the use of TVUS.10-12 But TVUS is now in widespread use and included in the systematic review and meta-analysis this PURL addresses.

Study summary:

Progesterone test is predictive—when combined with ultrasound
Verhaegen et al performed a comprehensive literature search to identify studies in which a single serum progesterone measurement was used to predict the viability of pregnancy vs miscarriage or ectopic pregnancy. They included studies of women with spontaneous pregnancy of <14 weeks. Trials of women who had conceived after ovulation induction or in vitro fertilization or received progesterone supplementation were excluded.

Twenty-six cohort studies met the inclusion criteria. These included 7 mostly high-quality studies, with a total of 2379 women with pain or bleeding and inconclusive TVUS, and 19 intermediate-quality studies (n=7057) of women who had pain or bleeding but no ultrasound.

Five of the 7 studies in women with symptoms and inconclusive TVUS had a similar progesterone test cutoff value (3.2-6 ng/mL). In these 5 studies (n=1998), the progesterone test predicted a nonviable pregnancy with a pooled sensitivity of 74.6% (95% CI, 50.6%-89.4%) and specificity of 98.4% (95% CI, 90.9%-99.7%), a positive likelihood ratio of 45 (7.1- 289) and a negative likelihood ratio of 0.26 (0.12-0.57). When progesterone was below the cutoff value, the probability of a nonviable pregnancy increased to 99.2%. In women with pain or bleeding but no ultrasound, a single progesterone test is less accurate in ruling out a viable pregnancy.

 

 

 

What's new

This test can end days of anxious waiting

This meta-analysis provides strong evidence that a single progesterone measurement is useful in predicting nonviable pregnancies in women with pain or bleeding when TVUS is inconclusive. In such patients, a low serum progesterone is highly predictive of a nonviable pregnancy.1 This finding enables the physician to counsel the woman immediately on the likely pregnancy loss, without waiting days for serial β-hCG results.

Caveats

Progesterone is a poor predictor of ectopic pregnancy

An important caveat to our recommendation is that a single serum progesterone test has a poor predictive value for ectopic pregnancy and should not be used for this purpose. A combination of TVUS and serial β-hCG remains the optimal strategy for diagnosing ectopic pregnancy.13

It is important to note that there is no universally accepted definition of a low serum progesterone level: This meta-analysis included studies with a cutoff value of 3.2 to 6 ng/mL in women who had had a previous ultrasound. What’s more, these studies did not evaluate the predictive value of a serum progesterone test combined with β-hCG measurements.

Challenges to implementation

There are none
We do not see any challenges to the implementation of this recommendation.

ACKNOWLEDGEMENT 

The PURLs Surveillance System was supported in part by Grant Number UL1RR024999 from the National Center for Research Resources, a Clinical Translational Science Award to the University of Chicago. The content is solely the responsibility of the authors and does not necessarily represent the official views of the National Center of Research Resources or the National Institutes of Health.

PRACTICE CHANGER

Measure serum progesterone levels of women with bleeding or pain and inconclusive ultrasound in early pregnancy to rule out viability, potentially eliminating the need for serial b-hormone human chorionic gonadotropin (b-hCG) testing.1

STRENGTH OF RECOMMENDATION
A: Based on a systematic review and meta-analysis of 26 diagnostic accuracy studies.

Verhaegen J, Gallos ID, van Mello NM, et al. Accuracy of a single progesterone test to predict early pregnancy outcome in women with pain or bleeding: meta-analysis of cohort studies. BMJ. 2012;345:e6077.

 

Illustrative case

A 20-year-old woman with an estimated gestational age of 7 weeks comes to your clinic because of vaginal bleeding, which started 4 hours ago. A transvaginal ultrasound is inconclusive for an intrauterine pregnancy. Should you obtain a serum progesterone measurement?

Between 21% and 27% of pregnant women experience vaginal bleeding in their first trimester.2,3 This leads to concern, both for patients and physicians, as it can be the first sign of a miscarriage or an ectopic pregnancy. A longitudinal population-based Swedish study of women who had ever been pregnant found that one in 4 had experienced an early pregnancy failure. Overall, about 12% of clinically recognized pregnancies ended in miscarriage.4

Our ability to predict early pregnancy loss is limited

Tools used by clinicians to evaluate vaginal bleeding or pain in the first trimester include transvaginal ultrasound (TVUS) and serial serum β-hCG measurements.5 Even when combined with risk factors for pregnancy loss (serum levels of estradiol, inhibin A, and inhibin B; maternal age; smoking; past history of spontaneous miscarriage; and vaginal bleeding), TVUS is not accurate at predicting early pregnancy loss.6,7

A suboptimal rise in β-hCG (<66%) after 48 hours has historically been used to indicate possible miscarriage or ectopic pregnancy,1but studies have found similarly low rates of increase in some viable pregnancies, as well.8,9 And β-hCG measurements need to be done on more than one occasion, making this an inconvenient means of predicting miscarriage.

Moreover, β-hCG levels vary based on gestational age, leaving family physicians with no solid diagnostic rule regarding the appropriate level of rise in a viable pregnancy.10 Thus, there is a need for a test that complements TVUS and β-hCG to increase diagnostic accuracy in predicting nonviable pregnancies.

Can serum progesterone testing fill the gap?
Serum progesterone measurement is a noninvasive predictive tool, with low values associated with miscarriage and ectopic pregnancy and higher levels with a viable pregnancy.10   Studies have found that serum progesterone combined with β-hCG measurements has the highest reliability in predicting nonviable pregnancy, with a diagnostic accuracy of 85.7% (sensitivity, 88.1%; specificity, 84.3%). This compares with a diagnostic accuracy of 72.5% (sensitivity, 76.1%; specificity, 70.4%) for a single progesterone test alone, and 74.8% (sensitivity, 64.1%; specificity, 81.4%) for β-hCG alone.10-12 The data are from older studies, including a meta-analysis, that did not include the use of TVUS.10-12 But TVUS is now in widespread use and included in the systematic review and meta-analysis this PURL addresses.

Study summary:

Progesterone test is predictive—when combined with ultrasound
Verhaegen et al performed a comprehensive literature search to identify studies in which a single serum progesterone measurement was used to predict the viability of pregnancy vs miscarriage or ectopic pregnancy. They included studies of women with spontaneous pregnancy of <14 weeks. Trials of women who had conceived after ovulation induction or in vitro fertilization or received progesterone supplementation were excluded.

Twenty-six cohort studies met the inclusion criteria. These included 7 mostly high-quality studies, with a total of 2379 women with pain or bleeding and inconclusive TVUS, and 19 intermediate-quality studies (n=7057) of women who had pain or bleeding but no ultrasound.

Five of the 7 studies in women with symptoms and inconclusive TVUS had a similar progesterone test cutoff value (3.2-6 ng/mL). In these 5 studies (n=1998), the progesterone test predicted a nonviable pregnancy with a pooled sensitivity of 74.6% (95% CI, 50.6%-89.4%) and specificity of 98.4% (95% CI, 90.9%-99.7%), a positive likelihood ratio of 45 (7.1- 289) and a negative likelihood ratio of 0.26 (0.12-0.57). When progesterone was below the cutoff value, the probability of a nonviable pregnancy increased to 99.2%. In women with pain or bleeding but no ultrasound, a single progesterone test is less accurate in ruling out a viable pregnancy.

 

 

 

What's new

This test can end days of anxious waiting

This meta-analysis provides strong evidence that a single progesterone measurement is useful in predicting nonviable pregnancies in women with pain or bleeding when TVUS is inconclusive. In such patients, a low serum progesterone is highly predictive of a nonviable pregnancy.1 This finding enables the physician to counsel the woman immediately on the likely pregnancy loss, without waiting days for serial β-hCG results.

Caveats

Progesterone is a poor predictor of ectopic pregnancy

An important caveat to our recommendation is that a single serum progesterone test has a poor predictive value for ectopic pregnancy and should not be used for this purpose. A combination of TVUS and serial β-hCG remains the optimal strategy for diagnosing ectopic pregnancy.13

It is important to note that there is no universally accepted definition of a low serum progesterone level: This meta-analysis included studies with a cutoff value of 3.2 to 6 ng/mL in women who had had a previous ultrasound. What’s more, these studies did not evaluate the predictive value of a serum progesterone test combined with β-hCG measurements.

Challenges to implementation

There are none
We do not see any challenges to the implementation of this recommendation.

ACKNOWLEDGEMENT 

The PURLs Surveillance System was supported in part by Grant Number UL1RR024999 from the National Center for Research Resources, a Clinical Translational Science Award to the University of Chicago. The content is solely the responsibility of the authors and does not necessarily represent the official views of the National Center of Research Resources or the National Institutes of Health.

References

1. Verhaegen J, Gallos ID, van Mello NM, et al. Accuracy of single progesterone test to predict early pregnancy outcome in women with pain or bleeding: meta-analysis of cohort studies. BMJ. 2012;345:e6077.

2. Hasan R, Baird DD, Herring AH, et al. Patterns and predictors of vaginal bleeding in the first trimester of pregnancy. Ann Epidemiol. 2010;20:524-531.

3. Everett C. Incidence and outcome of bleeding before the 20th week of pregnancy: prospective study from general practice. BMJ. 1997;315:32-34.

4. Blohm F, Friden B, Milsom I. A prospective longitudinal population-based study of clinical miscarriage in an urban Swedish population. BJOG. 2008;115:176-183.
Chen BA, Creinin MD. Contemporary management of early pregnancy failure. Clin Obstet Gynecol. 2007;50:67-88.

5. Jauniaux E, Johns J, Burton GJ. The role of ultrasound imaging in diagnosing and investigating early pregnancy failure. Ultrasound Obstet Gynecol. 2005;25:613–624.

6. Gagnon A, Wilson RD, Audibert F, et al. Obstetrical complications associated with abnormal maternal serum markers analytes.J Obstet Gynaecol Can. 2008;30:918-949.

7. Barnhart KT, Sammel MD, Rinaudo PF, et al. Symptomatic patients with an early viable intrauterine pregnancy; hCG curves redefined. Obstet Gynecol. 2004;104:50-54.

8. Morse CB, Sammel MD, Shaunik A, et al. Performance of human chorionic gonadotropin curves in women at risk for ectopic pregnancy: exceptions to the rules. Fertil Steril. 2012;97:101e2-106.e2.

9. Mol BW, Lijmer JG, Ankum WM, et al. The accuracy of single serum progesterone measurement in the diagnosis of ectopic pregnancy: a meta-analysis. Hum Reprod. 1998;13:3220-3227.

10. Duan L, Yan D, Zeng W, et al. Predictive power of progesterone combined with beta human chorionic gonadotropin measurements in the outcome of threatened miscarriage. Arch Gynecol Obstet. 2011;283:431-4355.

11. Phipps MG, Hogan JW, Peipert JF, et al. Progesterone, inhibin, and hCG multiple marker strategy to differentiate viable from nonviable pregnancies. Obstet Gynecol. 2000;95:227-231.

12. American College of Obstetricians and Gynecologists. Practice Bulletin no. 94: Medical management of ectopic pregnancy. Obstet Gynecol. 2008;111:1479-1485.

References

1. Verhaegen J, Gallos ID, van Mello NM, et al. Accuracy of single progesterone test to predict early pregnancy outcome in women with pain or bleeding: meta-analysis of cohort studies. BMJ. 2012;345:e6077.

2. Hasan R, Baird DD, Herring AH, et al. Patterns and predictors of vaginal bleeding in the first trimester of pregnancy. Ann Epidemiol. 2010;20:524-531.

3. Everett C. Incidence and outcome of bleeding before the 20th week of pregnancy: prospective study from general practice. BMJ. 1997;315:32-34.

4. Blohm F, Friden B, Milsom I. A prospective longitudinal population-based study of clinical miscarriage in an urban Swedish population. BJOG. 2008;115:176-183.
Chen BA, Creinin MD. Contemporary management of early pregnancy failure. Clin Obstet Gynecol. 2007;50:67-88.

5. Jauniaux E, Johns J, Burton GJ. The role of ultrasound imaging in diagnosing and investigating early pregnancy failure. Ultrasound Obstet Gynecol. 2005;25:613–624.

6. Gagnon A, Wilson RD, Audibert F, et al. Obstetrical complications associated with abnormal maternal serum markers analytes.J Obstet Gynaecol Can. 2008;30:918-949.

7. Barnhart KT, Sammel MD, Rinaudo PF, et al. Symptomatic patients with an early viable intrauterine pregnancy; hCG curves redefined. Obstet Gynecol. 2004;104:50-54.

8. Morse CB, Sammel MD, Shaunik A, et al. Performance of human chorionic gonadotropin curves in women at risk for ectopic pregnancy: exceptions to the rules. Fertil Steril. 2012;97:101e2-106.e2.

9. Mol BW, Lijmer JG, Ankum WM, et al. The accuracy of single serum progesterone measurement in the diagnosis of ectopic pregnancy: a meta-analysis. Hum Reprod. 1998;13:3220-3227.

10. Duan L, Yan D, Zeng W, et al. Predictive power of progesterone combined with beta human chorionic gonadotropin measurements in the outcome of threatened miscarriage. Arch Gynecol Obstet. 2011;283:431-4355.

11. Phipps MG, Hogan JW, Peipert JF, et al. Progesterone, inhibin, and hCG multiple marker strategy to differentiate viable from nonviable pregnancies. Obstet Gynecol. 2000;95:227-231.

12. American College of Obstetricians and Gynecologists. Practice Bulletin no. 94: Medical management of ectopic pregnancy. Obstet Gynecol. 2008;111:1479-1485.

Issue
The Journal of Family Practice - 62(06)
Issue
The Journal of Family Practice - 62(06)
Page Number
305-305, 316
Page Number
305-305, 316
Publications
Publications
Topics
Article Type
Display Headline
Is this pregnancy viable?
Display Headline
Is this pregnancy viable?
Legacy Keywords
Andrew H. Slattengren; DO; Shailendra Prasad; MBBS; MPH; Sonia Oyola; MD; PURLs; pregnancy; serum progesterone; β-hCG; TVUS; progesterone test
Legacy Keywords
Andrew H. Slattengren; DO; Shailendra Prasad; MBBS; MPH; Sonia Oyola; MD; PURLs; pregnancy; serum progesterone; β-hCG; TVUS; progesterone test
Sections
PURLs Copyright

Copyright © 2013. The Family Physicians Inquiries Network. All rights reserved.

Disallow All Ads
Alternative CME
Use ProPublica
Hide sidebar & use full width
render the right sidebar.
Conference Recap Checkbox
Not Conference Recap
Clinical Edge
Display the Slideshow in this Article
Media Files

Corticosteroids for a sore throat?

Article Type
Changed
Tue, 07/14/2020 - 14:42
Display Headline
Corticosteroids for a sore throat?

PRACTICE CHANGER

Consider prescribing a single dose of corticosteroids for patients with sore throat, which has been found to bring quicker pain relief and resolution of symptoms.1

STRENGTH OF RECOMMENDATION

A: Based on a meta-analysis of randomized controlled trials (RCTs) in ambulatory care settings.

Hayward G, Thompson M, Perera R, et al. Corticosteroids as standalone or add-on treatment for sore throat. Cochrane Database Syst Rev. 2012;(10):CDC008268.

 

Illustrative case

A 28-year-old woman comes to your clinic because she’s had a severe sore throat and low-grade fever for the past 2 days. She has no associated cough. Examination reveals erythematous posterior oropharynx with exudate. A rapid strep test is negative. The patient says the sore throat is very painful and asks for medication to make it better. What should you prescribe?

Most sore throats—particularly in adults—are viral and self-limiting.2,3 Group A B-hemolytic Streptococcusinfections account for just 10% of sore throats in adults and 15% to 30% in children.4 Yet US physicians have been found to prescribe antibiotics for more than half of patients who present with sore throat.5-7

Do patients want antibiotics, or simply pain relief?Antibiotics produce only a modest reduction in symptoms of pharyngitis (fever and throat soreness), presumably in patients with bacterial infections, and increase the risk of adverse events.5,6 Research suggests that patients who request antibiotics for sore throat may primarily be seeking pain relief.8 Thus, a treatment that’s more effective in alleviating symptoms of a sore throat would likely contribute to a decrease in unnecessary use of antibiotics.

A short course of corticosteroids has been used successfully and shown to be safe for conditions such as acute sinusitis, croup, and asthma.9-11 Could the anti-inflammatory effects of corticosteroids reduce pain in patients with sore throat, as well? A 2010 systematic review suggested that was the case.12 Cochrane reviewers recently took another look.1

Study summary: Steroids bring speedier pain relief

This meta-analysis included 8 RCTs (the same 8 trials used in the systematic review9) that compared corticosteroids with placebo for the symptomatic treatment of exudative or severe sore throat.1 Sore throat was defined as clinical evidence of pharyngitis and/or tonsillitis or the clinical syndrome of painful throat and odynophagia.

Five studies were conducted in the United States, and one each in Canada, Turkey, and Israel. Five studies focused on adults (N=413); the other 3 studied children (N=393). Overall, 47% of participants had exudative sore throat, and 44% were positive for group A B-hemolytic Streptococcus.

In all 8 RCTs, antibiotics were given to those in both the treatment and placebo groups. In addition, all participants were allowed to use traditional analgesia— either acetaminophen or nonsteroidal anti-inflammatory drugs. Corticosteroids (oral dexamethasone, oral prednisone, or intramuscular [IM] dexamethasone) were used as an adjunctive treatment in all the RCTs.

Primary outcomes varied among the studies. Four of the 8 RCTs included the proportion of patients with improvement or complete resolution of symptoms within 24 to 48 hours. Mean time to onset of pain relief was the primary outcome in 5 of the 8 studies. Some of the secondary outcomes in the individual trials included relapse rates, adverse events, and days missed from school or work.

Overall, patients who received corticosteroids were 3 times more likely to report complete resolution of symptoms at 24 hours (relative risk=3.2; 95% confidence interval, 2.0-5.1; P<.001) and had a reduced mean time to onset of pain relief of about 6 hours. The number needed to treat to prevent one patient from experiencing pain at 24 hours was <4.

Adverse events were reported in only one of the trials (N=125): Five patients (3 in the steroid group and 2 on placebo) were hospitalized for fluid rehydration, and 3 patients (one in the steroid group and 2 on placebo) developed peritonsillar abscess.12 Three RCTs did not find any significant difference in days missed from school or work, and 4 trials reported no difference in recurrence of symptoms. One of the trials found that 16% of the patients in the placebo group returned to seek additional care, while none in the steroid group did.13

What's new: Steroids haven't been tested as standalone treatment

Steroids are not currently recommended for routine use to treat symptoms of sore throat. This Cochrane review found that patients with severe or exudative sore throat benefit from pain reduction with corticosteroids, used as an adjunct to antibiotics and other analgesics without increased risk of harm. Nonetheless, the use of steroids in this patient population would address a practical concern of those seeking symptom relief and has the potential to decrease unnecessary antibiotic use.

 

 

 

Caveats: Questions about effects on antibiotic use, heterogeneity remain

The studies in this meta-analysis did not assess whether the use corticosteroids would reduce unnecessary use of antibiotics, so we cannot conclude that this would be the case. Because the effect was similar in all sub-groups analyzed, however, it is reasonable to expect that reduced antibiotic use could be a positive effect. The main documented benefit was resolution of pain, an important patient-centered outcome that justifies consideration of treating painful pharyngitis with corticosteroids.                                                                                                     

Corticosteroids have an immunosuppressant effect and carry the FAST TRACK
Research suggests that patients who request antibiotics for a sore throat may be seeking pain relief.
 theoretical risk of exacerbating an existing infection. That did not occur in these studies. Nor has it occurred when used for short courses in other illnesses such as croup, infectious mononucleosis, asthma, contact dermatitis, and chronic obstructive pulmonary disease.14 Thus, this theoretical risk is not a barrier to implementation.

It is important to note that single and multiple doses of corticosteroids and oral and IM routes were effective, with only minimal differences in results.  

Challenges to implementation: Determining the severity

Acetaminophen and NSAIDs are used for pain relief in sore throat, and have been shown to be effective—but may be inadequate for severe pain.15 There are no head-to-head trials that have compared steroids to NSAIDs or acetaminophen in this clinical scenario.  So the challenge for clinicians will be to decide when pharyngitis is severe enough to justify the use of corticosteroids, rather than simple analgesics alone.

Acknowledgement

The PURLs Surveillance System was supported in part by Grant number ul1rr024999 from the national center for research resources, a clinical Translational Science award to the university of chicago. The content is solely the responsibility of the authors and does not necessarily represent the official views of the national center for research resources or the national institutes of health.

Files
References

1.    Hayward G, Thompson M, Perera R, et al. Corticosteroids as stand-alone or add-on treatment for sore throat. Cochrane
Database Syst Rev
. 2012;(10):CDC008268.

2.    Cherry DK, Woodwell DA. National Ambulatory Medical Care Survey: 2000 summary. Adv Data. 2002;328:1-32.

3.    Bisno AL. Acute pharyngitis. N Engl J Med . 2001;344:205-211.

4.    Del Mar CB, Glasziou PP, Sprinks AB. Antibiotics for sore throat. Cochrane Database Syst Rev. 2006;(4):CD000023.

5.    Linder JA, Stafford RS. Antibiotic treatment of adults with sore throat by community primary care physicians: a national survey, 1989-1999. JAMA. 2001;286:1181-1186.

6.    Linder JA, Bates DW, Lee GM, et al. Antibiotic treatment of children with sore throat. JAMA. 2005;294:2315-2322.

7.    Hong SY, Taur Y, Jordan MR. Antimicrobial prescribing in the USA for adult acute pharyngitis in relation to treatment guidelines.
J Eval Clin Pract. 2011;17: 1176-1183.

8.    van Driel ML, De Sutter A, Deveugele M, et al Are sore throat patients who hope for antibiotics actually asking for pain relief? Ann Fam Med. 2006;4:494-499.

9.    Venekamp RP, Thompson MJ, Hayward G, et al. Systemic corticosteroids for acute sinusitis. Cochrane Database Syst Rev. 2011;(12):CD008115.

10.  Russell KF, Liang Y, O’Gorman K, et al. Glucocorticoids for croup. Cochrane Database Syst Rev. 2011;(1):CD001955.

11.  Rowe BH, Spooner C, Ducharme F, et al. Early emergency department treatment of acute asthma with systemic corticosteroids. Cochrane Database Syst Rev. 2001;(1):CD002178.

12.  Korb K, Scherer M, Cenot J. Steroids as adjuvant therapy for acute pharyngitis in ambulatory patients: a systematic review. Ann Fam Med. 2010;8:58-63.

13.  Olympia RP, Khine H, Avner JR. Effectiveness of oral dexamethasone in the treatment of moderate to severe pharyngitis in children. Arch Pediatr Adolesc Med. 2005;159:278-282.

14.  Manson SC, Brown RE, Cerulli A, et al. The cumulative burden of oral corticosteroid side effects and the economic implications of steroid use. Respir Med. 2009;103:975-994.

15.  Wei JL, Kasperbauer JL, Weaver AL, et al. Efficacy of single-dose dexamethasone as adjuvant therapy for acute pharyngitis. Laryngoscope. 2002;112:87-93.

Article PDF
Author and Disclosure Information

Keri Bergeson, MD
North Memorial Family Medicine Residency Program, University of Minnesota

Nina Rogers, MD
Department of Family Medicine, The University of Chicago

Shailendra Prasad, MBBS, MPH
North Memorial Family Medicine Residency Program, University of Minnesota

PURLs EDITOR
Bernard Ewigman, MD, MSPH
Department of Family Medicine, The University of Chicago

Issue
The Journal of Family Practice - 62(7)
Publications
Topics
Page Number
372-374
Legacy Keywords
Keri Bergeson; MD; Nina Rogers; MD; Shailendra Prasad; MBBS; MPH; sore throat; corticosteroids; PURLs; steroids; NSAIDs
Sections
Files
Files
Author and Disclosure Information

Keri Bergeson, MD
North Memorial Family Medicine Residency Program, University of Minnesota

Nina Rogers, MD
Department of Family Medicine, The University of Chicago

Shailendra Prasad, MBBS, MPH
North Memorial Family Medicine Residency Program, University of Minnesota

PURLs EDITOR
Bernard Ewigman, MD, MSPH
Department of Family Medicine, The University of Chicago

Author and Disclosure Information

Keri Bergeson, MD
North Memorial Family Medicine Residency Program, University of Minnesota

Nina Rogers, MD
Department of Family Medicine, The University of Chicago

Shailendra Prasad, MBBS, MPH
North Memorial Family Medicine Residency Program, University of Minnesota

PURLs EDITOR
Bernard Ewigman, MD, MSPH
Department of Family Medicine, The University of Chicago

Article PDF
Article PDF

PRACTICE CHANGER

Consider prescribing a single dose of corticosteroids for patients with sore throat, which has been found to bring quicker pain relief and resolution of symptoms.1

STRENGTH OF RECOMMENDATION

A: Based on a meta-analysis of randomized controlled trials (RCTs) in ambulatory care settings.

Hayward G, Thompson M, Perera R, et al. Corticosteroids as standalone or add-on treatment for sore throat. Cochrane Database Syst Rev. 2012;(10):CDC008268.

 

Illustrative case

A 28-year-old woman comes to your clinic because she’s had a severe sore throat and low-grade fever for the past 2 days. She has no associated cough. Examination reveals erythematous posterior oropharynx with exudate. A rapid strep test is negative. The patient says the sore throat is very painful and asks for medication to make it better. What should you prescribe?

Most sore throats—particularly in adults—are viral and self-limiting.2,3 Group A B-hemolytic Streptococcusinfections account for just 10% of sore throats in adults and 15% to 30% in children.4 Yet US physicians have been found to prescribe antibiotics for more than half of patients who present with sore throat.5-7

Do patients want antibiotics, or simply pain relief?Antibiotics produce only a modest reduction in symptoms of pharyngitis (fever and throat soreness), presumably in patients with bacterial infections, and increase the risk of adverse events.5,6 Research suggests that patients who request antibiotics for sore throat may primarily be seeking pain relief.8 Thus, a treatment that’s more effective in alleviating symptoms of a sore throat would likely contribute to a decrease in unnecessary use of antibiotics.

A short course of corticosteroids has been used successfully and shown to be safe for conditions such as acute sinusitis, croup, and asthma.9-11 Could the anti-inflammatory effects of corticosteroids reduce pain in patients with sore throat, as well? A 2010 systematic review suggested that was the case.12 Cochrane reviewers recently took another look.1

Study summary: Steroids bring speedier pain relief

This meta-analysis included 8 RCTs (the same 8 trials used in the systematic review9) that compared corticosteroids with placebo for the symptomatic treatment of exudative or severe sore throat.1 Sore throat was defined as clinical evidence of pharyngitis and/or tonsillitis or the clinical syndrome of painful throat and odynophagia.

Five studies were conducted in the United States, and one each in Canada, Turkey, and Israel. Five studies focused on adults (N=413); the other 3 studied children (N=393). Overall, 47% of participants had exudative sore throat, and 44% were positive for group A B-hemolytic Streptococcus.

In all 8 RCTs, antibiotics were given to those in both the treatment and placebo groups. In addition, all participants were allowed to use traditional analgesia— either acetaminophen or nonsteroidal anti-inflammatory drugs. Corticosteroids (oral dexamethasone, oral prednisone, or intramuscular [IM] dexamethasone) were used as an adjunctive treatment in all the RCTs.

Primary outcomes varied among the studies. Four of the 8 RCTs included the proportion of patients with improvement or complete resolution of symptoms within 24 to 48 hours. Mean time to onset of pain relief was the primary outcome in 5 of the 8 studies. Some of the secondary outcomes in the individual trials included relapse rates, adverse events, and days missed from school or work.

Overall, patients who received corticosteroids were 3 times more likely to report complete resolution of symptoms at 24 hours (relative risk=3.2; 95% confidence interval, 2.0-5.1; P<.001) and had a reduced mean time to onset of pain relief of about 6 hours. The number needed to treat to prevent one patient from experiencing pain at 24 hours was <4.

Adverse events were reported in only one of the trials (N=125): Five patients (3 in the steroid group and 2 on placebo) were hospitalized for fluid rehydration, and 3 patients (one in the steroid group and 2 on placebo) developed peritonsillar abscess.12 Three RCTs did not find any significant difference in days missed from school or work, and 4 trials reported no difference in recurrence of symptoms. One of the trials found that 16% of the patients in the placebo group returned to seek additional care, while none in the steroid group did.13

What's new: Steroids haven't been tested as standalone treatment

Steroids are not currently recommended for routine use to treat symptoms of sore throat. This Cochrane review found that patients with severe or exudative sore throat benefit from pain reduction with corticosteroids, used as an adjunct to antibiotics and other analgesics without increased risk of harm. Nonetheless, the use of steroids in this patient population would address a practical concern of those seeking symptom relief and has the potential to decrease unnecessary antibiotic use.

 

 

 

Caveats: Questions about effects on antibiotic use, heterogeneity remain

The studies in this meta-analysis did not assess whether the use corticosteroids would reduce unnecessary use of antibiotics, so we cannot conclude that this would be the case. Because the effect was similar in all sub-groups analyzed, however, it is reasonable to expect that reduced antibiotic use could be a positive effect. The main documented benefit was resolution of pain, an important patient-centered outcome that justifies consideration of treating painful pharyngitis with corticosteroids.                                                                                                     

Corticosteroids have an immunosuppressant effect and carry the FAST TRACK
Research suggests that patients who request antibiotics for a sore throat may be seeking pain relief.
 theoretical risk of exacerbating an existing infection. That did not occur in these studies. Nor has it occurred when used for short courses in other illnesses such as croup, infectious mononucleosis, asthma, contact dermatitis, and chronic obstructive pulmonary disease.14 Thus, this theoretical risk is not a barrier to implementation.

It is important to note that single and multiple doses of corticosteroids and oral and IM routes were effective, with only minimal differences in results.  

Challenges to implementation: Determining the severity

Acetaminophen and NSAIDs are used for pain relief in sore throat, and have been shown to be effective—but may be inadequate for severe pain.15 There are no head-to-head trials that have compared steroids to NSAIDs or acetaminophen in this clinical scenario.  So the challenge for clinicians will be to decide when pharyngitis is severe enough to justify the use of corticosteroids, rather than simple analgesics alone.

Acknowledgement

The PURLs Surveillance System was supported in part by Grant number ul1rr024999 from the national center for research resources, a clinical Translational Science award to the university of chicago. The content is solely the responsibility of the authors and does not necessarily represent the official views of the national center for research resources or the national institutes of health.

PRACTICE CHANGER

Consider prescribing a single dose of corticosteroids for patients with sore throat, which has been found to bring quicker pain relief and resolution of symptoms.1

STRENGTH OF RECOMMENDATION

A: Based on a meta-analysis of randomized controlled trials (RCTs) in ambulatory care settings.

Hayward G, Thompson M, Perera R, et al. Corticosteroids as standalone or add-on treatment for sore throat. Cochrane Database Syst Rev. 2012;(10):CDC008268.

 

Illustrative case

A 28-year-old woman comes to your clinic because she’s had a severe sore throat and low-grade fever for the past 2 days. She has no associated cough. Examination reveals erythematous posterior oropharynx with exudate. A rapid strep test is negative. The patient says the sore throat is very painful and asks for medication to make it better. What should you prescribe?

Most sore throats—particularly in adults—are viral and self-limiting.2,3 Group A B-hemolytic Streptococcusinfections account for just 10% of sore throats in adults and 15% to 30% in children.4 Yet US physicians have been found to prescribe antibiotics for more than half of patients who present with sore throat.5-7

Do patients want antibiotics, or simply pain relief?Antibiotics produce only a modest reduction in symptoms of pharyngitis (fever and throat soreness), presumably in patients with bacterial infections, and increase the risk of adverse events.5,6 Research suggests that patients who request antibiotics for sore throat may primarily be seeking pain relief.8 Thus, a treatment that’s more effective in alleviating symptoms of a sore throat would likely contribute to a decrease in unnecessary use of antibiotics.

A short course of corticosteroids has been used successfully and shown to be safe for conditions such as acute sinusitis, croup, and asthma.9-11 Could the anti-inflammatory effects of corticosteroids reduce pain in patients with sore throat, as well? A 2010 systematic review suggested that was the case.12 Cochrane reviewers recently took another look.1

Study summary: Steroids bring speedier pain relief

This meta-analysis included 8 RCTs (the same 8 trials used in the systematic review9) that compared corticosteroids with placebo for the symptomatic treatment of exudative or severe sore throat.1 Sore throat was defined as clinical evidence of pharyngitis and/or tonsillitis or the clinical syndrome of painful throat and odynophagia.

Five studies were conducted in the United States, and one each in Canada, Turkey, and Israel. Five studies focused on adults (N=413); the other 3 studied children (N=393). Overall, 47% of participants had exudative sore throat, and 44% were positive for group A B-hemolytic Streptococcus.

In all 8 RCTs, antibiotics were given to those in both the treatment and placebo groups. In addition, all participants were allowed to use traditional analgesia— either acetaminophen or nonsteroidal anti-inflammatory drugs. Corticosteroids (oral dexamethasone, oral prednisone, or intramuscular [IM] dexamethasone) were used as an adjunctive treatment in all the RCTs.

Primary outcomes varied among the studies. Four of the 8 RCTs included the proportion of patients with improvement or complete resolution of symptoms within 24 to 48 hours. Mean time to onset of pain relief was the primary outcome in 5 of the 8 studies. Some of the secondary outcomes in the individual trials included relapse rates, adverse events, and days missed from school or work.

Overall, patients who received corticosteroids were 3 times more likely to report complete resolution of symptoms at 24 hours (relative risk=3.2; 95% confidence interval, 2.0-5.1; P<.001) and had a reduced mean time to onset of pain relief of about 6 hours. The number needed to treat to prevent one patient from experiencing pain at 24 hours was <4.

Adverse events were reported in only one of the trials (N=125): Five patients (3 in the steroid group and 2 on placebo) were hospitalized for fluid rehydration, and 3 patients (one in the steroid group and 2 on placebo) developed peritonsillar abscess.12 Three RCTs did not find any significant difference in days missed from school or work, and 4 trials reported no difference in recurrence of symptoms. One of the trials found that 16% of the patients in the placebo group returned to seek additional care, while none in the steroid group did.13

What's new: Steroids haven't been tested as standalone treatment

Steroids are not currently recommended for routine use to treat symptoms of sore throat. This Cochrane review found that patients with severe or exudative sore throat benefit from pain reduction with corticosteroids, used as an adjunct to antibiotics and other analgesics without increased risk of harm. Nonetheless, the use of steroids in this patient population would address a practical concern of those seeking symptom relief and has the potential to decrease unnecessary antibiotic use.

 

 

 

Caveats: Questions about effects on antibiotic use, heterogeneity remain

The studies in this meta-analysis did not assess whether the use corticosteroids would reduce unnecessary use of antibiotics, so we cannot conclude that this would be the case. Because the effect was similar in all sub-groups analyzed, however, it is reasonable to expect that reduced antibiotic use could be a positive effect. The main documented benefit was resolution of pain, an important patient-centered outcome that justifies consideration of treating painful pharyngitis with corticosteroids.                                                                                                     

Corticosteroids have an immunosuppressant effect and carry the FAST TRACK
Research suggests that patients who request antibiotics for a sore throat may be seeking pain relief.
 theoretical risk of exacerbating an existing infection. That did not occur in these studies. Nor has it occurred when used for short courses in other illnesses such as croup, infectious mononucleosis, asthma, contact dermatitis, and chronic obstructive pulmonary disease.14 Thus, this theoretical risk is not a barrier to implementation.

It is important to note that single and multiple doses of corticosteroids and oral and IM routes were effective, with only minimal differences in results.  

Challenges to implementation: Determining the severity

Acetaminophen and NSAIDs are used for pain relief in sore throat, and have been shown to be effective—but may be inadequate for severe pain.15 There are no head-to-head trials that have compared steroids to NSAIDs or acetaminophen in this clinical scenario.  So the challenge for clinicians will be to decide when pharyngitis is severe enough to justify the use of corticosteroids, rather than simple analgesics alone.

Acknowledgement

The PURLs Surveillance System was supported in part by Grant number ul1rr024999 from the national center for research resources, a clinical Translational Science award to the university of chicago. The content is solely the responsibility of the authors and does not necessarily represent the official views of the national center for research resources or the national institutes of health.

References

1.    Hayward G, Thompson M, Perera R, et al. Corticosteroids as stand-alone or add-on treatment for sore throat. Cochrane
Database Syst Rev
. 2012;(10):CDC008268.

2.    Cherry DK, Woodwell DA. National Ambulatory Medical Care Survey: 2000 summary. Adv Data. 2002;328:1-32.

3.    Bisno AL. Acute pharyngitis. N Engl J Med . 2001;344:205-211.

4.    Del Mar CB, Glasziou PP, Sprinks AB. Antibiotics for sore throat. Cochrane Database Syst Rev. 2006;(4):CD000023.

5.    Linder JA, Stafford RS. Antibiotic treatment of adults with sore throat by community primary care physicians: a national survey, 1989-1999. JAMA. 2001;286:1181-1186.

6.    Linder JA, Bates DW, Lee GM, et al. Antibiotic treatment of children with sore throat. JAMA. 2005;294:2315-2322.

7.    Hong SY, Taur Y, Jordan MR. Antimicrobial prescribing in the USA for adult acute pharyngitis in relation to treatment guidelines.
J Eval Clin Pract. 2011;17: 1176-1183.

8.    van Driel ML, De Sutter A, Deveugele M, et al Are sore throat patients who hope for antibiotics actually asking for pain relief? Ann Fam Med. 2006;4:494-499.

9.    Venekamp RP, Thompson MJ, Hayward G, et al. Systemic corticosteroids for acute sinusitis. Cochrane Database Syst Rev. 2011;(12):CD008115.

10.  Russell KF, Liang Y, O’Gorman K, et al. Glucocorticoids for croup. Cochrane Database Syst Rev. 2011;(1):CD001955.

11.  Rowe BH, Spooner C, Ducharme F, et al. Early emergency department treatment of acute asthma with systemic corticosteroids. Cochrane Database Syst Rev. 2001;(1):CD002178.

12.  Korb K, Scherer M, Cenot J. Steroids as adjuvant therapy for acute pharyngitis in ambulatory patients: a systematic review. Ann Fam Med. 2010;8:58-63.

13.  Olympia RP, Khine H, Avner JR. Effectiveness of oral dexamethasone in the treatment of moderate to severe pharyngitis in children. Arch Pediatr Adolesc Med. 2005;159:278-282.

14.  Manson SC, Brown RE, Cerulli A, et al. The cumulative burden of oral corticosteroid side effects and the economic implications of steroid use. Respir Med. 2009;103:975-994.

15.  Wei JL, Kasperbauer JL, Weaver AL, et al. Efficacy of single-dose dexamethasone as adjuvant therapy for acute pharyngitis. Laryngoscope. 2002;112:87-93.

References

1.    Hayward G, Thompson M, Perera R, et al. Corticosteroids as stand-alone or add-on treatment for sore throat. Cochrane
Database Syst Rev
. 2012;(10):CDC008268.

2.    Cherry DK, Woodwell DA. National Ambulatory Medical Care Survey: 2000 summary. Adv Data. 2002;328:1-32.

3.    Bisno AL. Acute pharyngitis. N Engl J Med . 2001;344:205-211.

4.    Del Mar CB, Glasziou PP, Sprinks AB. Antibiotics for sore throat. Cochrane Database Syst Rev. 2006;(4):CD000023.

5.    Linder JA, Stafford RS. Antibiotic treatment of adults with sore throat by community primary care physicians: a national survey, 1989-1999. JAMA. 2001;286:1181-1186.

6.    Linder JA, Bates DW, Lee GM, et al. Antibiotic treatment of children with sore throat. JAMA. 2005;294:2315-2322.

7.    Hong SY, Taur Y, Jordan MR. Antimicrobial prescribing in the USA for adult acute pharyngitis in relation to treatment guidelines.
J Eval Clin Pract. 2011;17: 1176-1183.

8.    van Driel ML, De Sutter A, Deveugele M, et al Are sore throat patients who hope for antibiotics actually asking for pain relief? Ann Fam Med. 2006;4:494-499.

9.    Venekamp RP, Thompson MJ, Hayward G, et al. Systemic corticosteroids for acute sinusitis. Cochrane Database Syst Rev. 2011;(12):CD008115.

10.  Russell KF, Liang Y, O’Gorman K, et al. Glucocorticoids for croup. Cochrane Database Syst Rev. 2011;(1):CD001955.

11.  Rowe BH, Spooner C, Ducharme F, et al. Early emergency department treatment of acute asthma with systemic corticosteroids. Cochrane Database Syst Rev. 2001;(1):CD002178.

12.  Korb K, Scherer M, Cenot J. Steroids as adjuvant therapy for acute pharyngitis in ambulatory patients: a systematic review. Ann Fam Med. 2010;8:58-63.

13.  Olympia RP, Khine H, Avner JR. Effectiveness of oral dexamethasone in the treatment of moderate to severe pharyngitis in children. Arch Pediatr Adolesc Med. 2005;159:278-282.

14.  Manson SC, Brown RE, Cerulli A, et al. The cumulative burden of oral corticosteroid side effects and the economic implications of steroid use. Respir Med. 2009;103:975-994.

15.  Wei JL, Kasperbauer JL, Weaver AL, et al. Efficacy of single-dose dexamethasone as adjuvant therapy for acute pharyngitis. Laryngoscope. 2002;112:87-93.

Issue
The Journal of Family Practice - 62(7)
Issue
The Journal of Family Practice - 62(7)
Page Number
372-374
Page Number
372-374
Publications
Publications
Topics
Article Type
Display Headline
Corticosteroids for a sore throat?
Display Headline
Corticosteroids for a sore throat?
Legacy Keywords
Keri Bergeson; MD; Nina Rogers; MD; Shailendra Prasad; MBBS; MPH; sore throat; corticosteroids; PURLs; steroids; NSAIDs
Legacy Keywords
Keri Bergeson; MD; Nina Rogers; MD; Shailendra Prasad; MBBS; MPH; sore throat; corticosteroids; PURLs; steroids; NSAIDs
Sections
PURLs Copyright

Copyright © 2013. The Family Physicians Inquiries Network. All Rights Reserved.

Disallow All Ads
Alternative CME
Use ProPublica
Hide sidebar & use full width
render the right sidebar.
Conference Recap Checkbox
Not Conference Recap
Clinical Edge
Display the Slideshow in this Article
Article PDF Media
Media Files

A Spoonful of Honey Helps a Coughing Child Sleep

Article Type
Changed
Tue, 12/13/2016 - 12:08
Display Headline
A Spoonful of Honey Helps a Coughing Child Sleep
Finally, we have a safe and effective alternative to OTC cough and cold remedies for young children with upper respiratory infections.

PRACTICE CHANGER
When a parent brings in a child (ages 1 to 5 years) with cough, runny nose, and other symptoms of a viral upper respiratory infection (URI), recommend that honey be given at bedtime.1

STRENGTH OF RECOMMENDATION
A: Based on a well-designed, randomized controlled trial.1

ILLUSTRATIVE CASE
A mother brings in her 18-month-old son because he’s had a runny nose and low-grade fever for the past four days—and a cough that kept them both up last night. You diagnose a viral URI, and she requests a strong cough medicine so he (and she) can get a good night’s sleep. What can you recommend that is both safe and effective for a child of this age?

Primary care office visits by coughing kids with URIs are common. In addition to the cost of such visits, Americans spend about $3.5 billion a year on OTC cough and cold remedies—often giving them to young children.

It’s not enough to tell parents what not to do
As clinicians (and parents), we understand the desire to give a coughing child something to ease the symptoms. We also know that OTC cough and cold medications can lead to serious complications and even death. Between 1983 and 2007, 118 pediatric deaths were attributed to the misuse of such preparations.2 And in a three-year span (2005 to 2008), the American Association of Poison Control Centers received 64,658 calls related to exposure to cough and cold remedies in children younger than 2; 28 of them resulted in a major adverse reaction or death.3

The FDA recommends against the use of OTC cough and cold medications in children younger than 2 years,4 and the American Academy of Pediatrics has issued strict warnings about the use of OTC cough and cold preparations in children younger than 6.5 But warning parents of the dangers of giving these remedies to young children without offering an alternative doesn’t satisfy anyone’s needs, and many parents continue to use them.

What about honey?
A study published in 2007 evaluated buckwheat honey and found it to be superior to no treatment and equal to honey-flavored dextromethorphan in reducing cough severity and improving sleep for children and their parents.6 Honey is known to have both antioxidant and antimicrobial properties—a possible scientific explanation for its effect. Before honey could be recommended for kids with URIs, however, more evidence of its efficacy was needed.

STUDY SUMMARY
Honey reduces cough frequency and severity

Cohen et al sought to determine whether honey, administered before bedtime, would decrease coughing in children between the ages of 1 and 5—and improve sleep for both the children and their caregivers.1 They enrolled 300 children with a nocturnal cough of < 7 days’ duration and a diagnosis of URI in a one-night study.

Children were excluded if they had signs or symptoms of asthma, pneumonia, sinusitis, allergic rhinitis, or laryngotracheobronchitis, or if they had been given any cough remedy (including honey) the night before. Parents completed a five-question survey, using a 7-point Likert scale, to assess the child’s cough and both the child’s and parents’ sleep the previous night. Only children whose parents rated their child’s cough severity ≥ 3 for two of the three related questions were included in the trial.

The study had a double-blind randomized design, with four treatment arms. Three groups received 10 g (about 1.5 tsp) of one of three types of honey: eucalyptus, citrus, or labiatae (derived from plants including sage, mint, and thyme); the fourth group received a placebo of silan date extract, which is similar to honey in color, texture, and taste.

Children in all four groups received the preparation 30 minutes before bedtime. Neither the parents, the physicians, nor the study coordinators knew which preparation the children received. The following day, research assistants telephoned the parent who had completed the initial survey and asked the same five questions. The primary outcome measure was the change in cough frequency from the night before to the night after treatment. Secondary measures included cough severity and the effect on sleep for both the child and the parent.

Of the 300 children initially enrolled, 270 (90%) completed the trial, with an even distribution among the groups. While there were improvements across all outcomes for both the treatment and placebo groups, the changes were statistically significant only in the treatment groups.

There were no significant differences in efficacy noted among the three types of honey. Adverse effects (stomachache, nausea, or vomiting) were noted by four parents in the treatment groups and one in the placebo group, a difference that was not statistically significant.

 

 

WHAT’S NEW?
We have more evidence of honey’s efficacy

For children older than 1 with a viral URI, we can now recommend 1.5 tsp honey to be given prior to bedtime as a cough remedy. This may reduce the use of potentially harmful and often ineffective OTC cough and cold remedies.

CAVEATS
Honey is unsafe for the youngest children
An obvious limitation of this study was its brevity. Although one night of improved cough and sleep is important, a study that showed honey’s sustained benefit as a cough suppressant would be more convincing. What’s more, there are safety concerns that are age-related.

Honey is considered unsafe for children younger than 1 because of the risk of botulism. And honey has the potential to increase dental caries if it is given nightly for a prolonged period of time.

We do not know whether all varieties of honey will have the same benefit, and the source of store-bought honey is not always identified. The authors of this study received funding from the Honey Board of Israel.

CHALLENGES TO IMPLEMENTATION
Parents may be reluctant to abandon OTCs

Changing the behavior of parents and other caregivers who are accustomed to treating children with OTC cough and cold remedies is likely to be an uphill battle.

Because honey is readily available, however—often as close as the pantry—and perceived to be safe and nutritious, a recommendation from a trusted clinician could go a long way toward aiding the implementation of its use as an alternative symptom-reliever for kids with cough.           

REFERENCES
1. Cohen HA, Rozen J, Kristal H, et al. Effect of honey on nocturnal cough and sleep quality: a double-blind, randomized, placebo-controlled study. Pediatrics. 2012;130:465-471.

2. Dart RC, Paul IM, Bond GR, et al. Pediatric fatalities associated with over the counter (nonprescription) cough and cold medications. Ann Emerg Med. 2009;53:411-417.

3. Srinivasan A, Budnitz D, Shehab N, et al. Infant deaths associated with cough and cold medications—two states, 2005. JAMA. 2007;297:800-801.

4. FDA. Public health advisory: FDA recommends that over-the-counter (OTC) cough and cold products not be used for Infants and children under 2 years of age. www.fda.gov/Drugs/DrugSafety/PostmarketDrugSafetyInformationforPatientsandProviders/DrugSafetyInformationforHeathcareProfessionals/PublicHealthAdvisories/ucm051137.htm. Accessed May 30, 2013.

5. American Academy of Pediatrics. Withdrawal of cold medicines: addressing parent concerns. www.aap.org/en-us/professional-resources/practice-support/Pages/Withdrawal-of-Cold-Medicines-Addressing-Parent-Concerns.aspx. Accessed May 30, 2013.

6. Paul IM, Beiler J, McMonagle A, et al. Effect of honey, dextromethorphan, and no treatment on nocturnal cough and sleep quality for coughing children and their parents. Arch Pediatr Adolesc Med. 2007;161:1140-1146.

Acknowledgement
The PURLs Surveillance System was developed with support from Grant Number UL1RR024999 from the National Center for Research Resources, a Clinical Translational Science Award to the University of Chicago. The content is solely the responsibility of the authors and does not necessarily represent the official views of the National Center for Research Resources or the National Institutes of Health.

Copyright © 2013. The Family Physicians Inquiries Network. All rights reserved.

Reprinted with permission from the Family Physicians Inquiries Network and The Journal of Family Practice. 2013;62(3):145-147.

Author and Disclosure Information

Evan Ashkin, MD, Anne Mounsey, MD

Issue
Clinician Reviews - 23(7)
Publications
Topics
Page Number
28
Legacy Keywords
cough, child, toddler, pediatric patient, OTC, over the counter, cough medicine, cold remedies, honey, viral upper respiratory infection, dextromethorphan
Sections
Author and Disclosure Information

Evan Ashkin, MD, Anne Mounsey, MD

Author and Disclosure Information

Evan Ashkin, MD, Anne Mounsey, MD

Finally, we have a safe and effective alternative to OTC cough and cold remedies for young children with upper respiratory infections.
Finally, we have a safe and effective alternative to OTC cough and cold remedies for young children with upper respiratory infections.

PRACTICE CHANGER
When a parent brings in a child (ages 1 to 5 years) with cough, runny nose, and other symptoms of a viral upper respiratory infection (URI), recommend that honey be given at bedtime.1

STRENGTH OF RECOMMENDATION
A: Based on a well-designed, randomized controlled trial.1

ILLUSTRATIVE CASE
A mother brings in her 18-month-old son because he’s had a runny nose and low-grade fever for the past four days—and a cough that kept them both up last night. You diagnose a viral URI, and she requests a strong cough medicine so he (and she) can get a good night’s sleep. What can you recommend that is both safe and effective for a child of this age?

Primary care office visits by coughing kids with URIs are common. In addition to the cost of such visits, Americans spend about $3.5 billion a year on OTC cough and cold remedies—often giving them to young children.

It’s not enough to tell parents what not to do
As clinicians (and parents), we understand the desire to give a coughing child something to ease the symptoms. We also know that OTC cough and cold medications can lead to serious complications and even death. Between 1983 and 2007, 118 pediatric deaths were attributed to the misuse of such preparations.2 And in a three-year span (2005 to 2008), the American Association of Poison Control Centers received 64,658 calls related to exposure to cough and cold remedies in children younger than 2; 28 of them resulted in a major adverse reaction or death.3

The FDA recommends against the use of OTC cough and cold medications in children younger than 2 years,4 and the American Academy of Pediatrics has issued strict warnings about the use of OTC cough and cold preparations in children younger than 6.5 But warning parents of the dangers of giving these remedies to young children without offering an alternative doesn’t satisfy anyone’s needs, and many parents continue to use them.

What about honey?
A study published in 2007 evaluated buckwheat honey and found it to be superior to no treatment and equal to honey-flavored dextromethorphan in reducing cough severity and improving sleep for children and their parents.6 Honey is known to have both antioxidant and antimicrobial properties—a possible scientific explanation for its effect. Before honey could be recommended for kids with URIs, however, more evidence of its efficacy was needed.

STUDY SUMMARY
Honey reduces cough frequency and severity

Cohen et al sought to determine whether honey, administered before bedtime, would decrease coughing in children between the ages of 1 and 5—and improve sleep for both the children and their caregivers.1 They enrolled 300 children with a nocturnal cough of < 7 days’ duration and a diagnosis of URI in a one-night study.

Children were excluded if they had signs or symptoms of asthma, pneumonia, sinusitis, allergic rhinitis, or laryngotracheobronchitis, or if they had been given any cough remedy (including honey) the night before. Parents completed a five-question survey, using a 7-point Likert scale, to assess the child’s cough and both the child’s and parents’ sleep the previous night. Only children whose parents rated their child’s cough severity ≥ 3 for two of the three related questions were included in the trial.

The study had a double-blind randomized design, with four treatment arms. Three groups received 10 g (about 1.5 tsp) of one of three types of honey: eucalyptus, citrus, or labiatae (derived from plants including sage, mint, and thyme); the fourth group received a placebo of silan date extract, which is similar to honey in color, texture, and taste.

Children in all four groups received the preparation 30 minutes before bedtime. Neither the parents, the physicians, nor the study coordinators knew which preparation the children received. The following day, research assistants telephoned the parent who had completed the initial survey and asked the same five questions. The primary outcome measure was the change in cough frequency from the night before to the night after treatment. Secondary measures included cough severity and the effect on sleep for both the child and the parent.

Of the 300 children initially enrolled, 270 (90%) completed the trial, with an even distribution among the groups. While there were improvements across all outcomes for both the treatment and placebo groups, the changes were statistically significant only in the treatment groups.

There were no significant differences in efficacy noted among the three types of honey. Adverse effects (stomachache, nausea, or vomiting) were noted by four parents in the treatment groups and one in the placebo group, a difference that was not statistically significant.

 

 

WHAT’S NEW?
We have more evidence of honey’s efficacy

For children older than 1 with a viral URI, we can now recommend 1.5 tsp honey to be given prior to bedtime as a cough remedy. This may reduce the use of potentially harmful and often ineffective OTC cough and cold remedies.

CAVEATS
Honey is unsafe for the youngest children
An obvious limitation of this study was its brevity. Although one night of improved cough and sleep is important, a study that showed honey’s sustained benefit as a cough suppressant would be more convincing. What’s more, there are safety concerns that are age-related.

Honey is considered unsafe for children younger than 1 because of the risk of botulism. And honey has the potential to increase dental caries if it is given nightly for a prolonged period of time.

We do not know whether all varieties of honey will have the same benefit, and the source of store-bought honey is not always identified. The authors of this study received funding from the Honey Board of Israel.

CHALLENGES TO IMPLEMENTATION
Parents may be reluctant to abandon OTCs

Changing the behavior of parents and other caregivers who are accustomed to treating children with OTC cough and cold remedies is likely to be an uphill battle.

Because honey is readily available, however—often as close as the pantry—and perceived to be safe and nutritious, a recommendation from a trusted clinician could go a long way toward aiding the implementation of its use as an alternative symptom-reliever for kids with cough.           

REFERENCES
1. Cohen HA, Rozen J, Kristal H, et al. Effect of honey on nocturnal cough and sleep quality: a double-blind, randomized, placebo-controlled study. Pediatrics. 2012;130:465-471.

2. Dart RC, Paul IM, Bond GR, et al. Pediatric fatalities associated with over the counter (nonprescription) cough and cold medications. Ann Emerg Med. 2009;53:411-417.

3. Srinivasan A, Budnitz D, Shehab N, et al. Infant deaths associated with cough and cold medications—two states, 2005. JAMA. 2007;297:800-801.

4. FDA. Public health advisory: FDA recommends that over-the-counter (OTC) cough and cold products not be used for Infants and children under 2 years of age. www.fda.gov/Drugs/DrugSafety/PostmarketDrugSafetyInformationforPatientsandProviders/DrugSafetyInformationforHeathcareProfessionals/PublicHealthAdvisories/ucm051137.htm. Accessed May 30, 2013.

5. American Academy of Pediatrics. Withdrawal of cold medicines: addressing parent concerns. www.aap.org/en-us/professional-resources/practice-support/Pages/Withdrawal-of-Cold-Medicines-Addressing-Parent-Concerns.aspx. Accessed May 30, 2013.

6. Paul IM, Beiler J, McMonagle A, et al. Effect of honey, dextromethorphan, and no treatment on nocturnal cough and sleep quality for coughing children and their parents. Arch Pediatr Adolesc Med. 2007;161:1140-1146.

Acknowledgement
The PURLs Surveillance System was developed with support from Grant Number UL1RR024999 from the National Center for Research Resources, a Clinical Translational Science Award to the University of Chicago. The content is solely the responsibility of the authors and does not necessarily represent the official views of the National Center for Research Resources or the National Institutes of Health.

Copyright © 2013. The Family Physicians Inquiries Network. All rights reserved.

Reprinted with permission from the Family Physicians Inquiries Network and The Journal of Family Practice. 2013;62(3):145-147.

PRACTICE CHANGER
When a parent brings in a child (ages 1 to 5 years) with cough, runny nose, and other symptoms of a viral upper respiratory infection (URI), recommend that honey be given at bedtime.1

STRENGTH OF RECOMMENDATION
A: Based on a well-designed, randomized controlled trial.1

ILLUSTRATIVE CASE
A mother brings in her 18-month-old son because he’s had a runny nose and low-grade fever for the past four days—and a cough that kept them both up last night. You diagnose a viral URI, and she requests a strong cough medicine so he (and she) can get a good night’s sleep. What can you recommend that is both safe and effective for a child of this age?

Primary care office visits by coughing kids with URIs are common. In addition to the cost of such visits, Americans spend about $3.5 billion a year on OTC cough and cold remedies—often giving them to young children.

It’s not enough to tell parents what not to do
As clinicians (and parents), we understand the desire to give a coughing child something to ease the symptoms. We also know that OTC cough and cold medications can lead to serious complications and even death. Between 1983 and 2007, 118 pediatric deaths were attributed to the misuse of such preparations.2 And in a three-year span (2005 to 2008), the American Association of Poison Control Centers received 64,658 calls related to exposure to cough and cold remedies in children younger than 2; 28 of them resulted in a major adverse reaction or death.3

The FDA recommends against the use of OTC cough and cold medications in children younger than 2 years,4 and the American Academy of Pediatrics has issued strict warnings about the use of OTC cough and cold preparations in children younger than 6.5 But warning parents of the dangers of giving these remedies to young children without offering an alternative doesn’t satisfy anyone’s needs, and many parents continue to use them.

What about honey?
A study published in 2007 evaluated buckwheat honey and found it to be superior to no treatment and equal to honey-flavored dextromethorphan in reducing cough severity and improving sleep for children and their parents.6 Honey is known to have both antioxidant and antimicrobial properties—a possible scientific explanation for its effect. Before honey could be recommended for kids with URIs, however, more evidence of its efficacy was needed.

STUDY SUMMARY
Honey reduces cough frequency and severity

Cohen et al sought to determine whether honey, administered before bedtime, would decrease coughing in children between the ages of 1 and 5—and improve sleep for both the children and their caregivers.1 They enrolled 300 children with a nocturnal cough of < 7 days’ duration and a diagnosis of URI in a one-night study.

Children were excluded if they had signs or symptoms of asthma, pneumonia, sinusitis, allergic rhinitis, or laryngotracheobronchitis, or if they had been given any cough remedy (including honey) the night before. Parents completed a five-question survey, using a 7-point Likert scale, to assess the child’s cough and both the child’s and parents’ sleep the previous night. Only children whose parents rated their child’s cough severity ≥ 3 for two of the three related questions were included in the trial.

The study had a double-blind randomized design, with four treatment arms. Three groups received 10 g (about 1.5 tsp) of one of three types of honey: eucalyptus, citrus, or labiatae (derived from plants including sage, mint, and thyme); the fourth group received a placebo of silan date extract, which is similar to honey in color, texture, and taste.

Children in all four groups received the preparation 30 minutes before bedtime. Neither the parents, the physicians, nor the study coordinators knew which preparation the children received. The following day, research assistants telephoned the parent who had completed the initial survey and asked the same five questions. The primary outcome measure was the change in cough frequency from the night before to the night after treatment. Secondary measures included cough severity and the effect on sleep for both the child and the parent.

Of the 300 children initially enrolled, 270 (90%) completed the trial, with an even distribution among the groups. While there were improvements across all outcomes for both the treatment and placebo groups, the changes were statistically significant only in the treatment groups.

There were no significant differences in efficacy noted among the three types of honey. Adverse effects (stomachache, nausea, or vomiting) were noted by four parents in the treatment groups and one in the placebo group, a difference that was not statistically significant.

 

 

WHAT’S NEW?
We have more evidence of honey’s efficacy

For children older than 1 with a viral URI, we can now recommend 1.5 tsp honey to be given prior to bedtime as a cough remedy. This may reduce the use of potentially harmful and often ineffective OTC cough and cold remedies.

CAVEATS
Honey is unsafe for the youngest children
An obvious limitation of this study was its brevity. Although one night of improved cough and sleep is important, a study that showed honey’s sustained benefit as a cough suppressant would be more convincing. What’s more, there are safety concerns that are age-related.

Honey is considered unsafe for children younger than 1 because of the risk of botulism. And honey has the potential to increase dental caries if it is given nightly for a prolonged period of time.

We do not know whether all varieties of honey will have the same benefit, and the source of store-bought honey is not always identified. The authors of this study received funding from the Honey Board of Israel.

CHALLENGES TO IMPLEMENTATION
Parents may be reluctant to abandon OTCs

Changing the behavior of parents and other caregivers who are accustomed to treating children with OTC cough and cold remedies is likely to be an uphill battle.

Because honey is readily available, however—often as close as the pantry—and perceived to be safe and nutritious, a recommendation from a trusted clinician could go a long way toward aiding the implementation of its use as an alternative symptom-reliever for kids with cough.           

REFERENCES
1. Cohen HA, Rozen J, Kristal H, et al. Effect of honey on nocturnal cough and sleep quality: a double-blind, randomized, placebo-controlled study. Pediatrics. 2012;130:465-471.

2. Dart RC, Paul IM, Bond GR, et al. Pediatric fatalities associated with over the counter (nonprescription) cough and cold medications. Ann Emerg Med. 2009;53:411-417.

3. Srinivasan A, Budnitz D, Shehab N, et al. Infant deaths associated with cough and cold medications—two states, 2005. JAMA. 2007;297:800-801.

4. FDA. Public health advisory: FDA recommends that over-the-counter (OTC) cough and cold products not be used for Infants and children under 2 years of age. www.fda.gov/Drugs/DrugSafety/PostmarketDrugSafetyInformationforPatientsandProviders/DrugSafetyInformationforHeathcareProfessionals/PublicHealthAdvisories/ucm051137.htm. Accessed May 30, 2013.

5. American Academy of Pediatrics. Withdrawal of cold medicines: addressing parent concerns. www.aap.org/en-us/professional-resources/practice-support/Pages/Withdrawal-of-Cold-Medicines-Addressing-Parent-Concerns.aspx. Accessed May 30, 2013.

6. Paul IM, Beiler J, McMonagle A, et al. Effect of honey, dextromethorphan, and no treatment on nocturnal cough and sleep quality for coughing children and their parents. Arch Pediatr Adolesc Med. 2007;161:1140-1146.

Acknowledgement
The PURLs Surveillance System was developed with support from Grant Number UL1RR024999 from the National Center for Research Resources, a Clinical Translational Science Award to the University of Chicago. The content is solely the responsibility of the authors and does not necessarily represent the official views of the National Center for Research Resources or the National Institutes of Health.

Copyright © 2013. The Family Physicians Inquiries Network. All rights reserved.

Reprinted with permission from the Family Physicians Inquiries Network and The Journal of Family Practice. 2013;62(3):145-147.

Issue
Clinician Reviews - 23(7)
Issue
Clinician Reviews - 23(7)
Page Number
28
Page Number
28
Publications
Publications
Topics
Article Type
Display Headline
A Spoonful of Honey Helps a Coughing Child Sleep
Display Headline
A Spoonful of Honey Helps a Coughing Child Sleep
Legacy Keywords
cough, child, toddler, pediatric patient, OTC, over the counter, cough medicine, cold remedies, honey, viral upper respiratory infection, dextromethorphan
Legacy Keywords
cough, child, toddler, pediatric patient, OTC, over the counter, cough medicine, cold remedies, honey, viral upper respiratory infection, dextromethorphan
Sections
Article Source

PURLs Copyright

Inside the Article

Suspect Carpal Tunnel? Try This

Article Type
Changed
Tue, 12/13/2016 - 12:08
Display Headline
Suspect Carpal Tunnel? Try This
An easy-to-administer modification of the traditional Phalen’s test for carpal tunnel syndrome increases the value of this diagnostic tool.

PRACTICE CHANGER

For best results, use the modified Phalen’s test (MPT) rather than the traditional Phalen’s when you suspect carpal tunnel syndrome (CTS).1

STRENGTH OF 
RECOMMENDATION
B: Based on a single diagnostic cohort study.

ILLUSTRATIVE CASE

A 60-year-old assembly line worker reports bilateral hand numbness and tingling that frequently awaken her at night. What is the best office test to determine if she has CTS?

CTS is one of the most common causes of disability in the United States.2 Among patients with hand paresthesias, one in five has CTS.2 Factory workers whose jobs involve repetitive hand movements, women, and the elderly are at increased risk.3 If left untreated, the symptoms are likely to become constant, with thenar muscle wasting and weakness.

Traditional diagnostic test 
has only 50% sensitivity
In the traditional Phalen’s test (TPT)—commonly used in an office setting—the patient holds his or her wrists in a position of fixed flexion for one minute. The onset of paresthesias is considered a positive result.

The TPT was found in the study reported here to be 100% specific;1 however, other studies have found a wider range of specificity (33% to 86%).4 The TPT has a sensitivity of only 50%, which increases the risk that cases of CTS will be missed. This is an important consideration, because establishing a diagnosis early in the course of CTS has been shown to minimize disability.5

STUDY SUMMARY
Modified Phalen’s has higher sensitivity

Bilkis et al developed a modified Phalen’s test and compared it with the TPT, as well as with electrodiagnostic studies (EDS)—the gold standard for CTS diagnosis. The MPT begins with the TPT position and adds sensory testing with a Semmes-Weinstein 2.83-unit monofilament.

See how the modified Phalen’s test is done


Courtesy of Clinically Relevant Technologies

The filament is applied perpendicular to the palmar and lateral surface of each distal finger three times, with enough pressure to bend the monofilament. In this study, the test was considered “positive” if the patient did not feel the monofilament in any finger along the distribution of the median nerve. The MPT was “negative” if the patient correctly reported being touched along this distribution. The fifth, or “pinkie,” finger, which is less likely to be affected by CTS, was used as a control.

Participants in the study were adult patients—mostly women between the ages of 27 and 88—at a neurology clinic. Exclusion criteria included cervical radiculopathy, a history of stroke, diabetes, and concomitant neck injury. A total of 66 hands (and 37 participants) underwent TPT and MPT testing by trained examiners, followed by EDS to confirm the findings.

EDS found evidence of CTS in 46 of the 66 hands studied. The MPT correctly identified 39 of the 46, while the TPT correctly identified 23. Both the traditional and the modified Phalen’s tests were found to be 100% specific, but the sensitivity of the MPT was 85% (95% confidence interval [CI], 71% to 93%), compared with 50% (95% CI, 35% to 65%) for the TPT.

WHAT’S NEW
Better results can be achieved in seconds

The addition of monofilament testing to the TPT increases the sensitivity in identifying CTS. The MPT is simple to learn and, based on our observations, adds only about 10 to 15 seconds to the clinical exam.

CAVEATS
Modification is untested in primary care
A diagnosis of CTS is rarely made on the basis of one test, but rather on a set of signs, symptoms, and physical exam maneuvers. The added value of the MPT needs to be evaluated in the larger context of the comprehensive clinical examination for CTS.6

Notably, the study participants were seen in a neurology clinic, which suggests that they may have had more advanced CTS than typical primary care patients. That would help explain the 100% specificity of both the traditional and modified tests reported by the researchers. The sensitivity of the MPT may therefore be lower in a family practice because the spectrum of disease may be wider. Another study is needed to evaluate the performance of the MPT in a primary care setting.

The monofilament used (Semmes-Weinstein 2.83) is not the same as the typical 5.07 (10-g) monofilament used in diabetic foot screenings. Using this heavier monofilament with a stronger pressure point would likely decrease the sensitivity of the MPT.

CHALLENGES TO IMPLEMENTATION
Taking the time, obtaining the monofilament

Additional time to obtain the correct monofilament and administer the MPT are the key challenges to implementation.

REFERENCES
1. Bilkis S, Loveman DM, Eldridge JA, et al. Modified Phalen’s test as an aid in diagnosing carpal tunnel syndrome. Arthritis Care Res. 2012;64:287-289.

 

 

2. Atroshi I, Gummesson C, Johnsson R, et al. Prevalence of carpal tunnel syndrome in a general population. JAMA. 1999;282:153-158.

3. National Institute of Neurological Disorders and Stroke. Carpal tunnel syndrome fact sheet. National Institutes of Health. July 2012. www.ninds.nih.gov/disorders/carpal_tunnel/detail_carpal_tunnel.htm. Accessed April 15, 2013.

4. McGee SR. Evidence-Based Physical Diagnosis. 3rd ed. Philadelphia, PA: Saunders; 2012:chap 62.

5. Daniell WE, Fulton-Kehoe D, Franklin GM. Work-related carpal tunnel syndrome in Washington State workers’ compensation: utilization of surgery and the duration of lost work. Am J Ind Med. 2009;52:931-942.

6. D’Arcy CA, McGee S. Does this patient have carpal tunnel syndrome? JAMA. 2000;282:3110-3117.

ACKNOWLEDGEMENT
The PURLs Surveillance System was developed with support from Grant Number UL1RR024999 from the National Center for Research Resources, a Clinical Translational Science Award to the University of Chicago. The content is solely the responsibility of the authors and does not necessarily represent the official views of the National Center for Research Resources or the National Institutes of Health.

Copyright © 2013. The Family Physicians Inquiries Network. All rights reserved.

Reprinted with permission from the Family Physicians Inquiries Network and The Journal of Family Practice. 2013;62(5):253-254.

Author and Disclosure Information

Sonia Oyola, MD, Kohar Jones, MD, Goutham Rao, MD

Issue
Clinician Reviews - 23(6)
Publications
Topics
Page Number
22
Legacy Keywords
carpal tunnel syndrome, Phalen’s test, modified Phalen’s test, repetitive hand movements, paresthesia, Semmes-Weinstein, monofilament
Sections
Author and Disclosure Information

Sonia Oyola, MD, Kohar Jones, MD, Goutham Rao, MD

Author and Disclosure Information

Sonia Oyola, MD, Kohar Jones, MD, Goutham Rao, MD

An easy-to-administer modification of the traditional Phalen’s test for carpal tunnel syndrome increases the value of this diagnostic tool.
An easy-to-administer modification of the traditional Phalen’s test for carpal tunnel syndrome increases the value of this diagnostic tool.

PRACTICE CHANGER

For best results, use the modified Phalen’s test (MPT) rather than the traditional Phalen’s when you suspect carpal tunnel syndrome (CTS).1

STRENGTH OF 
RECOMMENDATION
B: Based on a single diagnostic cohort study.

ILLUSTRATIVE CASE

A 60-year-old assembly line worker reports bilateral hand numbness and tingling that frequently awaken her at night. What is the best office test to determine if she has CTS?

CTS is one of the most common causes of disability in the United States.2 Among patients with hand paresthesias, one in five has CTS.2 Factory workers whose jobs involve repetitive hand movements, women, and the elderly are at increased risk.3 If left untreated, the symptoms are likely to become constant, with thenar muscle wasting and weakness.

Traditional diagnostic test 
has only 50% sensitivity
In the traditional Phalen’s test (TPT)—commonly used in an office setting—the patient holds his or her wrists in a position of fixed flexion for one minute. The onset of paresthesias is considered a positive result.

The TPT was found in the study reported here to be 100% specific;1 however, other studies have found a wider range of specificity (33% to 86%).4 The TPT has a sensitivity of only 50%, which increases the risk that cases of CTS will be missed. This is an important consideration, because establishing a diagnosis early in the course of CTS has been shown to minimize disability.5

STUDY SUMMARY
Modified Phalen’s has higher sensitivity

Bilkis et al developed a modified Phalen’s test and compared it with the TPT, as well as with electrodiagnostic studies (EDS)—the gold standard for CTS diagnosis. The MPT begins with the TPT position and adds sensory testing with a Semmes-Weinstein 2.83-unit monofilament.

See how the modified Phalen’s test is done


Courtesy of Clinically Relevant Technologies

The filament is applied perpendicular to the palmar and lateral surface of each distal finger three times, with enough pressure to bend the monofilament. In this study, the test was considered “positive” if the patient did not feel the monofilament in any finger along the distribution of the median nerve. The MPT was “negative” if the patient correctly reported being touched along this distribution. The fifth, or “pinkie,” finger, which is less likely to be affected by CTS, was used as a control.

Participants in the study were adult patients—mostly women between the ages of 27 and 88—at a neurology clinic. Exclusion criteria included cervical radiculopathy, a history of stroke, diabetes, and concomitant neck injury. A total of 66 hands (and 37 participants) underwent TPT and MPT testing by trained examiners, followed by EDS to confirm the findings.

EDS found evidence of CTS in 46 of the 66 hands studied. The MPT correctly identified 39 of the 46, while the TPT correctly identified 23. Both the traditional and the modified Phalen’s tests were found to be 100% specific, but the sensitivity of the MPT was 85% (95% confidence interval [CI], 71% to 93%), compared with 50% (95% CI, 35% to 65%) for the TPT.

WHAT’S NEW
Better results can be achieved in seconds

The addition of monofilament testing to the TPT increases the sensitivity in identifying CTS. The MPT is simple to learn and, based on our observations, adds only about 10 to 15 seconds to the clinical exam.

CAVEATS
Modification is untested in primary care
A diagnosis of CTS is rarely made on the basis of one test, but rather on a set of signs, symptoms, and physical exam maneuvers. The added value of the MPT needs to be evaluated in the larger context of the comprehensive clinical examination for CTS.6

Notably, the study participants were seen in a neurology clinic, which suggests that they may have had more advanced CTS than typical primary care patients. That would help explain the 100% specificity of both the traditional and modified tests reported by the researchers. The sensitivity of the MPT may therefore be lower in a family practice because the spectrum of disease may be wider. Another study is needed to evaluate the performance of the MPT in a primary care setting.

The monofilament used (Semmes-Weinstein 2.83) is not the same as the typical 5.07 (10-g) monofilament used in diabetic foot screenings. Using this heavier monofilament with a stronger pressure point would likely decrease the sensitivity of the MPT.

CHALLENGES TO IMPLEMENTATION
Taking the time, obtaining the monofilament

Additional time to obtain the correct monofilament and administer the MPT are the key challenges to implementation.

REFERENCES
1. Bilkis S, Loveman DM, Eldridge JA, et al. Modified Phalen’s test as an aid in diagnosing carpal tunnel syndrome. Arthritis Care Res. 2012;64:287-289.

 

 

2. Atroshi I, Gummesson C, Johnsson R, et al. Prevalence of carpal tunnel syndrome in a general population. JAMA. 1999;282:153-158.

3. National Institute of Neurological Disorders and Stroke. Carpal tunnel syndrome fact sheet. National Institutes of Health. July 2012. www.ninds.nih.gov/disorders/carpal_tunnel/detail_carpal_tunnel.htm. Accessed April 15, 2013.

4. McGee SR. Evidence-Based Physical Diagnosis. 3rd ed. Philadelphia, PA: Saunders; 2012:chap 62.

5. Daniell WE, Fulton-Kehoe D, Franklin GM. Work-related carpal tunnel syndrome in Washington State workers’ compensation: utilization of surgery and the duration of lost work. Am J Ind Med. 2009;52:931-942.

6. D’Arcy CA, McGee S. Does this patient have carpal tunnel syndrome? JAMA. 2000;282:3110-3117.

ACKNOWLEDGEMENT
The PURLs Surveillance System was developed with support from Grant Number UL1RR024999 from the National Center for Research Resources, a Clinical Translational Science Award to the University of Chicago. The content is solely the responsibility of the authors and does not necessarily represent the official views of the National Center for Research Resources or the National Institutes of Health.

Copyright © 2013. The Family Physicians Inquiries Network. All rights reserved.

Reprinted with permission from the Family Physicians Inquiries Network and The Journal of Family Practice. 2013;62(5):253-254.

PRACTICE CHANGER

For best results, use the modified Phalen’s test (MPT) rather than the traditional Phalen’s when you suspect carpal tunnel syndrome (CTS).1

STRENGTH OF 
RECOMMENDATION
B: Based on a single diagnostic cohort study.

ILLUSTRATIVE CASE

A 60-year-old assembly line worker reports bilateral hand numbness and tingling that frequently awaken her at night. What is the best office test to determine if she has CTS?

CTS is one of the most common causes of disability in the United States.2 Among patients with hand paresthesias, one in five has CTS.2 Factory workers whose jobs involve repetitive hand movements, women, and the elderly are at increased risk.3 If left untreated, the symptoms are likely to become constant, with thenar muscle wasting and weakness.

Traditional diagnostic test 
has only 50% sensitivity
In the traditional Phalen’s test (TPT)—commonly used in an office setting—the patient holds his or her wrists in a position of fixed flexion for one minute. The onset of paresthesias is considered a positive result.

The TPT was found in the study reported here to be 100% specific;1 however, other studies have found a wider range of specificity (33% to 86%).4 The TPT has a sensitivity of only 50%, which increases the risk that cases of CTS will be missed. This is an important consideration, because establishing a diagnosis early in the course of CTS has been shown to minimize disability.5

STUDY SUMMARY
Modified Phalen’s has higher sensitivity

Bilkis et al developed a modified Phalen’s test and compared it with the TPT, as well as with electrodiagnostic studies (EDS)—the gold standard for CTS diagnosis. The MPT begins with the TPT position and adds sensory testing with a Semmes-Weinstein 2.83-unit monofilament.

See how the modified Phalen’s test is done


Courtesy of Clinically Relevant Technologies

The filament is applied perpendicular to the palmar and lateral surface of each distal finger three times, with enough pressure to bend the monofilament. In this study, the test was considered “positive” if the patient did not feel the monofilament in any finger along the distribution of the median nerve. The MPT was “negative” if the patient correctly reported being touched along this distribution. The fifth, or “pinkie,” finger, which is less likely to be affected by CTS, was used as a control.

Participants in the study were adult patients—mostly women between the ages of 27 and 88—at a neurology clinic. Exclusion criteria included cervical radiculopathy, a history of stroke, diabetes, and concomitant neck injury. A total of 66 hands (and 37 participants) underwent TPT and MPT testing by trained examiners, followed by EDS to confirm the findings.

EDS found evidence of CTS in 46 of the 66 hands studied. The MPT correctly identified 39 of the 46, while the TPT correctly identified 23. Both the traditional and the modified Phalen’s tests were found to be 100% specific, but the sensitivity of the MPT was 85% (95% confidence interval [CI], 71% to 93%), compared with 50% (95% CI, 35% to 65%) for the TPT.

WHAT’S NEW
Better results can be achieved in seconds

The addition of monofilament testing to the TPT increases the sensitivity in identifying CTS. The MPT is simple to learn and, based on our observations, adds only about 10 to 15 seconds to the clinical exam.

CAVEATS
Modification is untested in primary care
A diagnosis of CTS is rarely made on the basis of one test, but rather on a set of signs, symptoms, and physical exam maneuvers. The added value of the MPT needs to be evaluated in the larger context of the comprehensive clinical examination for CTS.6

Notably, the study participants were seen in a neurology clinic, which suggests that they may have had more advanced CTS than typical primary care patients. That would help explain the 100% specificity of both the traditional and modified tests reported by the researchers. The sensitivity of the MPT may therefore be lower in a family practice because the spectrum of disease may be wider. Another study is needed to evaluate the performance of the MPT in a primary care setting.

The monofilament used (Semmes-Weinstein 2.83) is not the same as the typical 5.07 (10-g) monofilament used in diabetic foot screenings. Using this heavier monofilament with a stronger pressure point would likely decrease the sensitivity of the MPT.

CHALLENGES TO IMPLEMENTATION
Taking the time, obtaining the monofilament

Additional time to obtain the correct monofilament and administer the MPT are the key challenges to implementation.

REFERENCES
1. Bilkis S, Loveman DM, Eldridge JA, et al. Modified Phalen’s test as an aid in diagnosing carpal tunnel syndrome. Arthritis Care Res. 2012;64:287-289.

 

 

2. Atroshi I, Gummesson C, Johnsson R, et al. Prevalence of carpal tunnel syndrome in a general population. JAMA. 1999;282:153-158.

3. National Institute of Neurological Disorders and Stroke. Carpal tunnel syndrome fact sheet. National Institutes of Health. July 2012. www.ninds.nih.gov/disorders/carpal_tunnel/detail_carpal_tunnel.htm. Accessed April 15, 2013.

4. McGee SR. Evidence-Based Physical Diagnosis. 3rd ed. Philadelphia, PA: Saunders; 2012:chap 62.

5. Daniell WE, Fulton-Kehoe D, Franklin GM. Work-related carpal tunnel syndrome in Washington State workers’ compensation: utilization of surgery and the duration of lost work. Am J Ind Med. 2009;52:931-942.

6. D’Arcy CA, McGee S. Does this patient have carpal tunnel syndrome? JAMA. 2000;282:3110-3117.

ACKNOWLEDGEMENT
The PURLs Surveillance System was developed with support from Grant Number UL1RR024999 from the National Center for Research Resources, a Clinical Translational Science Award to the University of Chicago. The content is solely the responsibility of the authors and does not necessarily represent the official views of the National Center for Research Resources or the National Institutes of Health.

Copyright © 2013. The Family Physicians Inquiries Network. All rights reserved.

Reprinted with permission from the Family Physicians Inquiries Network and The Journal of Family Practice. 2013;62(5):253-254.

Issue
Clinician Reviews - 23(6)
Issue
Clinician Reviews - 23(6)
Page Number
22
Page Number
22
Publications
Publications
Topics
Article Type
Display Headline
Suspect Carpal Tunnel? Try This
Display Headline
Suspect Carpal Tunnel? Try This
Legacy Keywords
carpal tunnel syndrome, Phalen’s test, modified Phalen’s test, repetitive hand movements, paresthesia, Semmes-Weinstein, monofilament
Legacy Keywords
carpal tunnel syndrome, Phalen’s test, modified Phalen’s test, repetitive hand movements, paresthesia, Semmes-Weinstein, monofilament
Sections
Article Source

PURLs Copyright

Inside the Article

Trauma care—don’t delay with TXA

Article Type
Changed
Tue, 07/14/2020 - 14:30
Display Headline
Trauma care—don’t delay with TXA
PRACTICE CHANGER

Ensure that patients who incur serious trauma receive tranexamic acid (TXA) within 3 hours of the injury.1

STRENGTH OF RECOMMENDATION

B: An analysis of a large randomized controlled trial (RCT).

CRASH-2 collaborators; Roberts I, Shakur H, Afolabi A, et al. The importance of early treatment with tranexamic acid in bleeding trauma patients: an exploratory analysis of the CRASH-2 randomised controlled trial. Lancet. 2011;377:1096-1101.

 

ILLUSTRATIVE CASE

You are working in the emergency department (ED) of a rural hospital when a 26-year-old man is brought in with multiple injuries sustained in a high-speed collision and motor vehicle rollover. The nearest trauma center is more than an hour away. Should you administer TXA to the patient before transferring him?

Trauma is a leading cause of death among those younger than 40 years, and in 30% of such fatalities, hemorrhage is the cause.2 Tranexamic acid (TXA) minimizes blood loss by inhibiting lysine binding sites on plasminogen, thereby preventing the conversion of plasminogen to plasmin. This inhibits fibrinolysis and reduces clot breakdown, resulting in a reduction in bleeding.3

TXA has a proven track record
TXA is not new. It has been used to minimize blood loss associated with surgery for decades.4 A retrospective cohort study involving recent military engagements in Afghanistan showed a reduction in both coagulopathy and mortality in trauma patients who were given TXA.5 The CRASH-2 randomized controlled trial (RCT), initially published in 20106 and further analyzed in the study detailed below,1 was the first extensive multicenter trial to evaluate the use of TXA in civilian trauma care.

STUDY SUMMARY: TXA saves lives— within a 3-hour window

CRASH-2 studied the early administration of TXA in adult trauma patients in 274 hospitals in 40 countries.1,6 Patients (N = 20,211) were enrolled if the treating physician judged them to have or be at risk for significant hemorrhage and were randomized to either TXA or placebo, administered in identical-looking packs. Within 8 hours of injury, participants received a 1-g intravenous (IV) loading dose of either TXA or placebo over 10 minutes; a 1-g infusion over 8 hours followed. Patients and study staff were blinded to the treatment groups.

The primary outcome was overall mortality in the 4 weeks after injury. Secondary outcomes included vascular occlusive events (myocardial infarction, stroke, pulmonary embolism, and deep venous thrombosis), major surgical intervention, quantity of blood transfusion (if any), and cause of death (bleeding, vascular occlusion, multi-organ failure, head injury, or other cause). In this analysis, the researchers considered the effect of TXA on mortality based on time to administration of treatment after injury, severity of blood loss as assessed by systolic blood pressure, Glasgow coma scale score, and type of injury. All analyses were intention to treat, and follow-up was 99.6%.

TXA reduced all-cause mortality in the first month after trauma (relative risk [RR] = 0.91; 95% confidence interval [CI], 0.85-0.97; P = .0035; number needed to treat [NNT] = 68). There were 3076 deaths from all causes in both groups, 35% of which were the result of bleeding. Among patients who received TXA, the overall risk of death due to bleeding was 4.9%, vs 5.4% in the placebo group (RR = 0.85; 95% CI, 0.76-0.96; P = .0077; NNT = 119).

After 3 hours, TXA may do more harm than good
For those treated with TXA within the first hour of injury, the risk of death due to bleeding was 5.3%, vs 7.7% for the placebo group (RR = 0.68; 95% CI, 0.57-0.82; P<.0001; NNT = 41). Giving TXA between one and 3 hours of injury also reduced the risk of death due to bleeding, to 4.8% vs 6.1% for the placebo group (RR = 0.79; 95% CI, 0.64-0.97; P = .03; NNT = 77).

TXA administered more than 3 hours after injury, however, appeared to increase the risk of death due to bleeding, to 4.4% compared with 3.1% for the placebo group (RR = 1.44; 95% CI, 1.12-1.84; P = .004; number needed to harm = 77). The researchers found no evidence that TXA’s effect on death due to bleeding varied on the basis of systolic pressure, Glasgow coma score, or type of injury.

The rate of occlusive vascular events over the 4-week study period was similar in both groups (RR = 0.84; 95% CI, 0.68-1.02; P = .08). Of note, the rate of myocardial infarction was reduced by TXA (RR = 0.64; 95% CI, 0.42-0.97; P = .035; NNT = 504).

 

 

 

WHAT’S NEW: Greater emphasis on TXA timing

Current practice for the treatment of traumatic hemorrhage includes fluid resuscitation and the administration of blood products. This analysis of the CRASH-2 refines our understanding of TXA, revealing that the earlier it is given after injury, the better the outcome. A 2011 Cochrane review found only one other small RCT (N = 240), which had findings consistent with the CRASH-2 results.7

TXA is easy to administer and to store and does not require refrigeration or reconstitution prior to administration. TXA has been included in both the US and British Army trauma protocols.8 In addition, TXA is used by National Health Service ambulances in the United Kingdom, and given to all adults and teenagers who incur major traumatic injury.8

CAVEATS: Potential for thromboembolic events, need for high time sensitivity

Because the enrollment criteria for the study were based entirely on clinical findings, there may have been some participants who were not actively bleeding. However, this would have been true for both the treatment and placebo groups and, if anything, would have diluted the effects of TXA.

There was no increase in vaso-occlusive events in the CRASH-2 study. However, some studies of TXA have found an increase in instances of pulmonary embolism, deep vein thrombosis, and ureteral obstruction in patients with genitourinary bleeding.3

This analysis showed that early administration of TXA is the key to its success—and highlighted the importance of avoiding giving it more than 3 hours after traumatic injury. Although most of the 40 countries in which the CRASH-2 study was conducted have less well developed trauma systems than those in the United States, a subgroup analysis of patients in Europe, North America, and Australia (n = 1960) still showed a mortality benefit (RR = 0.63; 95% CI, 0.42-0.94).8

CHALLENGES TO IMPLEMENTATION: Bringing TXA into the mainstream

The acceptance of TXA in trauma care guidelines may be one of the biggest barriers to its use. Currently, the American College of Surgeons does not include the use of TXA in its Advanced Trauma Life Support manual.9

Given the short time window for its benefit, TXA may be most appropriate in the prehospital setting. However, there are no studies of its use in this setting. Lack of knowledge and access are also barriers in the emergency setting, as many ED clinicians, particularly in rural settings, may not yet have access to TXA. Physicians in the United Kingdom have tried a variety of methods, including the unorthodox use of comic books targeted to health care providers,10 in an effort to get the word out.

Acknowledgement

The PURLs Surveillance System was supported in part by Grant Number UL1RR024999 from the National Center for Research Resources, a Clinical Translational Science Award to the University of Chicago. The content is solely the responsibility of the authors and does not necessarily represent the official views of the National Center for Research Resources or the National Institutes of Health.

Files
References

1. CRASH-2 collaborators; Roberts I, Shakur H, Afolabi A, et al. The importance of early treatment with tranexamic acid in bleeding trauma patients: an exploratory analysis of the CRASH-2 randomised controlled trial. Lancet. 2011;377:1096-1101.

2. Kauvar DS, Lefering R, Wade CE. Impact of hemorrhage on trauma outcome: an overview of epidemiology, clinical presentations, and therapeutic considerations. J Trauma. 2006;60(suppl):S3-S11.

3. Cap AP, Baer DG, Orman JA, et al. Tranexamic acid for trauma patients: a critical review of the literature. J Trauma. 2011;71(suppl):S9-S14.

4. Ker K, Edwards P, Perel P, et al. Effect of tranexamic acid on surgical bleeding: systematic review and cumulative meta-analysis. BMJ. 2012;344:e3054.-

5. Morrison JJ, Dubose JJ, Rasmussen TE, et al. Military Application of Tranexamic Acid in Trauma Emergency Resuscitation (MATTERs) Study. Arch Surg. 2012;147:113-119.

6. Shakur H, Roberts I, Bautista R, et al. Effects of tranexamic acid on death, vascular occlusive events, and blood transfusion in trauma patients with significant haemorrhage (CRASH-2): a randomised, placebo-controlled trial. Lancet. 2010;376:23-32.

7. Roberts I, Shakur H, Ker K, et al. Antifibrinolytic drugs for acute traumatic injury. Cochrane Database Syst Rev. 2011;(1):CD004896.-

8. Ker K, Kiriya J, Perel P, et al. Avoidable mortality from giving tranexamic acid to bleeding trauma patients: an estimation based on WHO mortality data, a systematic literature review and data from the CRASH-2 trial. BMC Emerg Med. 2012;12:3.-

9. American College of Surgeons, Committee on Trauma. Advanced Trauma Life Support (ATLS) Student Course Manual. 9th ed. Chicago, IL: American College of Surgeons; 2012.

10. Roberts I, Vieceli E, Duffield P. TXA. London, UK: London School of Hygiene & Tropical Medicine; 2012. Available at: http://download.thelancet.com/flatcontentassets/pdfs/TXA-manga.pdf. Accessed April 23, 2013.

Article PDF
Author and Disclosure Information

Jack C. Wells, MD, MHA
Department of Family and Community Medicine, University of Missouri-Columbia

James J. Stevermer, MD, MSPH
Department of Family and Community Medicine, University of Missouri-Columbia

PURLs EDITOR
Anne Mounsey, MD
University of North Carolina at Chapel Hill

Issue
The Journal of Family Practice - 62(5)
Publications
Page Number
E4-E6
Legacy Keywords
Jack C. Wells; MD; MHA; James J. Stevermer; MD; MSPH; tranexamic acid; TXA; CRASH-2 randomized controlled trial; reduced all-cause mortality; TXA timing; within 3 hours; occlusive vascular events; PURLs
Sections
Files
Files
Author and Disclosure Information

Jack C. Wells, MD, MHA
Department of Family and Community Medicine, University of Missouri-Columbia

James J. Stevermer, MD, MSPH
Department of Family and Community Medicine, University of Missouri-Columbia

PURLs EDITOR
Anne Mounsey, MD
University of North Carolina at Chapel Hill

Author and Disclosure Information

Jack C. Wells, MD, MHA
Department of Family and Community Medicine, University of Missouri-Columbia

James J. Stevermer, MD, MSPH
Department of Family and Community Medicine, University of Missouri-Columbia

PURLs EDITOR
Anne Mounsey, MD
University of North Carolina at Chapel Hill

Article PDF
Article PDF
PRACTICE CHANGER

Ensure that patients who incur serious trauma receive tranexamic acid (TXA) within 3 hours of the injury.1

STRENGTH OF RECOMMENDATION

B: An analysis of a large randomized controlled trial (RCT).

CRASH-2 collaborators; Roberts I, Shakur H, Afolabi A, et al. The importance of early treatment with tranexamic acid in bleeding trauma patients: an exploratory analysis of the CRASH-2 randomised controlled trial. Lancet. 2011;377:1096-1101.

 

ILLUSTRATIVE CASE

You are working in the emergency department (ED) of a rural hospital when a 26-year-old man is brought in with multiple injuries sustained in a high-speed collision and motor vehicle rollover. The nearest trauma center is more than an hour away. Should you administer TXA to the patient before transferring him?

Trauma is a leading cause of death among those younger than 40 years, and in 30% of such fatalities, hemorrhage is the cause.2 Tranexamic acid (TXA) minimizes blood loss by inhibiting lysine binding sites on plasminogen, thereby preventing the conversion of plasminogen to plasmin. This inhibits fibrinolysis and reduces clot breakdown, resulting in a reduction in bleeding.3

TXA has a proven track record
TXA is not new. It has been used to minimize blood loss associated with surgery for decades.4 A retrospective cohort study involving recent military engagements in Afghanistan showed a reduction in both coagulopathy and mortality in trauma patients who were given TXA.5 The CRASH-2 randomized controlled trial (RCT), initially published in 20106 and further analyzed in the study detailed below,1 was the first extensive multicenter trial to evaluate the use of TXA in civilian trauma care.

STUDY SUMMARY: TXA saves lives— within a 3-hour window

CRASH-2 studied the early administration of TXA in adult trauma patients in 274 hospitals in 40 countries.1,6 Patients (N = 20,211) were enrolled if the treating physician judged them to have or be at risk for significant hemorrhage and were randomized to either TXA or placebo, administered in identical-looking packs. Within 8 hours of injury, participants received a 1-g intravenous (IV) loading dose of either TXA or placebo over 10 minutes; a 1-g infusion over 8 hours followed. Patients and study staff were blinded to the treatment groups.

The primary outcome was overall mortality in the 4 weeks after injury. Secondary outcomes included vascular occlusive events (myocardial infarction, stroke, pulmonary embolism, and deep venous thrombosis), major surgical intervention, quantity of blood transfusion (if any), and cause of death (bleeding, vascular occlusion, multi-organ failure, head injury, or other cause). In this analysis, the researchers considered the effect of TXA on mortality based on time to administration of treatment after injury, severity of blood loss as assessed by systolic blood pressure, Glasgow coma scale score, and type of injury. All analyses were intention to treat, and follow-up was 99.6%.

TXA reduced all-cause mortality in the first month after trauma (relative risk [RR] = 0.91; 95% confidence interval [CI], 0.85-0.97; P = .0035; number needed to treat [NNT] = 68). There were 3076 deaths from all causes in both groups, 35% of which were the result of bleeding. Among patients who received TXA, the overall risk of death due to bleeding was 4.9%, vs 5.4% in the placebo group (RR = 0.85; 95% CI, 0.76-0.96; P = .0077; NNT = 119).

After 3 hours, TXA may do more harm than good
For those treated with TXA within the first hour of injury, the risk of death due to bleeding was 5.3%, vs 7.7% for the placebo group (RR = 0.68; 95% CI, 0.57-0.82; P<.0001; NNT = 41). Giving TXA between one and 3 hours of injury also reduced the risk of death due to bleeding, to 4.8% vs 6.1% for the placebo group (RR = 0.79; 95% CI, 0.64-0.97; P = .03; NNT = 77).

TXA administered more than 3 hours after injury, however, appeared to increase the risk of death due to bleeding, to 4.4% compared with 3.1% for the placebo group (RR = 1.44; 95% CI, 1.12-1.84; P = .004; number needed to harm = 77). The researchers found no evidence that TXA’s effect on death due to bleeding varied on the basis of systolic pressure, Glasgow coma score, or type of injury.

The rate of occlusive vascular events over the 4-week study period was similar in both groups (RR = 0.84; 95% CI, 0.68-1.02; P = .08). Of note, the rate of myocardial infarction was reduced by TXA (RR = 0.64; 95% CI, 0.42-0.97; P = .035; NNT = 504).

 

 

 

WHAT’S NEW: Greater emphasis on TXA timing

Current practice for the treatment of traumatic hemorrhage includes fluid resuscitation and the administration of blood products. This analysis of the CRASH-2 refines our understanding of TXA, revealing that the earlier it is given after injury, the better the outcome. A 2011 Cochrane review found only one other small RCT (N = 240), which had findings consistent with the CRASH-2 results.7

TXA is easy to administer and to store and does not require refrigeration or reconstitution prior to administration. TXA has been included in both the US and British Army trauma protocols.8 In addition, TXA is used by National Health Service ambulances in the United Kingdom, and given to all adults and teenagers who incur major traumatic injury.8

CAVEATS: Potential for thromboembolic events, need for high time sensitivity

Because the enrollment criteria for the study were based entirely on clinical findings, there may have been some participants who were not actively bleeding. However, this would have been true for both the treatment and placebo groups and, if anything, would have diluted the effects of TXA.

There was no increase in vaso-occlusive events in the CRASH-2 study. However, some studies of TXA have found an increase in instances of pulmonary embolism, deep vein thrombosis, and ureteral obstruction in patients with genitourinary bleeding.3

This analysis showed that early administration of TXA is the key to its success—and highlighted the importance of avoiding giving it more than 3 hours after traumatic injury. Although most of the 40 countries in which the CRASH-2 study was conducted have less well developed trauma systems than those in the United States, a subgroup analysis of patients in Europe, North America, and Australia (n = 1960) still showed a mortality benefit (RR = 0.63; 95% CI, 0.42-0.94).8

CHALLENGES TO IMPLEMENTATION: Bringing TXA into the mainstream

The acceptance of TXA in trauma care guidelines may be one of the biggest barriers to its use. Currently, the American College of Surgeons does not include the use of TXA in its Advanced Trauma Life Support manual.9

Given the short time window for its benefit, TXA may be most appropriate in the prehospital setting. However, there are no studies of its use in this setting. Lack of knowledge and access are also barriers in the emergency setting, as many ED clinicians, particularly in rural settings, may not yet have access to TXA. Physicians in the United Kingdom have tried a variety of methods, including the unorthodox use of comic books targeted to health care providers,10 in an effort to get the word out.

Acknowledgement

The PURLs Surveillance System was supported in part by Grant Number UL1RR024999 from the National Center for Research Resources, a Clinical Translational Science Award to the University of Chicago. The content is solely the responsibility of the authors and does not necessarily represent the official views of the National Center for Research Resources or the National Institutes of Health.

PRACTICE CHANGER

Ensure that patients who incur serious trauma receive tranexamic acid (TXA) within 3 hours of the injury.1

STRENGTH OF RECOMMENDATION

B: An analysis of a large randomized controlled trial (RCT).

CRASH-2 collaborators; Roberts I, Shakur H, Afolabi A, et al. The importance of early treatment with tranexamic acid in bleeding trauma patients: an exploratory analysis of the CRASH-2 randomised controlled trial. Lancet. 2011;377:1096-1101.

 

ILLUSTRATIVE CASE

You are working in the emergency department (ED) of a rural hospital when a 26-year-old man is brought in with multiple injuries sustained in a high-speed collision and motor vehicle rollover. The nearest trauma center is more than an hour away. Should you administer TXA to the patient before transferring him?

Trauma is a leading cause of death among those younger than 40 years, and in 30% of such fatalities, hemorrhage is the cause.2 Tranexamic acid (TXA) minimizes blood loss by inhibiting lysine binding sites on plasminogen, thereby preventing the conversion of plasminogen to plasmin. This inhibits fibrinolysis and reduces clot breakdown, resulting in a reduction in bleeding.3

TXA has a proven track record
TXA is not new. It has been used to minimize blood loss associated with surgery for decades.4 A retrospective cohort study involving recent military engagements in Afghanistan showed a reduction in both coagulopathy and mortality in trauma patients who were given TXA.5 The CRASH-2 randomized controlled trial (RCT), initially published in 20106 and further analyzed in the study detailed below,1 was the first extensive multicenter trial to evaluate the use of TXA in civilian trauma care.

STUDY SUMMARY: TXA saves lives— within a 3-hour window

CRASH-2 studied the early administration of TXA in adult trauma patients in 274 hospitals in 40 countries.1,6 Patients (N = 20,211) were enrolled if the treating physician judged them to have or be at risk for significant hemorrhage and were randomized to either TXA or placebo, administered in identical-looking packs. Within 8 hours of injury, participants received a 1-g intravenous (IV) loading dose of either TXA or placebo over 10 minutes; a 1-g infusion over 8 hours followed. Patients and study staff were blinded to the treatment groups.

The primary outcome was overall mortality in the 4 weeks after injury. Secondary outcomes included vascular occlusive events (myocardial infarction, stroke, pulmonary embolism, and deep venous thrombosis), major surgical intervention, quantity of blood transfusion (if any), and cause of death (bleeding, vascular occlusion, multi-organ failure, head injury, or other cause). In this analysis, the researchers considered the effect of TXA on mortality based on time to administration of treatment after injury, severity of blood loss as assessed by systolic blood pressure, Glasgow coma scale score, and type of injury. All analyses were intention to treat, and follow-up was 99.6%.

TXA reduced all-cause mortality in the first month after trauma (relative risk [RR] = 0.91; 95% confidence interval [CI], 0.85-0.97; P = .0035; number needed to treat [NNT] = 68). There were 3076 deaths from all causes in both groups, 35% of which were the result of bleeding. Among patients who received TXA, the overall risk of death due to bleeding was 4.9%, vs 5.4% in the placebo group (RR = 0.85; 95% CI, 0.76-0.96; P = .0077; NNT = 119).

After 3 hours, TXA may do more harm than good
For those treated with TXA within the first hour of injury, the risk of death due to bleeding was 5.3%, vs 7.7% for the placebo group (RR = 0.68; 95% CI, 0.57-0.82; P<.0001; NNT = 41). Giving TXA between one and 3 hours of injury also reduced the risk of death due to bleeding, to 4.8% vs 6.1% for the placebo group (RR = 0.79; 95% CI, 0.64-0.97; P = .03; NNT = 77).

TXA administered more than 3 hours after injury, however, appeared to increase the risk of death due to bleeding, to 4.4% compared with 3.1% for the placebo group (RR = 1.44; 95% CI, 1.12-1.84; P = .004; number needed to harm = 77). The researchers found no evidence that TXA’s effect on death due to bleeding varied on the basis of systolic pressure, Glasgow coma score, or type of injury.

The rate of occlusive vascular events over the 4-week study period was similar in both groups (RR = 0.84; 95% CI, 0.68-1.02; P = .08). Of note, the rate of myocardial infarction was reduced by TXA (RR = 0.64; 95% CI, 0.42-0.97; P = .035; NNT = 504).

 

 

 

WHAT’S NEW: Greater emphasis on TXA timing

Current practice for the treatment of traumatic hemorrhage includes fluid resuscitation and the administration of blood products. This analysis of the CRASH-2 refines our understanding of TXA, revealing that the earlier it is given after injury, the better the outcome. A 2011 Cochrane review found only one other small RCT (N = 240), which had findings consistent with the CRASH-2 results.7

TXA is easy to administer and to store and does not require refrigeration or reconstitution prior to administration. TXA has been included in both the US and British Army trauma protocols.8 In addition, TXA is used by National Health Service ambulances in the United Kingdom, and given to all adults and teenagers who incur major traumatic injury.8

CAVEATS: Potential for thromboembolic events, need for high time sensitivity

Because the enrollment criteria for the study were based entirely on clinical findings, there may have been some participants who were not actively bleeding. However, this would have been true for both the treatment and placebo groups and, if anything, would have diluted the effects of TXA.

There was no increase in vaso-occlusive events in the CRASH-2 study. However, some studies of TXA have found an increase in instances of pulmonary embolism, deep vein thrombosis, and ureteral obstruction in patients with genitourinary bleeding.3

This analysis showed that early administration of TXA is the key to its success—and highlighted the importance of avoiding giving it more than 3 hours after traumatic injury. Although most of the 40 countries in which the CRASH-2 study was conducted have less well developed trauma systems than those in the United States, a subgroup analysis of patients in Europe, North America, and Australia (n = 1960) still showed a mortality benefit (RR = 0.63; 95% CI, 0.42-0.94).8

CHALLENGES TO IMPLEMENTATION: Bringing TXA into the mainstream

The acceptance of TXA in trauma care guidelines may be one of the biggest barriers to its use. Currently, the American College of Surgeons does not include the use of TXA in its Advanced Trauma Life Support manual.9

Given the short time window for its benefit, TXA may be most appropriate in the prehospital setting. However, there are no studies of its use in this setting. Lack of knowledge and access are also barriers in the emergency setting, as many ED clinicians, particularly in rural settings, may not yet have access to TXA. Physicians in the United Kingdom have tried a variety of methods, including the unorthodox use of comic books targeted to health care providers,10 in an effort to get the word out.

Acknowledgement

The PURLs Surveillance System was supported in part by Grant Number UL1RR024999 from the National Center for Research Resources, a Clinical Translational Science Award to the University of Chicago. The content is solely the responsibility of the authors and does not necessarily represent the official views of the National Center for Research Resources or the National Institutes of Health.

References

1. CRASH-2 collaborators; Roberts I, Shakur H, Afolabi A, et al. The importance of early treatment with tranexamic acid in bleeding trauma patients: an exploratory analysis of the CRASH-2 randomised controlled trial. Lancet. 2011;377:1096-1101.

2. Kauvar DS, Lefering R, Wade CE. Impact of hemorrhage on trauma outcome: an overview of epidemiology, clinical presentations, and therapeutic considerations. J Trauma. 2006;60(suppl):S3-S11.

3. Cap AP, Baer DG, Orman JA, et al. Tranexamic acid for trauma patients: a critical review of the literature. J Trauma. 2011;71(suppl):S9-S14.

4. Ker K, Edwards P, Perel P, et al. Effect of tranexamic acid on surgical bleeding: systematic review and cumulative meta-analysis. BMJ. 2012;344:e3054.-

5. Morrison JJ, Dubose JJ, Rasmussen TE, et al. Military Application of Tranexamic Acid in Trauma Emergency Resuscitation (MATTERs) Study. Arch Surg. 2012;147:113-119.

6. Shakur H, Roberts I, Bautista R, et al. Effects of tranexamic acid on death, vascular occlusive events, and blood transfusion in trauma patients with significant haemorrhage (CRASH-2): a randomised, placebo-controlled trial. Lancet. 2010;376:23-32.

7. Roberts I, Shakur H, Ker K, et al. Antifibrinolytic drugs for acute traumatic injury. Cochrane Database Syst Rev. 2011;(1):CD004896.-

8. Ker K, Kiriya J, Perel P, et al. Avoidable mortality from giving tranexamic acid to bleeding trauma patients: an estimation based on WHO mortality data, a systematic literature review and data from the CRASH-2 trial. BMC Emerg Med. 2012;12:3.-

9. American College of Surgeons, Committee on Trauma. Advanced Trauma Life Support (ATLS) Student Course Manual. 9th ed. Chicago, IL: American College of Surgeons; 2012.

10. Roberts I, Vieceli E, Duffield P. TXA. London, UK: London School of Hygiene & Tropical Medicine; 2012. Available at: http://download.thelancet.com/flatcontentassets/pdfs/TXA-manga.pdf. Accessed April 23, 2013.

References

1. CRASH-2 collaborators; Roberts I, Shakur H, Afolabi A, et al. The importance of early treatment with tranexamic acid in bleeding trauma patients: an exploratory analysis of the CRASH-2 randomised controlled trial. Lancet. 2011;377:1096-1101.

2. Kauvar DS, Lefering R, Wade CE. Impact of hemorrhage on trauma outcome: an overview of epidemiology, clinical presentations, and therapeutic considerations. J Trauma. 2006;60(suppl):S3-S11.

3. Cap AP, Baer DG, Orman JA, et al. Tranexamic acid for trauma patients: a critical review of the literature. J Trauma. 2011;71(suppl):S9-S14.

4. Ker K, Edwards P, Perel P, et al. Effect of tranexamic acid on surgical bleeding: systematic review and cumulative meta-analysis. BMJ. 2012;344:e3054.-

5. Morrison JJ, Dubose JJ, Rasmussen TE, et al. Military Application of Tranexamic Acid in Trauma Emergency Resuscitation (MATTERs) Study. Arch Surg. 2012;147:113-119.

6. Shakur H, Roberts I, Bautista R, et al. Effects of tranexamic acid on death, vascular occlusive events, and blood transfusion in trauma patients with significant haemorrhage (CRASH-2): a randomised, placebo-controlled trial. Lancet. 2010;376:23-32.

7. Roberts I, Shakur H, Ker K, et al. Antifibrinolytic drugs for acute traumatic injury. Cochrane Database Syst Rev. 2011;(1):CD004896.-

8. Ker K, Kiriya J, Perel P, et al. Avoidable mortality from giving tranexamic acid to bleeding trauma patients: an estimation based on WHO mortality data, a systematic literature review and data from the CRASH-2 trial. BMC Emerg Med. 2012;12:3.-

9. American College of Surgeons, Committee on Trauma. Advanced Trauma Life Support (ATLS) Student Course Manual. 9th ed. Chicago, IL: American College of Surgeons; 2012.

10. Roberts I, Vieceli E, Duffield P. TXA. London, UK: London School of Hygiene & Tropical Medicine; 2012. Available at: http://download.thelancet.com/flatcontentassets/pdfs/TXA-manga.pdf. Accessed April 23, 2013.

Issue
The Journal of Family Practice - 62(5)
Issue
The Journal of Family Practice - 62(5)
Page Number
E4-E6
Page Number
E4-E6
Publications
Publications
Article Type
Display Headline
Trauma care—don’t delay with TXA
Display Headline
Trauma care—don’t delay with TXA
Legacy Keywords
Jack C. Wells; MD; MHA; James J. Stevermer; MD; MSPH; tranexamic acid; TXA; CRASH-2 randomized controlled trial; reduced all-cause mortality; TXA timing; within 3 hours; occlusive vascular events; PURLs
Legacy Keywords
Jack C. Wells; MD; MHA; James J. Stevermer; MD; MSPH; tranexamic acid; TXA; CRASH-2 randomized controlled trial; reduced all-cause mortality; TXA timing; within 3 hours; occlusive vascular events; PURLs
Sections
PURLs Copyright

Copyright © 2013 The Family Physicians Inquiries Network. All rights reserved.

Disallow All Ads
Alternative CME
Use ProPublica
Hide sidebar & use full width
render the right sidebar.
Conference Recap Checkbox
Not Conference Recap
Clinical Edge
Display the Slideshow in this Article
Article PDF Media
Media Files

Suspect carpal tunnel? Try this

Article Type
Changed
Tue, 07/14/2020 - 14:25
Display Headline
Suspect carpal tunnel? Try this
PRACTICE CHANGER

For best results, use the modified Phalen’s test (MPT) rather than the traditional Phalen’s when you suspect carpal tunnel syndrome (CTS).1

1. Bilkis S, Loveman DM, Eldridge JA, et al. Modified Phalen’s test as an aid in diagnosing carpal tunnel syndrome. Arthritis Care Res. 2012;64:287-289.

STRENGTH OF RECOMMENDATION

B: Based on a single diagnostic cohort study.

 

ILLUSTRATIVE CASE

A 60-year-old assembly line worker reports bilateral hand numbness and tingling that frequently awaken her at night. What is the best office test to determine if she has CTS?

CTS is one of the most common causes of disability in the United States.2 Among patients with hand paresthesias, one in 5 has CTS.2 Factory workers whose jobs involve repetitive hand movements, females, and the elderly are at increased risk.3 If left untreated, the symptoms are likely to become constant, with thenar muscle wasting and weakness.

Traditional diagnostic test has only 50% sensitivity
In the traditional Phalen’s test (TPT)—commonly used in an office setting—the patient holds his or her wrists in a position of fixed flexion for one minute. The onset of paresthesias is considered a positive result.

The TPT was found in the study reported here to be 100% specific;1 however, other studies have found a wider range of specificity (33%-86%).4 The TPT has a sensitivity of only 50%, which increases the risk that cases of CTS will be missed. This is an important consideration because establishing a diagnosis early in the course of CTS has been shown to minimize disability.5

STUDY SUMMARY: Modified Phalen’s has higher sensitivity

Bilkis et al developed a modified Phalen’s test (MPT) and compared it with the TPT, as well as with electrodiagnostic studies (EDS)—the gold standard for CTS diagnosis. The MPT begins with the TPT position and adds sensory testing with a Semmes-Weinstein 2.83-unit monofilament.

See how the modified Phalen’s test is done


Courtesy of Clinically Relevant Technologies

The filament is applied perpendicular to the palmar and lateral surface of each distal finger 3 times, with enough pressure to bend the monofilament. In this study, the test was considered positive if the patient did not feel the monofilament in any finger along the distribution of the median nerve. The MPT was negative if the patient correctly reported being touched along this distribution. The fifth, or “pinkie,” finger, which is less likely to be affected by CTS, was used as a control.

Participants in the study were adult patients—mostly women between the ages of 27 and 88 years—at a neurology clinic. Exclusion criteria included cervical radiculopathy, a history of stroke, diabetes mellitus, and concomitant neck injury. A total of 66 hands (and 37 participants) underwent TPT and MPT testing by trained examiners, followed by EDS to confirm the findings.

EDS found evidence of CTS in 46 of the 66 hands studied. The MPT correctly identified 39 of the 46, while the TPT correctly identified 23. Both the traditional and the modified Phalen’s were found to be 100% specific, but the sensitivity of the MPT was 85% (95% confidence interval [CI], 71%-93%), compared with 50% (95% CI, 35%-65%) for the TPT.

 

 

 

WHAT’S NEW: Better results can be achieved in seconds

The addition of monofilament testing to the TPT increases the sensitivity in identifying CTS. The MPT is simple to learn (watch the video on jfponline.com) and, based on our observations, adds only about 10 to 15 seconds to the clinical exam.

CAVEATS: Modification is untested in primary care

A diagnosis of CTS is rarely made on the basis of one test, but rather on a set of signs, symptoms, and physical exam maneuvers. The added value of the MPT needs to be evaluated in the larger context of the comprehensive clinical examination for CTS.6

Notably, the study participants were seen in a neurology clinic, which suggests that they may have had more advanced CTS than typical primary care patients. That would help explain the 100% specificity of both the traditional and modified tests reported by the researchers. The sensitivity of the MPT may therefore be lower in a family physician’s office because the spectrum of disease may be wider. Another study is needed to evaluate the performance of the MPT in a primary care setting.

The monofilament used (Semmes-Weinstein 2.83) is not the same as the typical 5.07 (10-g) monofilament used in diabetic foot screenings. Using this heavier monofilament with a stronger pressure point would likely decrease the sensitivity of the MPT.

CHALLENGES TO IMPLEMENTATION: Taking the time, obtaining the monofilament

Additional time to obtain the correct monofilament and administer the MPT are the key challenges to implementation.

Acknowledgement

The PURLs Surveillance System was supported in part by Grant Number UL1RR024999 from the National Center for Research Resources, a Clinical Translational Science Award to the University of Chicago. The content is solely the responsibility of the authors and does not necessarily represent the official views of the National Center for Research Resources or the National Institutes of Health.

Files
References

1. Bilkis S, Loveman DM, Eldridge JA, et al. Modified Phalen’s test as an aid in diagnosing carpal tunnel syndrome. Arthritis Care Res. 2012;64:287-289.

2. Atroshi I, Gummesson C, Johnsson R, et al. Prevalence of carpal tunnel syndrome in a general population. JAMA. 1999;282:153-158.

3. National Institute of Neurological Disorders and Stroke. Carpal tunnel syndrome fact sheet. National Institutes of Health. July 2012. Available at http://www.ninds.nih.gov/disorders/carpal_tunnel/detail_carpal_tunnel.htm. Accessed April 15, 2013.

4. McGee SR. Evidence-Based Physical Diagnosis. 3rd ed. Philadelphia, Pa: Saunders; 2012:chap 62.

5. Daniell WE, Fulton-Kehoe D, Franklin GM. Work-related carpal tunnel syndrome in Washington State workers’ compensation: utilization of surgery and the duration of lost work. Am J Ind Med. 2009;52:931-942.

6. D’Arcy CA, McGee S. Does this patient have carpal tunnel syndrome? JAMA. 2000;282:3110-3117.

Article PDF
Author and Disclosure Information

Sonia Oyola, MD
The University of Chicago

Kohar Jones, MD
The University of Chicago

Goutham Rao, MD
The University of Chicago

PURLs EDITOR
James J. Stevermer, MD, MSPH
Department of Family Medicine, University of Missouri-Columbia

Issue
The Journal of Family Practice - 62(5)
Publications
Topics
Page Number
253-254
Legacy Keywords
Sonia Oyola; MD; Kohar Jones; MD; Goutham Rao; MD; carpal tunnel syndrome; modified Phalen's test; CTS; paresthesias; thenar muscle wasting; electrodiagnostic studies; monofilament; PURLs
Sections
Files
Files
Author and Disclosure Information

Sonia Oyola, MD
The University of Chicago

Kohar Jones, MD
The University of Chicago

Goutham Rao, MD
The University of Chicago

PURLs EDITOR
James J. Stevermer, MD, MSPH
Department of Family Medicine, University of Missouri-Columbia

Author and Disclosure Information

Sonia Oyola, MD
The University of Chicago

Kohar Jones, MD
The University of Chicago

Goutham Rao, MD
The University of Chicago

PURLs EDITOR
James J. Stevermer, MD, MSPH
Department of Family Medicine, University of Missouri-Columbia

Article PDF
Article PDF
PRACTICE CHANGER

For best results, use the modified Phalen’s test (MPT) rather than the traditional Phalen’s when you suspect carpal tunnel syndrome (CTS).1

1. Bilkis S, Loveman DM, Eldridge JA, et al. Modified Phalen’s test as an aid in diagnosing carpal tunnel syndrome. Arthritis Care Res. 2012;64:287-289.

STRENGTH OF RECOMMENDATION

B: Based on a single diagnostic cohort study.

 

ILLUSTRATIVE CASE

A 60-year-old assembly line worker reports bilateral hand numbness and tingling that frequently awaken her at night. What is the best office test to determine if she has CTS?

CTS is one of the most common causes of disability in the United States.2 Among patients with hand paresthesias, one in 5 has CTS.2 Factory workers whose jobs involve repetitive hand movements, females, and the elderly are at increased risk.3 If left untreated, the symptoms are likely to become constant, with thenar muscle wasting and weakness.

Traditional diagnostic test has only 50% sensitivity
In the traditional Phalen’s test (TPT)—commonly used in an office setting—the patient holds his or her wrists in a position of fixed flexion for one minute. The onset of paresthesias is considered a positive result.

The TPT was found in the study reported here to be 100% specific;1 however, other studies have found a wider range of specificity (33%-86%).4 The TPT has a sensitivity of only 50%, which increases the risk that cases of CTS will be missed. This is an important consideration because establishing a diagnosis early in the course of CTS has been shown to minimize disability.5

STUDY SUMMARY: Modified Phalen’s has higher sensitivity

Bilkis et al developed a modified Phalen’s test (MPT) and compared it with the TPT, as well as with electrodiagnostic studies (EDS)—the gold standard for CTS diagnosis. The MPT begins with the TPT position and adds sensory testing with a Semmes-Weinstein 2.83-unit monofilament.

See how the modified Phalen’s test is done


Courtesy of Clinically Relevant Technologies

The filament is applied perpendicular to the palmar and lateral surface of each distal finger 3 times, with enough pressure to bend the monofilament. In this study, the test was considered positive if the patient did not feel the monofilament in any finger along the distribution of the median nerve. The MPT was negative if the patient correctly reported being touched along this distribution. The fifth, or “pinkie,” finger, which is less likely to be affected by CTS, was used as a control.

Participants in the study were adult patients—mostly women between the ages of 27 and 88 years—at a neurology clinic. Exclusion criteria included cervical radiculopathy, a history of stroke, diabetes mellitus, and concomitant neck injury. A total of 66 hands (and 37 participants) underwent TPT and MPT testing by trained examiners, followed by EDS to confirm the findings.

EDS found evidence of CTS in 46 of the 66 hands studied. The MPT correctly identified 39 of the 46, while the TPT correctly identified 23. Both the traditional and the modified Phalen’s were found to be 100% specific, but the sensitivity of the MPT was 85% (95% confidence interval [CI], 71%-93%), compared with 50% (95% CI, 35%-65%) for the TPT.

 

 

 

WHAT’S NEW: Better results can be achieved in seconds

The addition of monofilament testing to the TPT increases the sensitivity in identifying CTS. The MPT is simple to learn (watch the video on jfponline.com) and, based on our observations, adds only about 10 to 15 seconds to the clinical exam.

CAVEATS: Modification is untested in primary care

A diagnosis of CTS is rarely made on the basis of one test, but rather on a set of signs, symptoms, and physical exam maneuvers. The added value of the MPT needs to be evaluated in the larger context of the comprehensive clinical examination for CTS.6

Notably, the study participants were seen in a neurology clinic, which suggests that they may have had more advanced CTS than typical primary care patients. That would help explain the 100% specificity of both the traditional and modified tests reported by the researchers. The sensitivity of the MPT may therefore be lower in a family physician’s office because the spectrum of disease may be wider. Another study is needed to evaluate the performance of the MPT in a primary care setting.

The monofilament used (Semmes-Weinstein 2.83) is not the same as the typical 5.07 (10-g) monofilament used in diabetic foot screenings. Using this heavier monofilament with a stronger pressure point would likely decrease the sensitivity of the MPT.

CHALLENGES TO IMPLEMENTATION: Taking the time, obtaining the monofilament

Additional time to obtain the correct monofilament and administer the MPT are the key challenges to implementation.

Acknowledgement

The PURLs Surveillance System was supported in part by Grant Number UL1RR024999 from the National Center for Research Resources, a Clinical Translational Science Award to the University of Chicago. The content is solely the responsibility of the authors and does not necessarily represent the official views of the National Center for Research Resources or the National Institutes of Health.

PRACTICE CHANGER

For best results, use the modified Phalen’s test (MPT) rather than the traditional Phalen’s when you suspect carpal tunnel syndrome (CTS).1

1. Bilkis S, Loveman DM, Eldridge JA, et al. Modified Phalen’s test as an aid in diagnosing carpal tunnel syndrome. Arthritis Care Res. 2012;64:287-289.

STRENGTH OF RECOMMENDATION

B: Based on a single diagnostic cohort study.

 

ILLUSTRATIVE CASE

A 60-year-old assembly line worker reports bilateral hand numbness and tingling that frequently awaken her at night. What is the best office test to determine if she has CTS?

CTS is one of the most common causes of disability in the United States.2 Among patients with hand paresthesias, one in 5 has CTS.2 Factory workers whose jobs involve repetitive hand movements, females, and the elderly are at increased risk.3 If left untreated, the symptoms are likely to become constant, with thenar muscle wasting and weakness.

Traditional diagnostic test has only 50% sensitivity
In the traditional Phalen’s test (TPT)—commonly used in an office setting—the patient holds his or her wrists in a position of fixed flexion for one minute. The onset of paresthesias is considered a positive result.

The TPT was found in the study reported here to be 100% specific;1 however, other studies have found a wider range of specificity (33%-86%).4 The TPT has a sensitivity of only 50%, which increases the risk that cases of CTS will be missed. This is an important consideration because establishing a diagnosis early in the course of CTS has been shown to minimize disability.5

STUDY SUMMARY: Modified Phalen’s has higher sensitivity

Bilkis et al developed a modified Phalen’s test (MPT) and compared it with the TPT, as well as with electrodiagnostic studies (EDS)—the gold standard for CTS diagnosis. The MPT begins with the TPT position and adds sensory testing with a Semmes-Weinstein 2.83-unit monofilament.

See how the modified Phalen’s test is done


Courtesy of Clinically Relevant Technologies

The filament is applied perpendicular to the palmar and lateral surface of each distal finger 3 times, with enough pressure to bend the monofilament. In this study, the test was considered positive if the patient did not feel the monofilament in any finger along the distribution of the median nerve. The MPT was negative if the patient correctly reported being touched along this distribution. The fifth, or “pinkie,” finger, which is less likely to be affected by CTS, was used as a control.

Participants in the study were adult patients—mostly women between the ages of 27 and 88 years—at a neurology clinic. Exclusion criteria included cervical radiculopathy, a history of stroke, diabetes mellitus, and concomitant neck injury. A total of 66 hands (and 37 participants) underwent TPT and MPT testing by trained examiners, followed by EDS to confirm the findings.

EDS found evidence of CTS in 46 of the 66 hands studied. The MPT correctly identified 39 of the 46, while the TPT correctly identified 23. Both the traditional and the modified Phalen’s were found to be 100% specific, but the sensitivity of the MPT was 85% (95% confidence interval [CI], 71%-93%), compared with 50% (95% CI, 35%-65%) for the TPT.

 

 

 

WHAT’S NEW: Better results can be achieved in seconds

The addition of monofilament testing to the TPT increases the sensitivity in identifying CTS. The MPT is simple to learn (watch the video on jfponline.com) and, based on our observations, adds only about 10 to 15 seconds to the clinical exam.

CAVEATS: Modification is untested in primary care

A diagnosis of CTS is rarely made on the basis of one test, but rather on a set of signs, symptoms, and physical exam maneuvers. The added value of the MPT needs to be evaluated in the larger context of the comprehensive clinical examination for CTS.6

Notably, the study participants were seen in a neurology clinic, which suggests that they may have had more advanced CTS than typical primary care patients. That would help explain the 100% specificity of both the traditional and modified tests reported by the researchers. The sensitivity of the MPT may therefore be lower in a family physician’s office because the spectrum of disease may be wider. Another study is needed to evaluate the performance of the MPT in a primary care setting.

The monofilament used (Semmes-Weinstein 2.83) is not the same as the typical 5.07 (10-g) monofilament used in diabetic foot screenings. Using this heavier monofilament with a stronger pressure point would likely decrease the sensitivity of the MPT.

CHALLENGES TO IMPLEMENTATION: Taking the time, obtaining the monofilament

Additional time to obtain the correct monofilament and administer the MPT are the key challenges to implementation.

Acknowledgement

The PURLs Surveillance System was supported in part by Grant Number UL1RR024999 from the National Center for Research Resources, a Clinical Translational Science Award to the University of Chicago. The content is solely the responsibility of the authors and does not necessarily represent the official views of the National Center for Research Resources or the National Institutes of Health.

References

1. Bilkis S, Loveman DM, Eldridge JA, et al. Modified Phalen’s test as an aid in diagnosing carpal tunnel syndrome. Arthritis Care Res. 2012;64:287-289.

2. Atroshi I, Gummesson C, Johnsson R, et al. Prevalence of carpal tunnel syndrome in a general population. JAMA. 1999;282:153-158.

3. National Institute of Neurological Disorders and Stroke. Carpal tunnel syndrome fact sheet. National Institutes of Health. July 2012. Available at http://www.ninds.nih.gov/disorders/carpal_tunnel/detail_carpal_tunnel.htm. Accessed April 15, 2013.

4. McGee SR. Evidence-Based Physical Diagnosis. 3rd ed. Philadelphia, Pa: Saunders; 2012:chap 62.

5. Daniell WE, Fulton-Kehoe D, Franklin GM. Work-related carpal tunnel syndrome in Washington State workers’ compensation: utilization of surgery and the duration of lost work. Am J Ind Med. 2009;52:931-942.

6. D’Arcy CA, McGee S. Does this patient have carpal tunnel syndrome? JAMA. 2000;282:3110-3117.

References

1. Bilkis S, Loveman DM, Eldridge JA, et al. Modified Phalen’s test as an aid in diagnosing carpal tunnel syndrome. Arthritis Care Res. 2012;64:287-289.

2. Atroshi I, Gummesson C, Johnsson R, et al. Prevalence of carpal tunnel syndrome in a general population. JAMA. 1999;282:153-158.

3. National Institute of Neurological Disorders and Stroke. Carpal tunnel syndrome fact sheet. National Institutes of Health. July 2012. Available at http://www.ninds.nih.gov/disorders/carpal_tunnel/detail_carpal_tunnel.htm. Accessed April 15, 2013.

4. McGee SR. Evidence-Based Physical Diagnosis. 3rd ed. Philadelphia, Pa: Saunders; 2012:chap 62.

5. Daniell WE, Fulton-Kehoe D, Franklin GM. Work-related carpal tunnel syndrome in Washington State workers’ compensation: utilization of surgery and the duration of lost work. Am J Ind Med. 2009;52:931-942.

6. D’Arcy CA, McGee S. Does this patient have carpal tunnel syndrome? JAMA. 2000;282:3110-3117.

Issue
The Journal of Family Practice - 62(5)
Issue
The Journal of Family Practice - 62(5)
Page Number
253-254
Page Number
253-254
Publications
Publications
Topics
Article Type
Display Headline
Suspect carpal tunnel? Try this
Display Headline
Suspect carpal tunnel? Try this
Legacy Keywords
Sonia Oyola; MD; Kohar Jones; MD; Goutham Rao; MD; carpal tunnel syndrome; modified Phalen's test; CTS; paresthesias; thenar muscle wasting; electrodiagnostic studies; monofilament; PURLs
Legacy Keywords
Sonia Oyola; MD; Kohar Jones; MD; Goutham Rao; MD; carpal tunnel syndrome; modified Phalen's test; CTS; paresthesias; thenar muscle wasting; electrodiagnostic studies; monofilament; PURLs
Sections
PURLs Copyright

Copyright © 2013 The Family Physicians Inquiries Network. All rights reserved.

Disallow All Ads
Alternative CME
Use ProPublica
Hide sidebar & use full width
render the right sidebar.
Conference Recap Checkbox
Not Conference Recap
Clinical Edge
Display the Slideshow in this Article
Article PDF Media
Media Files

A Safer Way to Prevent VTE Recurrence

Article Type
Changed
Tue, 12/13/2016 - 12:08
Display Headline
A Safer Way to Prevent VTE Recurrence

PRACTICE CHANGER
After patients with unprovoked venous thromboembolism (VTE) complete a 6- to 18-month course of oral anticoagulation therapy, consider a switch to aspirin.1

STRENGTH OF

RECOMMENDATION
A: Based on one well-designed, randomized controlled trial (RCT).

ILLUSTRATIVE CASE
A 62-year-old patient comes to your office for follow-up of a primary unprovoked VTE. He has been on an oral anticoagulant for 12 months. Should he continue anticoagulation therapy despite the increased risk for major bleeding?

Patients who survive VTE—defined as either deep venous thrombosis (DVT) or pulmonary embolism (PE)—are put on anticoagulant therapy to prevent a recurrence, typically for six to 18 months. But about 20% of patients with unprovoked VTE have a recurrence within two years of anticoagulation withdrawal.2 Extending anticoagulation prevents recurrences but increases the risk for bleeding.3

Is aspirin a viable alternative?

Until recently, the efficacy of aspirin for the prevention of recurrent VTE was unknown. Becattini et al1 investigated it in the multicenter RCT detailed in this PURL.

STUDY SUMMARY
Aspirin can prevent recurrence with minimal risk

To determine whether aspirin was a viable alternative to oral anticoagulation, the researchers compared aspirin with placebo in patients with primary unprovoked VTE who had completed a course of oral anticoagulation treatment.

To be considered for the study, patients had to be older than 18 and have had their first-ever objectively confirmed, symptomatic unprovoked proximal DVT PE, or both. They also had to have completed six to 18 months of anticoagulant therapy, with a target international normalized ratio (INR) of 2.0 to 3.0. Exclusion criteria included a history of cancer, clinically significant thrombophilia, atrial fibrillation, and a bleeding event that occurred during the course of anticoagulation therapy.

Becattini et al identified 403 eligible patients. Two weeks after stopping anticoagulation, patients were randomly assigned to receive either aspirin 100 mg/d (n = 205) or placebo (n = 198) for two years. (One patient in the placebo group never received treatment.)

At baseline, there were no significant differences in patient characteristics. All were evaluated every three months in the first year and every six months in the second year.

The primary efficacy outcome was objectively confirmed recurrent VTE. The primary safety outcome was major bleeding, defined as bleeding that occurred in a critical location (eg, intracranial bleeding), was associated with a decrease of hemoglobin of at least 2 g/dL, required a transfusion of two units of whole blood or red blood cells, or was fatal. Overt bleeding, which required medical intervention but did not meet the criteria for major bleeding, was a secondary safety outcome.

Twenty-eight of the 205 patients in the aspirin group experienced a recurrence, compared with 43 of the 197 patients on placebo (6.6% vs 11.2% per year; hazard ratio [HR] = 0.58).

Adverse events were reported by seven patients in the aspirin therapy group and six in the placebo group. One patient in each group experienced major bleeding, and three in each group experienced clinically relevant but nonmajor bleeding.

Withdrawal rates were similar (10 in the group receiving aspirin vs 9 in the group receiving placebo), as were the number of patients who developed new indications for aspirin or anticoagulation therapy or were lost to follow-up.

An analysis adjusted for age, sex, index event (DVT or PE) and duration of initial anticoagulation treatment confirmed that aspirin reduced the risk for recurrence (adjusted HR = 0.53). No association was found between recurrent VTE and duration of anticoagulation therapy (six months vs longer). Nor was there a difference in recurrence rates based on the index event.

WHAT'S NEW
Aspirin has a key role

in preventing recurrence
This study found that for patients with unprovoked VTE who completed a course of oral anticoagulation, aspirin was effective in preventing a recurrence, with no apparent increase in the risk for major bleeding. Protection in year 2 was nearly as great as in year 1.1

CAVEAT
Patients were followed

for just two years
It is unclear whether continuing aspirin therapy beyond two years would continue to confer protection against a VTE recurrence without an increase in adverse effects.

CHALLENGE TO IMPLEMENTATION
Some patients can't tolerate chronic aspirin therapy
Although this study investigated aspirin in a dosage of 100 mg/d, this strength is not readily available in the United States.4 There is no evidence to suggest that the 81-mg strength that is available in this country would provide a diminished antiplatelet effect.

And, as is already customary, patients undergoing chronic aspirin therapy must be monitored for major bleeding, GI irritation, and renal compromise. A few patients will be ineligible for prophylaxis due to a history of intolerance to aspirin or NSAIDs.

 

 

REFERENCES
1. Becattini C, Agnelli G, Schenone A, et al. Aspirin for preventing the recurrence of venous thromboembolism. N Engl J Med. 2012;366:1959-1967.

2. Prandoni P, Lensing AW, Cogo A, et al. The long-term clinical course of acute deep venous thrombosis. Ann Intern Med. 1996;125:1-7.

3. Kearon C, Akl EA, Comerota AJ, et al. Antithrombotic therapy for VTE disease: antithrombotic therapy and prevention of thrombosis. American College of Chest Physicians evidence-based clinical practice guidelines. Chest. 2012;141 (2 suppl):e419S-e494S.

4. Daily Med. Aspirin. dailymed.nlm.nih.gov/dailymed/search.cfm?startswith=aspirin. Accessed September 6, 2012.

Acknowledgement
The PURLs Surveillance System was supported in part by Grant Number UL1RR024999 from the National Center for Research Resources, a Clinical Translational Science Award to the University of Chicago. The content is solely the responsibility of the authors and does not necessarily represent the official views of the National Center for Research Resources or the National Institutes of Health.

Copyright © 2012. The Family Physicians Inquiries Network. All rights reserved.

Reprinted with permission from the Family Physicians Inquiries Network and The Journal of Family Practice. 2012;61:673-674.

Author and Disclosure Information

Altaf Kaiseruddin, MD, Sonia Oyola, MD, Goutham Rao, MD

Issue
Clinician Reviews - 23(4)
Publications
Topics
Page Number
19-20
Legacy Keywords
venous thromboembolism, deep venous thrombosis, pulmonary embolism, anticoagulation, aspirin, antiplatelet effectvenous thromboembolism, deep venous thrombosis, pulmonary embolism, anticoagulation, aspirin, antiplatelet effect
Sections
Author and Disclosure Information

Altaf Kaiseruddin, MD, Sonia Oyola, MD, Goutham Rao, MD

Author and Disclosure Information

Altaf Kaiseruddin, MD, Sonia Oyola, MD, Goutham Rao, MD

PRACTICE CHANGER
After patients with unprovoked venous thromboembolism (VTE) complete a 6- to 18-month course of oral anticoagulation therapy, consider a switch to aspirin.1

STRENGTH OF

RECOMMENDATION
A: Based on one well-designed, randomized controlled trial (RCT).

ILLUSTRATIVE CASE
A 62-year-old patient comes to your office for follow-up of a primary unprovoked VTE. He has been on an oral anticoagulant for 12 months. Should he continue anticoagulation therapy despite the increased risk for major bleeding?

Patients who survive VTE—defined as either deep venous thrombosis (DVT) or pulmonary embolism (PE)—are put on anticoagulant therapy to prevent a recurrence, typically for six to 18 months. But about 20% of patients with unprovoked VTE have a recurrence within two years of anticoagulation withdrawal.2 Extending anticoagulation prevents recurrences but increases the risk for bleeding.3

Is aspirin a viable alternative?

Until recently, the efficacy of aspirin for the prevention of recurrent VTE was unknown. Becattini et al1 investigated it in the multicenter RCT detailed in this PURL.

STUDY SUMMARY
Aspirin can prevent recurrence with minimal risk

To determine whether aspirin was a viable alternative to oral anticoagulation, the researchers compared aspirin with placebo in patients with primary unprovoked VTE who had completed a course of oral anticoagulation treatment.

To be considered for the study, patients had to be older than 18 and have had their first-ever objectively confirmed, symptomatic unprovoked proximal DVT PE, or both. They also had to have completed six to 18 months of anticoagulant therapy, with a target international normalized ratio (INR) of 2.0 to 3.0. Exclusion criteria included a history of cancer, clinically significant thrombophilia, atrial fibrillation, and a bleeding event that occurred during the course of anticoagulation therapy.

Becattini et al identified 403 eligible patients. Two weeks after stopping anticoagulation, patients were randomly assigned to receive either aspirin 100 mg/d (n = 205) or placebo (n = 198) for two years. (One patient in the placebo group never received treatment.)

At baseline, there were no significant differences in patient characteristics. All were evaluated every three months in the first year and every six months in the second year.

The primary efficacy outcome was objectively confirmed recurrent VTE. The primary safety outcome was major bleeding, defined as bleeding that occurred in a critical location (eg, intracranial bleeding), was associated with a decrease of hemoglobin of at least 2 g/dL, required a transfusion of two units of whole blood or red blood cells, or was fatal. Overt bleeding, which required medical intervention but did not meet the criteria for major bleeding, was a secondary safety outcome.

Twenty-eight of the 205 patients in the aspirin group experienced a recurrence, compared with 43 of the 197 patients on placebo (6.6% vs 11.2% per year; hazard ratio [HR] = 0.58).

Adverse events were reported by seven patients in the aspirin therapy group and six in the placebo group. One patient in each group experienced major bleeding, and three in each group experienced clinically relevant but nonmajor bleeding.

Withdrawal rates were similar (10 in the group receiving aspirin vs 9 in the group receiving placebo), as were the number of patients who developed new indications for aspirin or anticoagulation therapy or were lost to follow-up.

An analysis adjusted for age, sex, index event (DVT or PE) and duration of initial anticoagulation treatment confirmed that aspirin reduced the risk for recurrence (adjusted HR = 0.53). No association was found between recurrent VTE and duration of anticoagulation therapy (six months vs longer). Nor was there a difference in recurrence rates based on the index event.

WHAT'S NEW
Aspirin has a key role

in preventing recurrence
This study found that for patients with unprovoked VTE who completed a course of oral anticoagulation, aspirin was effective in preventing a recurrence, with no apparent increase in the risk for major bleeding. Protection in year 2 was nearly as great as in year 1.1

CAVEAT
Patients were followed

for just two years
It is unclear whether continuing aspirin therapy beyond two years would continue to confer protection against a VTE recurrence without an increase in adverse effects.

CHALLENGE TO IMPLEMENTATION
Some patients can't tolerate chronic aspirin therapy
Although this study investigated aspirin in a dosage of 100 mg/d, this strength is not readily available in the United States.4 There is no evidence to suggest that the 81-mg strength that is available in this country would provide a diminished antiplatelet effect.

And, as is already customary, patients undergoing chronic aspirin therapy must be monitored for major bleeding, GI irritation, and renal compromise. A few patients will be ineligible for prophylaxis due to a history of intolerance to aspirin or NSAIDs.

 

 

REFERENCES
1. Becattini C, Agnelli G, Schenone A, et al. Aspirin for preventing the recurrence of venous thromboembolism. N Engl J Med. 2012;366:1959-1967.

2. Prandoni P, Lensing AW, Cogo A, et al. The long-term clinical course of acute deep venous thrombosis. Ann Intern Med. 1996;125:1-7.

3. Kearon C, Akl EA, Comerota AJ, et al. Antithrombotic therapy for VTE disease: antithrombotic therapy and prevention of thrombosis. American College of Chest Physicians evidence-based clinical practice guidelines. Chest. 2012;141 (2 suppl):e419S-e494S.

4. Daily Med. Aspirin. dailymed.nlm.nih.gov/dailymed/search.cfm?startswith=aspirin. Accessed September 6, 2012.

Acknowledgement
The PURLs Surveillance System was supported in part by Grant Number UL1RR024999 from the National Center for Research Resources, a Clinical Translational Science Award to the University of Chicago. The content is solely the responsibility of the authors and does not necessarily represent the official views of the National Center for Research Resources or the National Institutes of Health.

Copyright © 2012. The Family Physicians Inquiries Network. All rights reserved.

Reprinted with permission from the Family Physicians Inquiries Network and The Journal of Family Practice. 2012;61:673-674.

PRACTICE CHANGER
After patients with unprovoked venous thromboembolism (VTE) complete a 6- to 18-month course of oral anticoagulation therapy, consider a switch to aspirin.1

STRENGTH OF

RECOMMENDATION
A: Based on one well-designed, randomized controlled trial (RCT).

ILLUSTRATIVE CASE
A 62-year-old patient comes to your office for follow-up of a primary unprovoked VTE. He has been on an oral anticoagulant for 12 months. Should he continue anticoagulation therapy despite the increased risk for major bleeding?

Patients who survive VTE—defined as either deep venous thrombosis (DVT) or pulmonary embolism (PE)—are put on anticoagulant therapy to prevent a recurrence, typically for six to 18 months. But about 20% of patients with unprovoked VTE have a recurrence within two years of anticoagulation withdrawal.2 Extending anticoagulation prevents recurrences but increases the risk for bleeding.3

Is aspirin a viable alternative?

Until recently, the efficacy of aspirin for the prevention of recurrent VTE was unknown. Becattini et al1 investigated it in the multicenter RCT detailed in this PURL.

STUDY SUMMARY
Aspirin can prevent recurrence with minimal risk

To determine whether aspirin was a viable alternative to oral anticoagulation, the researchers compared aspirin with placebo in patients with primary unprovoked VTE who had completed a course of oral anticoagulation treatment.

To be considered for the study, patients had to be older than 18 and have had their first-ever objectively confirmed, symptomatic unprovoked proximal DVT PE, or both. They also had to have completed six to 18 months of anticoagulant therapy, with a target international normalized ratio (INR) of 2.0 to 3.0. Exclusion criteria included a history of cancer, clinically significant thrombophilia, atrial fibrillation, and a bleeding event that occurred during the course of anticoagulation therapy.

Becattini et al identified 403 eligible patients. Two weeks after stopping anticoagulation, patients were randomly assigned to receive either aspirin 100 mg/d (n = 205) or placebo (n = 198) for two years. (One patient in the placebo group never received treatment.)

At baseline, there were no significant differences in patient characteristics. All were evaluated every three months in the first year and every six months in the second year.

The primary efficacy outcome was objectively confirmed recurrent VTE. The primary safety outcome was major bleeding, defined as bleeding that occurred in a critical location (eg, intracranial bleeding), was associated with a decrease of hemoglobin of at least 2 g/dL, required a transfusion of two units of whole blood or red blood cells, or was fatal. Overt bleeding, which required medical intervention but did not meet the criteria for major bleeding, was a secondary safety outcome.

Twenty-eight of the 205 patients in the aspirin group experienced a recurrence, compared with 43 of the 197 patients on placebo (6.6% vs 11.2% per year; hazard ratio [HR] = 0.58).

Adverse events were reported by seven patients in the aspirin therapy group and six in the placebo group. One patient in each group experienced major bleeding, and three in each group experienced clinically relevant but nonmajor bleeding.

Withdrawal rates were similar (10 in the group receiving aspirin vs 9 in the group receiving placebo), as were the number of patients who developed new indications for aspirin or anticoagulation therapy or were lost to follow-up.

An analysis adjusted for age, sex, index event (DVT or PE) and duration of initial anticoagulation treatment confirmed that aspirin reduced the risk for recurrence (adjusted HR = 0.53). No association was found between recurrent VTE and duration of anticoagulation therapy (six months vs longer). Nor was there a difference in recurrence rates based on the index event.

WHAT'S NEW
Aspirin has a key role

in preventing recurrence
This study found that for patients with unprovoked VTE who completed a course of oral anticoagulation, aspirin was effective in preventing a recurrence, with no apparent increase in the risk for major bleeding. Protection in year 2 was nearly as great as in year 1.1

CAVEAT
Patients were followed

for just two years
It is unclear whether continuing aspirin therapy beyond two years would continue to confer protection against a VTE recurrence without an increase in adverse effects.

CHALLENGE TO IMPLEMENTATION
Some patients can't tolerate chronic aspirin therapy
Although this study investigated aspirin in a dosage of 100 mg/d, this strength is not readily available in the United States.4 There is no evidence to suggest that the 81-mg strength that is available in this country would provide a diminished antiplatelet effect.

And, as is already customary, patients undergoing chronic aspirin therapy must be monitored for major bleeding, GI irritation, and renal compromise. A few patients will be ineligible for prophylaxis due to a history of intolerance to aspirin or NSAIDs.

 

 

REFERENCES
1. Becattini C, Agnelli G, Schenone A, et al. Aspirin for preventing the recurrence of venous thromboembolism. N Engl J Med. 2012;366:1959-1967.

2. Prandoni P, Lensing AW, Cogo A, et al. The long-term clinical course of acute deep venous thrombosis. Ann Intern Med. 1996;125:1-7.

3. Kearon C, Akl EA, Comerota AJ, et al. Antithrombotic therapy for VTE disease: antithrombotic therapy and prevention of thrombosis. American College of Chest Physicians evidence-based clinical practice guidelines. Chest. 2012;141 (2 suppl):e419S-e494S.

4. Daily Med. Aspirin. dailymed.nlm.nih.gov/dailymed/search.cfm?startswith=aspirin. Accessed September 6, 2012.

Acknowledgement
The PURLs Surveillance System was supported in part by Grant Number UL1RR024999 from the National Center for Research Resources, a Clinical Translational Science Award to the University of Chicago. The content is solely the responsibility of the authors and does not necessarily represent the official views of the National Center for Research Resources or the National Institutes of Health.

Copyright © 2012. The Family Physicians Inquiries Network. All rights reserved.

Reprinted with permission from the Family Physicians Inquiries Network and The Journal of Family Practice. 2012;61:673-674.

Issue
Clinician Reviews - 23(4)
Issue
Clinician Reviews - 23(4)
Page Number
19-20
Page Number
19-20
Publications
Publications
Topics
Article Type
Display Headline
A Safer Way to Prevent VTE Recurrence
Display Headline
A Safer Way to Prevent VTE Recurrence
Legacy Keywords
venous thromboembolism, deep venous thrombosis, pulmonary embolism, anticoagulation, aspirin, antiplatelet effectvenous thromboembolism, deep venous thrombosis, pulmonary embolism, anticoagulation, aspirin, antiplatelet effect
Legacy Keywords
venous thromboembolism, deep venous thrombosis, pulmonary embolism, anticoagulation, aspirin, antiplatelet effectvenous thromboembolism, deep venous thrombosis, pulmonary embolism, anticoagulation, aspirin, antiplatelet effect
Sections
Article Source

PURLs Copyright

Inside the Article

Prescribing an Antibiotic? Pair It With Probiotics

Article Type
Changed
Tue, 12/13/2016 - 12:08
Display Headline
Prescribing an Antibiotic? Pair It With Probiotics

PRACTICE CHANGER
Recommend that patients taking antibiotics also take probiotics, which have been found to be effective both for the prevention and treatment of antibiotic-associated diarrhea (AAD).1

STRENGTH OF

RECOMMENDATION
A: Based on a systematic review and meta-analysis of randomized controlled trials (RCTs).

ILLUSTRATIVE CASE
When you prescribe an antibiotic for a 45-year-old patient with Helicobacter pylori, he worries that the medication will cause diarrhea. Should you recommend that he take probiotics?

More than a third of patients taking antibiotics develop AAD,2 and in 17% of cases, AAD is fatal.3,4 Although the diarrhea may be the result of increased gastrointestinal (GI) motility in some cases, a disruption of the GI flora that normally acts as a barrier to infection and aids in the digestion of carbohydrates is a far more common cause.

Morbidity and mortality are high
AAD is associated with several pathogens, including Clostridium difficile, Clostridium perfringens, Klebsiella oxytoca, and Staphylococcus aureus,2 and varies widely in severity. Pseudomembranous colitis secondary to C difficile is the main cause of AAD-related mortality, which more than doubled from 2002 to 2009.3,4 C difficile infections cost the US health care system up to $1.3 billion annually.5 With such high rates of morbidity and mortality and high health care costs associated with AAD, even a small reduction in the number of cases would have a big impact.

Probiotics replenish the natural GI flora with nonpathogenic organisms. A 2006 meta-analysis of 31 RCTs assessing the efficacy of probiotics for both the prevention of AAD and treatment of C difficile found a pooled relative risk (RR) of 0.43 for AAD in the patients taking probiotics.6 However, many of the studies included in that meta-analysis were small. As a result, in 2010, the Society for Healthcare Epidemiology of America (SHEA) and the Infectious Diseases Society of America (IDSA) recommended against the use of probiotics for the prevention of primary C difficile infection, citing a lack of high-quality evidence.7

Nonetheless, that same year, 98% of gastroenterologists surveyed expressed a belief that probiotics had a role in the treatment of GI illness.8 And in 2011, the Third Yale Working Group on Probiotic Use published recommendations for probiotic use based on expert opinion.9 The meta-analysis detailed in this PURL, which included more than 30 trials published since the 2006 meta-analysis, addressed the efficacy of probiotics for prevention and treatment of AAD.

STUDY SUMMARY
Probiotics significantly reduce AAD
Hempel et al reviewed 82 studies and pooled data from 63 RCTs (N = 11,811) to identify the RR of AAD among patients who received probiotics during antibiotic treatment, compared with those who received no probiotics or were given a placebo.1 The studies encompassed a variety of antibiotics, taken alone or in combination, and several probiotics, including Lactobacillus, Bifidobacterium, Saccharomyces, and some combinations.

The outcome: The pooled RR for AAD in the probiotics groups was 0.58, with a number needed to treat of 13. Although the authors reported that the overall quality of the included trials was poor, a sensitivity analysis of the higher-quality studies yielded similar results.

Subgroup analyses by type of probiotic and duration of antibiotic treatment were also consistent with the overall pooled RR. In subgroup analysis by age, a similar decrease in AAD was found among the youngest patients (ages 0 to 17) and those between the ages of 17 and 65. Among patients older than 65—for whom there were just three studies—a nonsignificant decrease in risk was found. Twenty-three of the studies assessed adverse outcomes, of which none were reported.

WHAT'S NEW
A reason to pair antibiotics and probiotics

This meta-analysis reached a similar conclusion to the 2006 meta-analysis: Probiotics appear to be effective in preventing and treating AAD in children and adults receiving a wide variety of antibiotics for a number of conditions. The results were also consistent with those of a new meta-analysis that looked specifically at one pathogen—and found a reduction of 66% in C difficile–associated diarrhea in patients taking probiotics with their antibiotics.10

CAVEATS
Limited data on the safety

of probiotics exist
There was some heterogeneity among the studies in the meta-analysis by Hempel et al,1 and some of the studies were of poor quality. Because of this, the authors used subgroup and sensitivity analysis, which supported their initial conclusion.

Probiotics have generally been considered safe; however, there have been rare reports of sepsis and fungemia associated with probiotic use, especially in immunosuppressed patients.1 Fifty-nine of the included studies did not assess adverse events, which limited the ability of this meta-analysis to assess safety.1 Patients taking probiotics should be monitored for adverse effects.

CHALLENGES TO IMPLEMENTATION
Lack of guidance on dosing and duration
Since probiotics are considered food supplements, health insurance will not cover the cost (which will likely be more than $20 per month). No single probiotic strain has high-quality evidence; however, most of the RCTs included in the meta-analysis used combinations of Lactobacillus species, which are usually found in OTC antidiarrheal probiotic supplements. No standard dose exists, but dose ranges in RCTs are 107 to 1010 colony-forming units per capsule (taken one to three times daily);1 however, product labels have variable accuracy.11 The duration of treatment ranges from one to three weeks—or as long as the patient continues to take antibiotics. .

 

 

REFERENCES
1. Hempel S, Newberry S, Maher A, et al. Probiotics for the prevention and treatment of antibiotic-associated diarrhea. JAMA. 2012; 307:1959-1969.

2. McFarland LV. Antibiotic-associated diarrhea: epidemiology, trends and treatment. Future Microbiol. 2008;3:563-578.

3. Pepin J, Valiquette L, Cossette B. Mortality attributable to nosocomial Clostridium difficile–associated disease during an epidemic caused by a hypervirulent strain in Quebec. CMAJ. 2005;173:1037-1042.

4. Perry A, Dellon E, Lund J, et al. Burden of gastrointestinal disease in the United States: 2012 Update. Gastroenterology. 2012;143: 1179-1187.

5. Dubberke E, Wertheimer A. A review of current literature on the economic burden of Clostridium difficile infection. Infect Control Hosp Epidemiol. 2009;30:57-66.

6. McFarland L. Meta-analysis of probiotics for the prevention of antibiotic associated diarrhea and the treatment of Clostridium difficile disease. Am J Gastroenterol. 2006;101:

812-822.

7. Cohen S, Gerding D, Johnson S, et al. Clinical practice guidelines for Clostridium difficile infection in adults: 2010 update by the Society for Healthcare Epidemiology of America and the Infectious Diseases Society of America. Infect Control Hosp Epidemiol. 2010;31: 431-455.

8. Williams M, Ha C, Ciorba M. Probiotics as therapy in gastroenterology. J Clin Gastroenterol. 2010;44:631-636.

9. Floch M, Walker A, Madsne K, et al. Recommendations for probiotic use—2011 update. J Clin Gastroenterol. 2011;45(suppl):S168-S171.

10. Johnston BC, Ma SS, Goldenberg JZ, et al. Probiotics for the prevention of Clostridium difficile–associated diarrhea: a systematic review and meta-analysis. Ann Intern Med. 2012;157:878-888.

11. Hamilton-Miller J, Shah S. Deficiencies in microbiological quality and labeling of probiotic supplements. Int J Food Microbiol. 2002; 72:175-176.

Acknowledgement
The PURLs Surveillance System was developed with support from Grant Number UL1RR024999 from the National Center for Research Resources, a Clinical Translational Science Award to the University of Chicago. The content is solely the responsibility of the authors and does not necessarily represent the official views of the National Center for Research Resources or the National Institutes of Health.

Copyright © 2012. The Family Physicians Inquiries Network. All rights reserved.

Reprinted with permission from the Family Physicians Inquiries Network and The Journal of Family Practice. 2012;61:673-674.

Author and Disclosure Information

Blake Rodgers, MD, Kate Kirley, MD, Anne Mounsey, MD

Issue
Clinician Reviews - 23(4)
Publications
Page Number
49-50
Legacy Keywords
Blake Rodgers; MD; Kate Kirley; MD; Anne Mounsey; MD; probiotics, antibiotic-associated diarrhea, antibiotic course, Helicobacter pylori, Clostridium difficile, pseudomembranous colitis, Lactobacillus, Bifidobacterium, Saccharomycesprobiotics, antibiotic-associated diarrhea, antibiotic course, Helicobacter pylori, Clostridium difficile, pseudomembranous colitis, Lactobacillus, Bifidobacterium, Saccharomyces; PURLs
Sections
Author and Disclosure Information

Blake Rodgers, MD, Kate Kirley, MD, Anne Mounsey, MD

Author and Disclosure Information

Blake Rodgers, MD, Kate Kirley, MD, Anne Mounsey, MD

PRACTICE CHANGER
Recommend that patients taking antibiotics also take probiotics, which have been found to be effective both for the prevention and treatment of antibiotic-associated diarrhea (AAD).1

STRENGTH OF

RECOMMENDATION
A: Based on a systematic review and meta-analysis of randomized controlled trials (RCTs).

ILLUSTRATIVE CASE
When you prescribe an antibiotic for a 45-year-old patient with Helicobacter pylori, he worries that the medication will cause diarrhea. Should you recommend that he take probiotics?

More than a third of patients taking antibiotics develop AAD,2 and in 17% of cases, AAD is fatal.3,4 Although the diarrhea may be the result of increased gastrointestinal (GI) motility in some cases, a disruption of the GI flora that normally acts as a barrier to infection and aids in the digestion of carbohydrates is a far more common cause.

Morbidity and mortality are high
AAD is associated with several pathogens, including Clostridium difficile, Clostridium perfringens, Klebsiella oxytoca, and Staphylococcus aureus,2 and varies widely in severity. Pseudomembranous colitis secondary to C difficile is the main cause of AAD-related mortality, which more than doubled from 2002 to 2009.3,4 C difficile infections cost the US health care system up to $1.3 billion annually.5 With such high rates of morbidity and mortality and high health care costs associated with AAD, even a small reduction in the number of cases would have a big impact.

Probiotics replenish the natural GI flora with nonpathogenic organisms. A 2006 meta-analysis of 31 RCTs assessing the efficacy of probiotics for both the prevention of AAD and treatment of C difficile found a pooled relative risk (RR) of 0.43 for AAD in the patients taking probiotics.6 However, many of the studies included in that meta-analysis were small. As a result, in 2010, the Society for Healthcare Epidemiology of America (SHEA) and the Infectious Diseases Society of America (IDSA) recommended against the use of probiotics for the prevention of primary C difficile infection, citing a lack of high-quality evidence.7

Nonetheless, that same year, 98% of gastroenterologists surveyed expressed a belief that probiotics had a role in the treatment of GI illness.8 And in 2011, the Third Yale Working Group on Probiotic Use published recommendations for probiotic use based on expert opinion.9 The meta-analysis detailed in this PURL, which included more than 30 trials published since the 2006 meta-analysis, addressed the efficacy of probiotics for prevention and treatment of AAD.

STUDY SUMMARY
Probiotics significantly reduce AAD
Hempel et al reviewed 82 studies and pooled data from 63 RCTs (N = 11,811) to identify the RR of AAD among patients who received probiotics during antibiotic treatment, compared with those who received no probiotics or were given a placebo.1 The studies encompassed a variety of antibiotics, taken alone or in combination, and several probiotics, including Lactobacillus, Bifidobacterium, Saccharomyces, and some combinations.

The outcome: The pooled RR for AAD in the probiotics groups was 0.58, with a number needed to treat of 13. Although the authors reported that the overall quality of the included trials was poor, a sensitivity analysis of the higher-quality studies yielded similar results.

Subgroup analyses by type of probiotic and duration of antibiotic treatment were also consistent with the overall pooled RR. In subgroup analysis by age, a similar decrease in AAD was found among the youngest patients (ages 0 to 17) and those between the ages of 17 and 65. Among patients older than 65—for whom there were just three studies—a nonsignificant decrease in risk was found. Twenty-three of the studies assessed adverse outcomes, of which none were reported.

WHAT'S NEW
A reason to pair antibiotics and probiotics

This meta-analysis reached a similar conclusion to the 2006 meta-analysis: Probiotics appear to be effective in preventing and treating AAD in children and adults receiving a wide variety of antibiotics for a number of conditions. The results were also consistent with those of a new meta-analysis that looked specifically at one pathogen—and found a reduction of 66% in C difficile–associated diarrhea in patients taking probiotics with their antibiotics.10

CAVEATS
Limited data on the safety

of probiotics exist
There was some heterogeneity among the studies in the meta-analysis by Hempel et al,1 and some of the studies were of poor quality. Because of this, the authors used subgroup and sensitivity analysis, which supported their initial conclusion.

Probiotics have generally been considered safe; however, there have been rare reports of sepsis and fungemia associated with probiotic use, especially in immunosuppressed patients.1 Fifty-nine of the included studies did not assess adverse events, which limited the ability of this meta-analysis to assess safety.1 Patients taking probiotics should be monitored for adverse effects.

CHALLENGES TO IMPLEMENTATION
Lack of guidance on dosing and duration
Since probiotics are considered food supplements, health insurance will not cover the cost (which will likely be more than $20 per month). No single probiotic strain has high-quality evidence; however, most of the RCTs included in the meta-analysis used combinations of Lactobacillus species, which are usually found in OTC antidiarrheal probiotic supplements. No standard dose exists, but dose ranges in RCTs are 107 to 1010 colony-forming units per capsule (taken one to three times daily);1 however, product labels have variable accuracy.11 The duration of treatment ranges from one to three weeks—or as long as the patient continues to take antibiotics. .

 

 

REFERENCES
1. Hempel S, Newberry S, Maher A, et al. Probiotics for the prevention and treatment of antibiotic-associated diarrhea. JAMA. 2012; 307:1959-1969.

2. McFarland LV. Antibiotic-associated diarrhea: epidemiology, trends and treatment. Future Microbiol. 2008;3:563-578.

3. Pepin J, Valiquette L, Cossette B. Mortality attributable to nosocomial Clostridium difficile–associated disease during an epidemic caused by a hypervirulent strain in Quebec. CMAJ. 2005;173:1037-1042.

4. Perry A, Dellon E, Lund J, et al. Burden of gastrointestinal disease in the United States: 2012 Update. Gastroenterology. 2012;143: 1179-1187.

5. Dubberke E, Wertheimer A. A review of current literature on the economic burden of Clostridium difficile infection. Infect Control Hosp Epidemiol. 2009;30:57-66.

6. McFarland L. Meta-analysis of probiotics for the prevention of antibiotic associated diarrhea and the treatment of Clostridium difficile disease. Am J Gastroenterol. 2006;101:

812-822.

7. Cohen S, Gerding D, Johnson S, et al. Clinical practice guidelines for Clostridium difficile infection in adults: 2010 update by the Society for Healthcare Epidemiology of America and the Infectious Diseases Society of America. Infect Control Hosp Epidemiol. 2010;31: 431-455.

8. Williams M, Ha C, Ciorba M. Probiotics as therapy in gastroenterology. J Clin Gastroenterol. 2010;44:631-636.

9. Floch M, Walker A, Madsne K, et al. Recommendations for probiotic use—2011 update. J Clin Gastroenterol. 2011;45(suppl):S168-S171.

10. Johnston BC, Ma SS, Goldenberg JZ, et al. Probiotics for the prevention of Clostridium difficile–associated diarrhea: a systematic review and meta-analysis. Ann Intern Med. 2012;157:878-888.

11. Hamilton-Miller J, Shah S. Deficiencies in microbiological quality and labeling of probiotic supplements. Int J Food Microbiol. 2002; 72:175-176.

Acknowledgement
The PURLs Surveillance System was developed with support from Grant Number UL1RR024999 from the National Center for Research Resources, a Clinical Translational Science Award to the University of Chicago. The content is solely the responsibility of the authors and does not necessarily represent the official views of the National Center for Research Resources or the National Institutes of Health.

Copyright © 2012. The Family Physicians Inquiries Network. All rights reserved.

Reprinted with permission from the Family Physicians Inquiries Network and The Journal of Family Practice. 2012;61:673-674.

PRACTICE CHANGER
Recommend that patients taking antibiotics also take probiotics, which have been found to be effective both for the prevention and treatment of antibiotic-associated diarrhea (AAD).1

STRENGTH OF

RECOMMENDATION
A: Based on a systematic review and meta-analysis of randomized controlled trials (RCTs).

ILLUSTRATIVE CASE
When you prescribe an antibiotic for a 45-year-old patient with Helicobacter pylori, he worries that the medication will cause diarrhea. Should you recommend that he take probiotics?

More than a third of patients taking antibiotics develop AAD,2 and in 17% of cases, AAD is fatal.3,4 Although the diarrhea may be the result of increased gastrointestinal (GI) motility in some cases, a disruption of the GI flora that normally acts as a barrier to infection and aids in the digestion of carbohydrates is a far more common cause.

Morbidity and mortality are high
AAD is associated with several pathogens, including Clostridium difficile, Clostridium perfringens, Klebsiella oxytoca, and Staphylococcus aureus,2 and varies widely in severity. Pseudomembranous colitis secondary to C difficile is the main cause of AAD-related mortality, which more than doubled from 2002 to 2009.3,4 C difficile infections cost the US health care system up to $1.3 billion annually.5 With such high rates of morbidity and mortality and high health care costs associated with AAD, even a small reduction in the number of cases would have a big impact.

Probiotics replenish the natural GI flora with nonpathogenic organisms. A 2006 meta-analysis of 31 RCTs assessing the efficacy of probiotics for both the prevention of AAD and treatment of C difficile found a pooled relative risk (RR) of 0.43 for AAD in the patients taking probiotics.6 However, many of the studies included in that meta-analysis were small. As a result, in 2010, the Society for Healthcare Epidemiology of America (SHEA) and the Infectious Diseases Society of America (IDSA) recommended against the use of probiotics for the prevention of primary C difficile infection, citing a lack of high-quality evidence.7

Nonetheless, that same year, 98% of gastroenterologists surveyed expressed a belief that probiotics had a role in the treatment of GI illness.8 And in 2011, the Third Yale Working Group on Probiotic Use published recommendations for probiotic use based on expert opinion.9 The meta-analysis detailed in this PURL, which included more than 30 trials published since the 2006 meta-analysis, addressed the efficacy of probiotics for prevention and treatment of AAD.

STUDY SUMMARY
Probiotics significantly reduce AAD
Hempel et al reviewed 82 studies and pooled data from 63 RCTs (N = 11,811) to identify the RR of AAD among patients who received probiotics during antibiotic treatment, compared with those who received no probiotics or were given a placebo.1 The studies encompassed a variety of antibiotics, taken alone or in combination, and several probiotics, including Lactobacillus, Bifidobacterium, Saccharomyces, and some combinations.

The outcome: The pooled RR for AAD in the probiotics groups was 0.58, with a number needed to treat of 13. Although the authors reported that the overall quality of the included trials was poor, a sensitivity analysis of the higher-quality studies yielded similar results.

Subgroup analyses by type of probiotic and duration of antibiotic treatment were also consistent with the overall pooled RR. In subgroup analysis by age, a similar decrease in AAD was found among the youngest patients (ages 0 to 17) and those between the ages of 17 and 65. Among patients older than 65—for whom there were just three studies—a nonsignificant decrease in risk was found. Twenty-three of the studies assessed adverse outcomes, of which none were reported.

WHAT'S NEW
A reason to pair antibiotics and probiotics

This meta-analysis reached a similar conclusion to the 2006 meta-analysis: Probiotics appear to be effective in preventing and treating AAD in children and adults receiving a wide variety of antibiotics for a number of conditions. The results were also consistent with those of a new meta-analysis that looked specifically at one pathogen—and found a reduction of 66% in C difficile–associated diarrhea in patients taking probiotics with their antibiotics.10

CAVEATS
Limited data on the safety

of probiotics exist
There was some heterogeneity among the studies in the meta-analysis by Hempel et al,1 and some of the studies were of poor quality. Because of this, the authors used subgroup and sensitivity analysis, which supported their initial conclusion.

Probiotics have generally been considered safe; however, there have been rare reports of sepsis and fungemia associated with probiotic use, especially in immunosuppressed patients.1 Fifty-nine of the included studies did not assess adverse events, which limited the ability of this meta-analysis to assess safety.1 Patients taking probiotics should be monitored for adverse effects.

CHALLENGES TO IMPLEMENTATION
Lack of guidance on dosing and duration
Since probiotics are considered food supplements, health insurance will not cover the cost (which will likely be more than $20 per month). No single probiotic strain has high-quality evidence; however, most of the RCTs included in the meta-analysis used combinations of Lactobacillus species, which are usually found in OTC antidiarrheal probiotic supplements. No standard dose exists, but dose ranges in RCTs are 107 to 1010 colony-forming units per capsule (taken one to three times daily);1 however, product labels have variable accuracy.11 The duration of treatment ranges from one to three weeks—or as long as the patient continues to take antibiotics. .

 

 

REFERENCES
1. Hempel S, Newberry S, Maher A, et al. Probiotics for the prevention and treatment of antibiotic-associated diarrhea. JAMA. 2012; 307:1959-1969.

2. McFarland LV. Antibiotic-associated diarrhea: epidemiology, trends and treatment. Future Microbiol. 2008;3:563-578.

3. Pepin J, Valiquette L, Cossette B. Mortality attributable to nosocomial Clostridium difficile–associated disease during an epidemic caused by a hypervirulent strain in Quebec. CMAJ. 2005;173:1037-1042.

4. Perry A, Dellon E, Lund J, et al. Burden of gastrointestinal disease in the United States: 2012 Update. Gastroenterology. 2012;143: 1179-1187.

5. Dubberke E, Wertheimer A. A review of current literature on the economic burden of Clostridium difficile infection. Infect Control Hosp Epidemiol. 2009;30:57-66.

6. McFarland L. Meta-analysis of probiotics for the prevention of antibiotic associated diarrhea and the treatment of Clostridium difficile disease. Am J Gastroenterol. 2006;101:

812-822.

7. Cohen S, Gerding D, Johnson S, et al. Clinical practice guidelines for Clostridium difficile infection in adults: 2010 update by the Society for Healthcare Epidemiology of America and the Infectious Diseases Society of America. Infect Control Hosp Epidemiol. 2010;31: 431-455.

8. Williams M, Ha C, Ciorba M. Probiotics as therapy in gastroenterology. J Clin Gastroenterol. 2010;44:631-636.

9. Floch M, Walker A, Madsne K, et al. Recommendations for probiotic use—2011 update. J Clin Gastroenterol. 2011;45(suppl):S168-S171.

10. Johnston BC, Ma SS, Goldenberg JZ, et al. Probiotics for the prevention of Clostridium difficile–associated diarrhea: a systematic review and meta-analysis. Ann Intern Med. 2012;157:878-888.

11. Hamilton-Miller J, Shah S. Deficiencies in microbiological quality and labeling of probiotic supplements. Int J Food Microbiol. 2002; 72:175-176.

Acknowledgement
The PURLs Surveillance System was developed with support from Grant Number UL1RR024999 from the National Center for Research Resources, a Clinical Translational Science Award to the University of Chicago. The content is solely the responsibility of the authors and does not necessarily represent the official views of the National Center for Research Resources or the National Institutes of Health.

Copyright © 2012. The Family Physicians Inquiries Network. All rights reserved.

Reprinted with permission from the Family Physicians Inquiries Network and The Journal of Family Practice. 2012;61:673-674.

Issue
Clinician Reviews - 23(4)
Issue
Clinician Reviews - 23(4)
Page Number
49-50
Page Number
49-50
Publications
Publications
Article Type
Display Headline
Prescribing an Antibiotic? Pair It With Probiotics
Display Headline
Prescribing an Antibiotic? Pair It With Probiotics
Legacy Keywords
Blake Rodgers; MD; Kate Kirley; MD; Anne Mounsey; MD; probiotics, antibiotic-associated diarrhea, antibiotic course, Helicobacter pylori, Clostridium difficile, pseudomembranous colitis, Lactobacillus, Bifidobacterium, Saccharomycesprobiotics, antibiotic-associated diarrhea, antibiotic course, Helicobacter pylori, Clostridium difficile, pseudomembranous colitis, Lactobacillus, Bifidobacterium, Saccharomyces; PURLs
Legacy Keywords
Blake Rodgers; MD; Kate Kirley; MD; Anne Mounsey; MD; probiotics, antibiotic-associated diarrhea, antibiotic course, Helicobacter pylori, Clostridium difficile, pseudomembranous colitis, Lactobacillus, Bifidobacterium, Saccharomycesprobiotics, antibiotic-associated diarrhea, antibiotic course, Helicobacter pylori, Clostridium difficile, pseudomembranous colitis, Lactobacillus, Bifidobacterium, Saccharomyces; PURLs
Sections
Article Source

PURLs Copyright

Inside the Article

Optimal timing for peripheral IV replacement?

Article Type
Changed
Thu, 07/16/2020 - 14:47
Display Headline
Optimal timing for peripheral IV replacement?
PRACTICE CHANGER

Replace peripheral IV catheters as needed, rather than on a routine basis.1

STRENGTH OF RECOMMENDATION

A: Based on a randomized equivalence trial.

Rickard CM, Webster J, Wallis MC, et al. Routine versus clinically indicated replacement of peripheral intravenous catheters: a randomised controlled equivalence trial. Lancet. 2012;380:1066-1074.

 

ILLUSTRATIVE CASE

On Day 4 of her hospitalization for a wound infection requiring IV antibiotics, a 45-year-old patient is told by her nurse that her IV catheter must be replaced. It’s hospital policy, the RN says, to replace the catheter every 96 hours. The patient is afraid of needles and is not eager to have her catheter replaced every few days. Is it really necessary to replace the IV, she wants to know.

Each year, nearly 200 million peripheral IV catheters are placed in patients in hospitals throughout the United States.2 Many of the catheters need to be replaced due to phlebitis, infiltration, pain, or swelling at the IV site, but the rate of bloodstream infections associated with peripheral IVs is just 0.5 per 1000 catheter days.2

Timing of replacement is “unresolved”
The Centers for Disease Control and Prevention (CDC)’s 2011 guidelines state that it is not necessary to replace peripheral IV catheters in adults more than every 72 to 96 hours,3 but the CDC does not specify when the catheters should be replaced. For adult patients, the recommendation that a catheter be replaced only for clinical indications is an “unresolved issue,” according to the guidelines. For children, however, replacement only when clinically indicated is recommended by the CDC. Many hospitals have protocols that require replacement of IV catheters every 72 to 96 hours, regardless of clinical indication.

A 2008 study of 755 inpatients compared clinically indicated replacement of IV catheters with routine replacement and found no significant differences in phlebitis and infiltration rates between the 2 groups (38% vs 33%, respectively; relative risk [RR]=1.15; 95% confidence interval [CI], 0.95-1.40).4

A 2010 trial randomized 362 hospitalized patients to routine or clinically indicated replacement of peripheral IV lines, with median dwell times of 71 and 85 hours, respectively. There was no significant difference in rates of phlebitis between the routine replacement (7%) and clinically indicated (10%) groups (RR=1.44; 95% CI, 0.71-2.89; P=.34). No local infections or IV-related bloodstream infections occurred in either group.5

A 2010 Cochrane review included 5 randomized controlled trials (with a total of 3408 patients) that compared rates of suspected catheter-related phlebitis in patients whose catheters were routinely replaced with those in the clinically indicated group. The reviewers found no significant increase in phlebitis in the clinically indicated group (9%) vs the routine replacement group (7.2%) (odds ratio=1.24; 95% CI, 0.97-1.60; P=.09).6

Each of these studies had either a relatively small sample size or wide confidence intervals, raising the possibility of missing a real increase in infection due to inadequate statistical power. The study summarized here addressed these concerns.

STUDY SUMMARY: Forgoing routine replacement does not increase risk

Rickard et al1 conducted a multicenter, nonblinded randomized equivalence trial to determine whether routine or clinically indicated removal reduced rates of infection. In the routine group, catheters were replaced every 72 to 96 hours. In the clinically indicated group, catheters were replaced in instances of phlebitis, infiltration, occlusion, accidental removal, or suspected infection related to the catheter.

Participants (N=3283) were inpatients on medical and surgical units who had IV catheters in place and were expected to need treatment for at least 4 days. Individuals whose IV catheters had been placed in an emergency were excluded, as were those who had a known bloodstream infection or who were not expected to have the IV in place for at least 24 hours. Follow-up data were available for all participants.

 

 

 

The primary outcome was phlebitis, with a prespecified equivalence margin of 3%. In both groups, phlebitis occurred in 7% of patients (RR=1.06; 95% CI, 0.83-1.36; P=.64). The absolute risk difference was 0.41% (95% CI, -1.33 to 2.15), which was within the equivalence margin.

The mean IV catheter dwell time was 70 hours in the routine replacement group and 99 hours in the clinically indicated group. Nine patients in the routine replacement group developed bloodstream infections, vs 4 patients in the clinically indicated group (hazard ratio=0.46; 95% CI, 0.14-1.48; P=.19). One patient in the routine placement group had a catheter-related bloodstream infection; no one in the clinically indicated group did. The mortality rate for each group was <1%.

WHAT’S NEW: We can order clinically indicated IV replacement with confidence

The findings of this equivalence trial support prior studies and add greater statistical power. The results suggest that we can recommend clinically indicated replacement of peripheral IV catheters without increasing the rate of phlebitis. Implementing clinically indicated replacement of IVs could decrease hospital costs and improve patient satisfaction.

CAVEATS: Findings do not apply to patients with bacteremia

Patients with known bacteremia were excluded from this study, and the results are therefore not generalizable to this population.

The nonblinded nature of this trial raises the possibility of observer and reporting bias. However, measures were taken to minimize the potential for bias. A structured outcome assessment was used to standardize reporting of signs of phlebitis. Both patients’ pain scores and nurses’ assessments of the IV sites were used to determine whether an infection was present, and the investigators and research nurses were not involved in the removal of the IV catheters.

This study did not report on the daily maintenance protocols the investigators used for the peripheral IVs. The study was conducted in hospitals in Australia, and we don’t know whether the protocols used in that country are similar to standard protocols in US hospitals.

CHALLENGES TO IMPLEMENTATION: Changing hospital protocols won’t be easy

Implementing the findings of this study will require that physicians work with the nursing staff and administrators to create and implement new protocols for assessing peripheral IV catheters in hospitals with routine IV replacement policies already in place. It would be necessary to ensure that all clinicians who place peripheral IV catheters are taught the clinical signs of phlebitis and are using a standardized protocol. That said, we think that this is a worthwhile change to achieve the long-term benefits of fewer unnecessary IV catheter replacements.

Acknowledgement

The PURLs Surveillance System was supported in part by Grant Number UL1RR024999 from the National Center for Research Resources, a Clinical Translational Science Award to the University of Chicago. The content is solely the responsibility of the authors and does not necessarily represent the official views of the National Center for Research Resources or the National Institutes of Health.

Files
References

1. Rickard CM, Webster J, Wallis MC, et al. Routine versus clinically indicated replacement of peripheral intravenous catheters: a randomised controlled equivalence trial. Lancet. 2012;380:1066-1074.

2. Maki DG, Kluger DM, Crnich CJ. The risk of bloodstream infection in adults with different intravascular devices: a systematic review of 200 published prospective studies. Mayo Clin Proc. 2006;81:1159-1171.

3. Centers for Disease Control and Prevention. 2011 guidelines for the prevention of intravascular catheter-related infections. Available at: http://www.cdc.gov/hicpac/BSI/-BSI-guidelines-2011.html. Accessed March 13, 2013.

4. Webster J, Clarke S, Paterson D, et al. Routine care of peripheral intravenous catheters versus clinically indicated replacement: randomised controlled trial. BMJ. 2008;337:a339.-

5. Rickard CM, McCann D, Munnings J, et al. Routine resite of peripheral intravenous devices every 3 days did not reduce complications compared with clinically indicated resite: a randomised controlled trial. BMC Med. 2010;8:53.-

6. Webster J, Osborne S, Rickard C, et al. Clinically-indicated replacement versus routine replacement of peripheral venous catheters. Cochrane Database Syst Rev. 2010;(3):CD007798.-

Article PDF
Author and Disclosure Information

Dionna Brown, MD
The University of Chicago

Kate Rowland, MD, MS
The University of Chicago

PURLs EDITOR
Bernard Ewigman, MD, MSPH
The University of Chicago

Issue
The Journal of Family Practice - 62(4)
Publications
Topics
Page Number
200-202
Legacy Keywords
Dionna Brown; MD; Kate Rowland; MD; MS; peripheral IV catheters; IV replacement; clinically indicated replacement; routine replacement; 72 to 96 hours; interval; Cochrane review; PURLs
Sections
Files
Files
Author and Disclosure Information

Dionna Brown, MD
The University of Chicago

Kate Rowland, MD, MS
The University of Chicago

PURLs EDITOR
Bernard Ewigman, MD, MSPH
The University of Chicago

Author and Disclosure Information

Dionna Brown, MD
The University of Chicago

Kate Rowland, MD, MS
The University of Chicago

PURLs EDITOR
Bernard Ewigman, MD, MSPH
The University of Chicago

Article PDF
Article PDF
PRACTICE CHANGER

Replace peripheral IV catheters as needed, rather than on a routine basis.1

STRENGTH OF RECOMMENDATION

A: Based on a randomized equivalence trial.

Rickard CM, Webster J, Wallis MC, et al. Routine versus clinically indicated replacement of peripheral intravenous catheters: a randomised controlled equivalence trial. Lancet. 2012;380:1066-1074.

 

ILLUSTRATIVE CASE

On Day 4 of her hospitalization for a wound infection requiring IV antibiotics, a 45-year-old patient is told by her nurse that her IV catheter must be replaced. It’s hospital policy, the RN says, to replace the catheter every 96 hours. The patient is afraid of needles and is not eager to have her catheter replaced every few days. Is it really necessary to replace the IV, she wants to know.

Each year, nearly 200 million peripheral IV catheters are placed in patients in hospitals throughout the United States.2 Many of the catheters need to be replaced due to phlebitis, infiltration, pain, or swelling at the IV site, but the rate of bloodstream infections associated with peripheral IVs is just 0.5 per 1000 catheter days.2

Timing of replacement is “unresolved”
The Centers for Disease Control and Prevention (CDC)’s 2011 guidelines state that it is not necessary to replace peripheral IV catheters in adults more than every 72 to 96 hours,3 but the CDC does not specify when the catheters should be replaced. For adult patients, the recommendation that a catheter be replaced only for clinical indications is an “unresolved issue,” according to the guidelines. For children, however, replacement only when clinically indicated is recommended by the CDC. Many hospitals have protocols that require replacement of IV catheters every 72 to 96 hours, regardless of clinical indication.

A 2008 study of 755 inpatients compared clinically indicated replacement of IV catheters with routine replacement and found no significant differences in phlebitis and infiltration rates between the 2 groups (38% vs 33%, respectively; relative risk [RR]=1.15; 95% confidence interval [CI], 0.95-1.40).4

A 2010 trial randomized 362 hospitalized patients to routine or clinically indicated replacement of peripheral IV lines, with median dwell times of 71 and 85 hours, respectively. There was no significant difference in rates of phlebitis between the routine replacement (7%) and clinically indicated (10%) groups (RR=1.44; 95% CI, 0.71-2.89; P=.34). No local infections or IV-related bloodstream infections occurred in either group.5

A 2010 Cochrane review included 5 randomized controlled trials (with a total of 3408 patients) that compared rates of suspected catheter-related phlebitis in patients whose catheters were routinely replaced with those in the clinically indicated group. The reviewers found no significant increase in phlebitis in the clinically indicated group (9%) vs the routine replacement group (7.2%) (odds ratio=1.24; 95% CI, 0.97-1.60; P=.09).6

Each of these studies had either a relatively small sample size or wide confidence intervals, raising the possibility of missing a real increase in infection due to inadequate statistical power. The study summarized here addressed these concerns.

STUDY SUMMARY: Forgoing routine replacement does not increase risk

Rickard et al1 conducted a multicenter, nonblinded randomized equivalence trial to determine whether routine or clinically indicated removal reduced rates of infection. In the routine group, catheters were replaced every 72 to 96 hours. In the clinically indicated group, catheters were replaced in instances of phlebitis, infiltration, occlusion, accidental removal, or suspected infection related to the catheter.

Participants (N=3283) were inpatients on medical and surgical units who had IV catheters in place and were expected to need treatment for at least 4 days. Individuals whose IV catheters had been placed in an emergency were excluded, as were those who had a known bloodstream infection or who were not expected to have the IV in place for at least 24 hours. Follow-up data were available for all participants.

 

 

 

The primary outcome was phlebitis, with a prespecified equivalence margin of 3%. In both groups, phlebitis occurred in 7% of patients (RR=1.06; 95% CI, 0.83-1.36; P=.64). The absolute risk difference was 0.41% (95% CI, -1.33 to 2.15), which was within the equivalence margin.

The mean IV catheter dwell time was 70 hours in the routine replacement group and 99 hours in the clinically indicated group. Nine patients in the routine replacement group developed bloodstream infections, vs 4 patients in the clinically indicated group (hazard ratio=0.46; 95% CI, 0.14-1.48; P=.19). One patient in the routine placement group had a catheter-related bloodstream infection; no one in the clinically indicated group did. The mortality rate for each group was <1%.

WHAT’S NEW: We can order clinically indicated IV replacement with confidence

The findings of this equivalence trial support prior studies and add greater statistical power. The results suggest that we can recommend clinically indicated replacement of peripheral IV catheters without increasing the rate of phlebitis. Implementing clinically indicated replacement of IVs could decrease hospital costs and improve patient satisfaction.

CAVEATS: Findings do not apply to patients with bacteremia

Patients with known bacteremia were excluded from this study, and the results are therefore not generalizable to this population.

The nonblinded nature of this trial raises the possibility of observer and reporting bias. However, measures were taken to minimize the potential for bias. A structured outcome assessment was used to standardize reporting of signs of phlebitis. Both patients’ pain scores and nurses’ assessments of the IV sites were used to determine whether an infection was present, and the investigators and research nurses were not involved in the removal of the IV catheters.

This study did not report on the daily maintenance protocols the investigators used for the peripheral IVs. The study was conducted in hospitals in Australia, and we don’t know whether the protocols used in that country are similar to standard protocols in US hospitals.

CHALLENGES TO IMPLEMENTATION: Changing hospital protocols won’t be easy

Implementing the findings of this study will require that physicians work with the nursing staff and administrators to create and implement new protocols for assessing peripheral IV catheters in hospitals with routine IV replacement policies already in place. It would be necessary to ensure that all clinicians who place peripheral IV catheters are taught the clinical signs of phlebitis and are using a standardized protocol. That said, we think that this is a worthwhile change to achieve the long-term benefits of fewer unnecessary IV catheter replacements.

Acknowledgement

The PURLs Surveillance System was supported in part by Grant Number UL1RR024999 from the National Center for Research Resources, a Clinical Translational Science Award to the University of Chicago. The content is solely the responsibility of the authors and does not necessarily represent the official views of the National Center for Research Resources or the National Institutes of Health.

PRACTICE CHANGER

Replace peripheral IV catheters as needed, rather than on a routine basis.1

STRENGTH OF RECOMMENDATION

A: Based on a randomized equivalence trial.

Rickard CM, Webster J, Wallis MC, et al. Routine versus clinically indicated replacement of peripheral intravenous catheters: a randomised controlled equivalence trial. Lancet. 2012;380:1066-1074.

 

ILLUSTRATIVE CASE

On Day 4 of her hospitalization for a wound infection requiring IV antibiotics, a 45-year-old patient is told by her nurse that her IV catheter must be replaced. It’s hospital policy, the RN says, to replace the catheter every 96 hours. The patient is afraid of needles and is not eager to have her catheter replaced every few days. Is it really necessary to replace the IV, she wants to know.

Each year, nearly 200 million peripheral IV catheters are placed in patients in hospitals throughout the United States.2 Many of the catheters need to be replaced due to phlebitis, infiltration, pain, or swelling at the IV site, but the rate of bloodstream infections associated with peripheral IVs is just 0.5 per 1000 catheter days.2

Timing of replacement is “unresolved”
The Centers for Disease Control and Prevention (CDC)’s 2011 guidelines state that it is not necessary to replace peripheral IV catheters in adults more than every 72 to 96 hours,3 but the CDC does not specify when the catheters should be replaced. For adult patients, the recommendation that a catheter be replaced only for clinical indications is an “unresolved issue,” according to the guidelines. For children, however, replacement only when clinically indicated is recommended by the CDC. Many hospitals have protocols that require replacement of IV catheters every 72 to 96 hours, regardless of clinical indication.

A 2008 study of 755 inpatients compared clinically indicated replacement of IV catheters with routine replacement and found no significant differences in phlebitis and infiltration rates between the 2 groups (38% vs 33%, respectively; relative risk [RR]=1.15; 95% confidence interval [CI], 0.95-1.40).4

A 2010 trial randomized 362 hospitalized patients to routine or clinically indicated replacement of peripheral IV lines, with median dwell times of 71 and 85 hours, respectively. There was no significant difference in rates of phlebitis between the routine replacement (7%) and clinically indicated (10%) groups (RR=1.44; 95% CI, 0.71-2.89; P=.34). No local infections or IV-related bloodstream infections occurred in either group.5

A 2010 Cochrane review included 5 randomized controlled trials (with a total of 3408 patients) that compared rates of suspected catheter-related phlebitis in patients whose catheters were routinely replaced with those in the clinically indicated group. The reviewers found no significant increase in phlebitis in the clinically indicated group (9%) vs the routine replacement group (7.2%) (odds ratio=1.24; 95% CI, 0.97-1.60; P=.09).6

Each of these studies had either a relatively small sample size or wide confidence intervals, raising the possibility of missing a real increase in infection due to inadequate statistical power. The study summarized here addressed these concerns.

STUDY SUMMARY: Forgoing routine replacement does not increase risk

Rickard et al1 conducted a multicenter, nonblinded randomized equivalence trial to determine whether routine or clinically indicated removal reduced rates of infection. In the routine group, catheters were replaced every 72 to 96 hours. In the clinically indicated group, catheters were replaced in instances of phlebitis, infiltration, occlusion, accidental removal, or suspected infection related to the catheter.

Participants (N=3283) were inpatients on medical and surgical units who had IV catheters in place and were expected to need treatment for at least 4 days. Individuals whose IV catheters had been placed in an emergency were excluded, as were those who had a known bloodstream infection or who were not expected to have the IV in place for at least 24 hours. Follow-up data were available for all participants.

 

 

 

The primary outcome was phlebitis, with a prespecified equivalence margin of 3%. In both groups, phlebitis occurred in 7% of patients (RR=1.06; 95% CI, 0.83-1.36; P=.64). The absolute risk difference was 0.41% (95% CI, -1.33 to 2.15), which was within the equivalence margin.

The mean IV catheter dwell time was 70 hours in the routine replacement group and 99 hours in the clinically indicated group. Nine patients in the routine replacement group developed bloodstream infections, vs 4 patients in the clinically indicated group (hazard ratio=0.46; 95% CI, 0.14-1.48; P=.19). One patient in the routine placement group had a catheter-related bloodstream infection; no one in the clinically indicated group did. The mortality rate for each group was <1%.

WHAT’S NEW: We can order clinically indicated IV replacement with confidence

The findings of this equivalence trial support prior studies and add greater statistical power. The results suggest that we can recommend clinically indicated replacement of peripheral IV catheters without increasing the rate of phlebitis. Implementing clinically indicated replacement of IVs could decrease hospital costs and improve patient satisfaction.

CAVEATS: Findings do not apply to patients with bacteremia

Patients with known bacteremia were excluded from this study, and the results are therefore not generalizable to this population.

The nonblinded nature of this trial raises the possibility of observer and reporting bias. However, measures were taken to minimize the potential for bias. A structured outcome assessment was used to standardize reporting of signs of phlebitis. Both patients’ pain scores and nurses’ assessments of the IV sites were used to determine whether an infection was present, and the investigators and research nurses were not involved in the removal of the IV catheters.

This study did not report on the daily maintenance protocols the investigators used for the peripheral IVs. The study was conducted in hospitals in Australia, and we don’t know whether the protocols used in that country are similar to standard protocols in US hospitals.

CHALLENGES TO IMPLEMENTATION: Changing hospital protocols won’t be easy

Implementing the findings of this study will require that physicians work with the nursing staff and administrators to create and implement new protocols for assessing peripheral IV catheters in hospitals with routine IV replacement policies already in place. It would be necessary to ensure that all clinicians who place peripheral IV catheters are taught the clinical signs of phlebitis and are using a standardized protocol. That said, we think that this is a worthwhile change to achieve the long-term benefits of fewer unnecessary IV catheter replacements.

Acknowledgement

The PURLs Surveillance System was supported in part by Grant Number UL1RR024999 from the National Center for Research Resources, a Clinical Translational Science Award to the University of Chicago. The content is solely the responsibility of the authors and does not necessarily represent the official views of the National Center for Research Resources or the National Institutes of Health.

References

1. Rickard CM, Webster J, Wallis MC, et al. Routine versus clinically indicated replacement of peripheral intravenous catheters: a randomised controlled equivalence trial. Lancet. 2012;380:1066-1074.

2. Maki DG, Kluger DM, Crnich CJ. The risk of bloodstream infection in adults with different intravascular devices: a systematic review of 200 published prospective studies. Mayo Clin Proc. 2006;81:1159-1171.

3. Centers for Disease Control and Prevention. 2011 guidelines for the prevention of intravascular catheter-related infections. Available at: http://www.cdc.gov/hicpac/BSI/-BSI-guidelines-2011.html. Accessed March 13, 2013.

4. Webster J, Clarke S, Paterson D, et al. Routine care of peripheral intravenous catheters versus clinically indicated replacement: randomised controlled trial. BMJ. 2008;337:a339.-

5. Rickard CM, McCann D, Munnings J, et al. Routine resite of peripheral intravenous devices every 3 days did not reduce complications compared with clinically indicated resite: a randomised controlled trial. BMC Med. 2010;8:53.-

6. Webster J, Osborne S, Rickard C, et al. Clinically-indicated replacement versus routine replacement of peripheral venous catheters. Cochrane Database Syst Rev. 2010;(3):CD007798.-

References

1. Rickard CM, Webster J, Wallis MC, et al. Routine versus clinically indicated replacement of peripheral intravenous catheters: a randomised controlled equivalence trial. Lancet. 2012;380:1066-1074.

2. Maki DG, Kluger DM, Crnich CJ. The risk of bloodstream infection in adults with different intravascular devices: a systematic review of 200 published prospective studies. Mayo Clin Proc. 2006;81:1159-1171.

3. Centers for Disease Control and Prevention. 2011 guidelines for the prevention of intravascular catheter-related infections. Available at: http://www.cdc.gov/hicpac/BSI/-BSI-guidelines-2011.html. Accessed March 13, 2013.

4. Webster J, Clarke S, Paterson D, et al. Routine care of peripheral intravenous catheters versus clinically indicated replacement: randomised controlled trial. BMJ. 2008;337:a339.-

5. Rickard CM, McCann D, Munnings J, et al. Routine resite of peripheral intravenous devices every 3 days did not reduce complications compared with clinically indicated resite: a randomised controlled trial. BMC Med. 2010;8:53.-

6. Webster J, Osborne S, Rickard C, et al. Clinically-indicated replacement versus routine replacement of peripheral venous catheters. Cochrane Database Syst Rev. 2010;(3):CD007798.-

Issue
The Journal of Family Practice - 62(4)
Issue
The Journal of Family Practice - 62(4)
Page Number
200-202
Page Number
200-202
Publications
Publications
Topics
Article Type
Display Headline
Optimal timing for peripheral IV replacement?
Display Headline
Optimal timing for peripheral IV replacement?
Legacy Keywords
Dionna Brown; MD; Kate Rowland; MD; MS; peripheral IV catheters; IV replacement; clinically indicated replacement; routine replacement; 72 to 96 hours; interval; Cochrane review; PURLs
Legacy Keywords
Dionna Brown; MD; Kate Rowland; MD; MS; peripheral IV catheters; IV replacement; clinically indicated replacement; routine replacement; 72 to 96 hours; interval; Cochrane review; PURLs
Sections
PURLs Copyright

Copyright © 2013 The Family Physicians Inquiries Network. All rights reserved.

Disallow All Ads
Alternative CME
Use ProPublica
Hide sidebar & use full width
render the right sidebar.
Conference Recap Checkbox
Not Conference Recap
Clinical Edge
Display the Slideshow in this Article
Article PDF Media
Media Files